German Language and Culture - Rollins...

123
German Language and Culture Practice Exam and Notes Effective Fall 2011

Transcript of German Language and Culture - Rollins...

German Language and CulturePractice Exam and Notes

Effective Fall 2011

01443-87558 APSP German Practice Exam • InDS4 • Fonts: Minion Pro Serifa, Helvetica, Times, Universal • D1 2/23/11 RI64612 • D1a 2/28/11 RI64612 • D1b 3/1/11 RI64612 • D1c 3/11/11 RI64612 • D2 3/15/11 RI64612 • D2a 3/16/11 RI64612 • D3 3/17/11 RI64612

German Language and Culture2

About the College Board®

The College Board is a mission-driven not-for-profit organization that connects students to college success and opportunity. Founded in 1900, the College Board was created to expand access to higher education. Today, the membership association is made up of more than 5,900 of the world’s leading educational institutions and is dedicated to promoting excellence and equity in education. Each year, the College Board helps more than seven million students prepare for a successful transition to college through programs and services in college readiness and college success — including the SAT® and the Advanced Placement Program®. The organization also serves the education community through research and advocacy on behalf of students, educators and schools.

For further information visit www.collegeboard.org.

AP Equity and Access PolicyThe College Board strongly encourages educators to make equitable access a guiding principle for their AP programs by giving all willing and academically prepared students the opportunity to participate in AP. We encourage the elimination of barriers that restrict access to AP for students from ethnic, racial and socioeconomic groups that have been traditionally underserved. Schools should make every effort to ensure their AP classes reflect the diversity of their student population. The College Board also believes that all students should have access to academically challenging course work before they enroll in AP classes, which can prepare them for AP success. It is only through a commitment to equitable preparation and access that true equity and excellence can be achieved.

AP German Language and Culture Practice ExamMaterials included in this Practice Exam may not reflect the current AP Course Description and exam in this subject, and teachers are advised to take this into account as they use these materials to support their instruction of students. Please visit AP Central® (apcentral.collegeboard.com) to determine whether a more recent Course and Exam Description PDF is available.

© 2011 The College Board. College Board, ACCUPLACER, Advanced Placement Program, AP, SAT, SpringBoard and the acorn logo are registered trademarks of the College Board. inspiring minds is a trademark owned by the College Board. PSAT/NMSQT is a registered trademark of the College Board and National Merit Scholarship Corporation. All other products and services may be trademarks of their respective owners. (Visit The College Board on the Web: www.collegeboard.org.)

01443-87558 APSP German Practice Exam • InDS4 • Fonts: Minion Pro Serifa, Helvetica, Times, Universal • D1 2/23/11 RI64612 • D1a 2/28/11 RI64612 • D1b 3/1/11 RI64612 • D1c 3/11/11 RI64612 • D2 3/15/11 RI64612 • D2a 3/16/11 RI64612 • D3 3/17/11 RI64612

German Language and Culture 3

Contents

Introduction . . . . . . . . . . . . . . . . . . . . . . . . . . . . . . . . . . . . . . . . . . . . . . . . . . . . . . . 4

I. PracticeExamExam Content and Format . . . . . . . . . . . . . . . . . . . . . . . . . . . . . . . . . . . . 7Administering the Practice Exam . . . . . . . . . . . . . . . . . . . . . . . . . . . . . . 7Blank Answer Sheet . . . . . . . . . . . . . . . . . . . . . . . . . . . . . . . . . . . . . . . . . . 10AP® German Language and Culture Practice Exam . . . . . . . . . . . . . . . 11AP® German Language and Culture Practice Exam Audio Script . . . 58

II. NotesonthePracticeExamIntroduction . . . . . . . . . . . . . . . . . . . . . . . . . . . . . . . . . . . . . . . . . . . . . . . . 66Multiple-Choice Section . . . . . . . . . . . . . . . . . . . . . . . . . . . . . . . . . . . . . . 67

Information for Questions 1–5 . . . . . . . . . . . . . . . . . . . . . . . . . . . . . . 68Information for Questions 6–12 . . . . . . . . . . . . . . . . . . . . . . . . . . . . . 70Information for Questions 13–23 . . . . . . . . . . . . . . . . . . . . . . . . . . . . 74Information for Questions 24–30 . . . . . . . . . . . . . . . . . . . . . . . . . . . . 79Information for Questions 31–40 . . . . . . . . . . . . . . . . . . . . . . . . . . . . 83Information for Questions 41–47 . . . . . . . . . . . . . . . . . . . . . . . . . . . . 89Information for Questions 48–52 . . . . . . . . . . . . . . . . . . . . . . . . . . . . 92Information for Questions 53–57 . . . . . . . . . . . . . . . . . . . . . . . . . . . . 94Information for Questions 58–65 . . . . . . . . . . . . . . . . . . . . . . . . . . . . 97

Free-Response Section . . . . . . . . . . . . . . . . . . . . . . . . . . . . . . . . . . . . . . . 102Information for Interpersonal Writing: E-mail Reply . . . . . . . . . . . 103Information for Presentation Writing: Persuasive Essay . . . . . . . . . 107Information for Interpersonal Speaking: Conversation . . . . . . . . . 112Information for Presentational Speaking: Cultural Comparison . . 117

01443-87558 APSP German Practice Exam • InDS4 • Fonts: Minion Pro Serifa, Helvetica, Times, Universal • D1 2/23/11 RI64612 • D1a 2/28/11 RI64612 • D1b 3/1/11 RI64612 • D1c 3/11/11 RI64612 • D2 3/15/11 RI64612 • D2a 3/16/11 RI64612 • D3 3/17/11 RI64612

German Language and Culture4

IntroductionBeginning in May 2012, the AP German Language and Culture Exam will assess student proficiency in the three modes of communication described in the Standards for Foreign Language Learning in the 21st Century. The revised exam will remain consistent with the current exam in overall length and will continue to feature multiple-choice and free-response questions. The exam will also feature a greater variety of authentic materials, both print and audio.

Part I of this publication is the Practice Exam. This resembles an actual AP Exam, including instructions and sample questions. However, these exam items have never been administered in an operational exam, and, therefore, statistical analysis is not available. The purpose of this section is to provide educators with sample exam questions that accurately reflect the composition and design of the revised exam, in a format that allows teachers to test their students in a situation that closely resembles the actual exam administration.

Part II is the Notes on the Practice Exam. This section provides information on how curriculum and assessment are related through detailed explanation of how each exam question connects to the AP German Language and Culture Curriculum Framework. For multiple choice questions, correct answers are justified, and incorrect choices are explained. For free-response tasks, characteristics of strong, good, and fair responses are provided.

How AP Courses and Exams Are DevelopedAP courses and exams are designed by committees of college faculty and AP teachers who ensure that each AP course and exam reflects and assesses college-level expectations. These committees define the scope and expectations of the course, articulating through a curriculum framework what students should know and be able to do upon completion of the AP course. Their work is informed by data collected from a range of colleges and universities to ensure that AP course work reflects current scholarship and advances in the discipline.

These same committees are also responsible for designing and approving exam specifications and exam questions that clearly connect to the curriculum framework. The AP Exam development process is a multi-year endeavor; all AP Exams undergo extensive review, revision, piloting, and analysis to ensure that questions are high quality and fair and that the questions comprise an appropriate range of difficulty.

Throughout AP course and exam development, the College Board gathers feedback from secondary and post-secondary educators. This feedback is carefully considered to ensure that AP courses and exams provide students with a college-level learning experience and the opportunity to demonstrate their qualifications for advanced placement and college credit upon college entrance.

01443-87558 APSP German Practice Exam • InDS4 • Fonts: Minion Pro Serifa, Helvetica, Times, Universal • D1 2/23/11 RI64612 • D1a 2/28/11 RI64612 • D1b 3/1/11 RI64612 • D1c 3/11/11 RI64612 • D2 3/15/11 RI64612 • D2a 3/16/11 RI64612 • D3 3/17/11 RI64612

German Language and Culture 5

Methodology Guiding the Revision

The course and the exam are conceived and developed using similar methodologies. The course is designed using the principles of Understanding by Design, and the exam is defined using an evidence-centered design approach. Both processes begin by identifying the end goals which identify what students should know and be able to do by the end of their AP experience. These statements about students’ knowledge and abilities, along with descriptions of the observable evidence that delineate levels of student performance, serve simultaneously as the learning objectives for the course and the targets of measurement for the exam. The course and exam, by design, share the same foundation.

Course Development

Each committee first articulates its discipline’s high-level goals before identifying the course’s specific learning objectives. For AP courses in world languages and cultures, the committees representing six languages worked together to define learning objectives directly linked to Interpersonal, Interpretive, and Presentational communication. For each language, the end result is a set of learning objectives that comprise the curriculum framework.

Exam Development

Exam development begins with the committee making decisions about the overall nature of the exam. How will the learning objectives for the course be assessed? How can students best demonstrate their proficiencies in each mode of communication? How will the course content and skills be distributed across the exam? How many multiple-choice questions should there be? How many free-response tasks should be included? How much time will be devoted to each section? Answers to these questions become part of the exam specifications.

With the exam specifications set, test developers design questions that conform to these specifications. The committee reviews every exam question for alignment with the curriculum framework, accuracy, and a number of other criteria that ensure the integrity of the exam.

Exam questions are then piloted in AP classrooms to determine their statistical properties. Questions that have been approved by the committee and piloted successfully are included in an exam. When an exam is assembled, the committee conducts a final review to ensure overall conformity with the specifications.

How AP Exams Are ScoredThe exam scoring process, like the course and exam development process, relies on the expertise of both AP teachers and college faculty. While multiple-choice questions are scored by machine, the free-response questions are scored by thousands of college faculty and expert AP teachers at the annual AP Reading. AP Exam Readers are thoroughly trained, and their work is monitored throughout the Reading for fairness and consistency. In each subject, a highly respected college faculty member fills the role of Chief Reader, who, with the help of AP Readers

01443-87558 APSP German Practice Exam • InDS4 • Fonts: Minion Pro Serifa, Helvetica, Times, Universal • D1 2/23/11 RI64612 • D1a 2/28/11 RI64612 • D1b 3/1/11 RI64612 • D1c 3/11/11 RI64612 • D2 3/15/11 RI64612 • D2a 3/16/11 RI64612 • D3 3/17/11 RI64612

German Language and Culture6

in leadership positions, maintains the accuracy of the scoring standards. Scores on the free-response questions are weighted and combined with the results of the computer-scored multiple-choice questions, and this raw score is summed to give a composite AP score of 5, 4, 3, 2 or 1.

The score-setting process is both precise and labor intensive, involving numerous psychometric analyses of the results of a specific AP Exam in a specific year and of the particular group of students who took that exam. Additionally, to ensure alignment with college-level standards, part of the score-setting process involves comparing the performance of AP students with the performance of students enrolled in comparable courses in colleges throughout the United States. In general, the AP composite score points are set so that the lowest raw score needed to earn an AP score of 5 is equivalent to the average score among college students earning grades of A in the college course. Similarly, AP Exam scores of 4 are equivalent to college grades of A-, B+ and B. AP Exam scores of 3 are equivalent to college grades of B-, C+ and C.

Using and Interpreting AP ScoresThe extensive work done by college faculty and AP teachers in the development of the course and the exam and throughout the scoring process ensures that AP Exam scores accurately represent students’ achievement in the equivalent college course. While colleges and universities are responsible for setting their own credit and placement policies, AP scores signify how qualified students are to receive college credit and placement:

APScore Qualification

5 Extremely well qualified

4 Well qualified

3 Qualified

2 Possibly qualified

1 No recommendation

Additional ResourcesVisit apcentral.collegeboard.com for more information about the AP Program.

01443-87558 APSP German Practice Exam • InDS4 • Fonts: Minion Pro Serifa, Helvetica, Times, Universal • D1 2/23/11 RI64612 • D1a 2/28/11 RI64612 • D1b 3/1/11 RI64612 • D1c 3/11/11 RI64612 • D2 3/15/11 RI64612 • D2a 3/16/11 RI64612 • D3 3/17/11 RI64612

German Language and Culture 7

Practice Exam

Exam Content and FormatThe 2012 AP German Language and Culture Exam is approximately 3 hours in length. There are two sections:

• A 95-minute multiple-choice section consisting of 65 questions accounting for 50 percent of the final score.

• An 85-minute free-response section consisting of 2 writing tasks and 2 speaking tasks, together accounting for 50 percent of the final score.

Administering the Practice Exam This section contains instructions for administering the AP German Language and Culture Practice Exam. You may wish to use these instructions to create an exam situation that resembles an actual administration. If so, read the indented, boldface directions to the students; all other instructions are for administering the exam and need not be read aloud.

These instructions are for administering the exam in the standard sequence. Keep in mind that during the regular exam administration, multiple-choice Section I Parts A and B are administered first regardless of the sequence in which the parts of the free-response section are administered.

Proctors administering this exam should be thoroughly familiar with all equipment needed for testing. See the AP Coordinator’s Manual for more information.

SECTIONI:Multiple-ChoiceQuestions

When you are ready to begin Section I, say:

SectionIisthemultiple-choiceportionoftheexam.Markallofyourresponsesonyouranswersheet,oneresponseperquestion.Yourscoreonthemultiple-choicesectionwillbebasedsolelyonthenumberofquestionsansweredcorrectly.

SectionItakesapproximately1hourand35minutesandisdividedintotwoparts.Intheexambooklet,identicalinstructionsareprintedinEnglishandinGerman.Donotspendtimereadingtheinstructionsinbothlanguages.Youwillhave40minutesforPartA.WhenyouhavecompletedPartA,youmaycheckyourworkonPartA,butyoucannotmoveontoPartBuntilyouaretoldtodoso.Arethereanyquestions?

Then say:

NowbeginworkonPartA.Youhave40minutesforthispart.

Note Start Time for Part A here______. Note Stop Time here__________.

After 40 minutes, say:

StopworkonPartA.DonotbeginworkonPartBuntilyouareinstructedtodoso.

01443-87558 APSP German Practice Exam • InDS4 • Fonts: Minion Pro Serifa, Helvetica, Times, Universal • D1 2/23/11 RI64612 • D1a 2/28/11 RI64612 • D1b 3/1/11 RI64612 • D1c 3/11/11 RI64612 • D2 3/15/11 RI64612 • D2a 3/16/11 RI64612 • D3 3/17/11 RI64612

German Language and Culture8

To access the audio files for Section I, Part B of the AP German Language and Culture Practice Exam, visithttp://apcentral.collegeboard.com/apc/public/216420.html

Start the audio. Play the first few seconds of audio to adjust the volume and then stop the audio. Now say:

Youmaytakenoteswhileyouarelisteningtotheaudio.IwillnotstoptheaudiountiltheendofPartB.Youwillhaveapproximately55minutesforthispart.NowturntoPartBandwaitfortherecordingtobegin.Raiseyourhandiftheaudioisnotloudenough.

Begin playing the audio. Check that students are marking their answers on their answer sheets. Approximately 55 minutes after beginning Part B, you will hear “End of Recording.” At that point, stop the audio and say:

Stopworking.Iwillnowcollectyourtestmaterials.

Collect materials from each student.

There is a 10-minute break between Sections I and II. When all Section I materials have been collected and you are ready for the break, say:

Nowyouhavea10minutebreak.Testingwillresumeat_______.

SECTIONII,PartA:E-mailReply(Task1)andPersuasiveEssay(Task2)

Throughout Section II, students should be using only the pages that correspond to the task they are working on. Task numbers are indicated across the top of each page.

When you are ready to begin Section II, say:

WearenowreadytobeginSectionII.ThroughoutSectionII,youshouldbeusingonlythepagesthatcorrespondtothedesignatedtask.Tasknumbersareindicatedacrossthetopofeachpage.Youhave15minutestocompleteTask1.Youareresponsibleforpacingyourself.Arethereanyquestions?

NowturntoTask1,andbegin.Youhave15minutesforthistask.

Note Start Time for Part A Task 1 here_________. Note Stop Time here_________.

After 15 minutes, say:

StopworkonTask1.DonotbeginworkonTask2untilyouareinstructedtodoso.

To access the audio files for Section II, Part A: Persuasive Essay (Task 2) of the AP German Language and Culture Practice Exam, visit

http://apcentral.collegeboard.com/apc/public/216420.html

01443-87558 APSP German Practice Exam • InDS4 • Fonts: Minion Pro Serifa, Helvetica, Times, Universal • D1 2/23/11 RI64612 • D1a 2/28/11 RI64612 • D1b 3/1/11 RI64612 • D1c 3/11/11 RI64612 • D2 3/15/11 RI64612 • D2a 3/16/11 RI64612 • D3 3/17/11 RI64612

German Language and Culture 9

Start the audio for the Persuasive Essay. Play the first few seconds of the audio so that you can adjust the volume, and then stop the audio. Then say:

Youwillhaveapproximately55minutestocompleteTask2.Youmaytakenoteswhileyoulistentotheaudio.NowturntoTask2andwaitfortherecordingtobegin.Raiseyourhandiftheaudioisnotloudenough.

Start the audio. In approximately 55 minutes, you will hear “End of recording.” At that point, stop the recording. Then say:

Stopworking.Iwillcollectthetestmaterialsnow.Remaininyourseat,withouttalking,whiletheexammaterialsarecollected.

Collect Section II Part A materials from every student.

SECTIONII,PARTB:Conversation(Task3)andCulturalComparison(Task4)

For this section, each student must have a recording device (e.g. digital recorder, computer with microphone, digital language lab station, cassette recorder). Testing time for Part B is approximately 15 minutes. To access the audio files for Section II, Part B: Conversation (Task 3) and Cultural Comparison (Task 4) of the AP German Language and Culture Practice Exam, visithttp://apcentral.collegeboard.com/apc/public/216420.html

Start the audio for Task 3. Play the first few seconds of the audio so that you can adjust the volume, and then stop the audio. Make sure students are ready to record. When you are ready to begin, say:

Followtheinstructionsfromthemasteraudiotorecordyourresponses.Youshouldrecordyourresponsesatanormalspeakinglevel.Ifatanytimeyoucannotheartheaudioorifyouhavetroublewiththeequipment,raiseyourhand.

Iwillnotreplaytherecordingifyoudon’tunderstandthequestions.Iwillnotstoptherecordingunlesstheequipmentfails.Youshouldstart,pause,orstopyourrecordingonlywheninstructedtodoso.Youmaytakenotesifyoulike.Arethereanyquestions?

NowturntoTask3,andwaitfortherecordingtobegin.

Play the entire audio. Stop the audio when you hear the words “End of Recording.” Then say:

NowturntoTask4,andwaitfortherecordingtobeginagain.

Play the entire audio. Stop the audio when you hear the words “End of Recording.” Then say:

Theexamisover.Iwillnowcollectyourmaterials.

01443-87558 APSP German Practice Exam • InDS4 • Fonts: Minion Pro Serifa, Helvetica, Times, Universal • D1 2/23/11 RI64612 • D1a 2/28/11 RI64612 • D1b 3/1/11 RI64612 • D1c 3/11/11 RI64612 • D2 3/15/11 RI64612 • D2a 3/16/11 RI64612 • D3 3/17/11 RI64612

German Language and Culture10

Name:

AP® German Language and CultureStudent Answer Sheet

for Multiple-Choice Section

No. Answer No. Answer No. Answer

1 31 58

2 32 59

3 33 60

4 34 61

5 35 62

6 36 63

7 37 64

8 38 65

9 39

10 40

11 41

12 42

13 43

14 44

15 45

16 46

17 47

18 48

19 49

20 50

21 51

22 52

23 53

24 54

25 55

26 56

27 57

28

29

30

01443-87558 APSP German Practice Exam • InDS4 • Fonts: Minion Pro Serifa, Helvetica, Times, Universal • D1 2/23/11 RI64612 • D1a 2/28/11 RI64612 • D1b 3/1/11 RI64612 • D1c 3/11/11 RI64612 • D2 3/15/11 RI64612 • D2a 3/16/11 RI64612 • D3 3/17/11 RI64612

German Language and Culture 11

InstructionsSection I of this exam contains 65 multiple-choice questions.

Indicate all of your answers to the multiple-choice questions on the answer sheet. Nocredit will be given for anything written in this exam booklet, but you may use the bookletfor notes or scratch work. After you have decided which of the suggested answers is best,mark your response on your answer sheet, one response per question. If you change ananswer, be sure that the previous mark is erased completely. Here is a sample questionand answer.

Use your time effectively, working as quickly as you can without losing accuracy. Do notspend too much time on any one question. Go on to other questions and come back tothe ones you have not answered if you have time. It is not expected that everyone willknow the answers to all of the multiple-choice questions.

Your total score on the multiple-choice section is based only on the number of questionsanswered correctly. Points are not deducted for incorrect answers or unansweredquestions.

DO NOT OPEN THIS BOOKLET UNTIL YOU ARE TOLD TO DO SO.

SECTION I: Multiple-Choice Questions

AP® German Language and CulturePractice Exam

At a GlanceTotal TimeApproximately 1 hour,35 minutes

Number of Questions65

Percent of Total Score50%

Writing InstrumentPencil required

Section I Part ANumber of Questions30

Time40 minutes

Section I Part BNumber of Questions35

TimeApproximately55 minutes

01443-87558 APSP German Practice Exam • InDS4 • Fonts: Minion Pro Serifa, Helvetica, Times, Universal • D1 2/23/11 RI64612 • D1a 2/28/11 RI64612 • D1b 3/1/11 RI64612 • D1c 3/11/11 RI64612 • D2 3/15/11 RI64612 • D2a 3/16/11 RI64612 • D3 3/17/11 RI64612

German Language and Culture12

GERMAN LANGUAGE AND CULTURE

GO ON TO THE NEXT PAGE.

You will read several selections. Each selection isaccompanied by a number of questions. For eachquestion, choose the response that is best accordingto the selection and mark your answer on youranswer sheet.

Sie werden im folgenden Teil verschiedene Textelesen. Nach jeder Auswahl folgen einige Fragen.Wählen Sie für jede Frage die beste Antwort fürdiese Textauswahl und markieren Sie Ihre Antwortauf dem Antwortbogen!

Part A

Time — 4 0 minutes

Total Time — Approximately 1 hour and 35 minutes

A A A A A A A A A A A A A A A SECTION I

01443-87558 APSP German Practice Exam • InDS4 • Fonts: Minion Pro Serifa, Helvetica, Times, Universal • D1 2/23/11 RI64612 • D1a 2/28/11 RI64612 • D1b 3/1/11 RI64612 • D1c 3/11/11 RI64612 • D2 3/15/11 RI64612 • D2a 3/16/11 RI64612 • D3 3/17/11 RI64612

German Language and Culture 13

GO ON TO THE NEXT PAGE.

A A A A A A A A A A A A A A A Auswahl 1Thema: Alltag

Übersicht

In diesem Text geht es um ein Angebot der Jugendherberge Bad Marienberg. Die ursprüngliche Werbung wurde am29. September 2009 in Deutschland von dem deutschen Jugendherbergsverband veröffentlicht.

3 Tage-Erlebnisprogramm Jugendherberge Bad Marienberg

Erlebnisprogramm – Klasse(n)gemeinschaft Pur1. Tag : Die Anreise erfolgt zum Mittagessen. Anschließend wird im Wald einiges Interessantes

erfahren und beim Bau einer Laubhütte kann sich nach Herzenslust ausgetobt werden.

2. Tag : Heute können die Teams beweisen, was in ihnen steckt. Wer wird mit Cleverness undeinem guten Zusammenhalt als Sieger aus diesem Tag hervorgehen? Für diese actionreicheZeit ist alles vorbereitet!

3. Tag : Nach einem reichhaltigen Frühstücksbüfett tritt die Klasse die Heimreise an.

• 3 Tage mit Vollpension: 55,80 € pro Person

• Online-Vorzugspreis mit Vollpension: 41,80 € pro Person (15.11.–15.03.)

• Programmkosten: 28,20 € pro Person

Die Jugendherberge bietet außerdem für Klassenfahrten auch ein 5 Tage-Erlebnisprogramm sowieProgrammbausteine an.

Mehr Infos:

Tel.: 02661-5008 oder Internet: Jugendherberge Bad Marienberg

�Erlebnisprogramme für die Klassenfahrt� from www.diejugendherbergen, copyright © 2009 by Jacob Geditz. Used by permission.

Zeile

5

10

01443-87558 APSP German Practice Exam • InDS4 • Fonts: Minion Pro Serifa, Helvetica, Times, Universal • D1 2/23/11 RI64612 • D1a 2/28/11 RI64612 • D1b 3/1/11 RI64612 • D1c 3/11/11 RI64612 • D2 3/15/11 RI64612 • D2a 3/16/11 RI64612 • D3 3/17/11 RI64612

German Language and Culture14

GO ON TO THE NEXT PAGE.

A A A A A A A A A A A A A A A 1. Was ist der Zweck dieser Anzeige?

(A) Schulklassen auf einen Ausflugvorzubereiten

(B) Teilnehmer über einen Wettbewerb zuinformieren

(C) Über die Erlebnisse einer Reise zuberichten

(D) Für Programmteilnehmer in Schulen zuwerben

2. Wer wird sich wahrscheinlich am meisten fürdiese Anzeige interessieren?

(A) Rentnerinnen und Rentner

(B) Mütter und Väter

(C) Lehrerinnen und Lehrer

(D) Studentinnen und Studenten

3. Was ist ein Vorteil des Erlebnisprogramms?

(A) Gesund essen

(B) Gemeinschaft stärken

(C) Geld verdienen

(D) Gedanken austauschen

4. Was wird für 55,80 € alles geboten?

(A) Übernachtung mit Frühstücksbüfett

(B) Nur die Übernachtung selbst

(C) Alle Mahlzeiten plus Übernachtung

(D) Übernachtung und Erlebnisprogramm

5. Sie wollen mehr Informationen über dasKlassenfahrtsprogramm der Jugendherberge BadMarienberg erhalten und schreiben dem Leiterder Jugendherberge eine E-Mail. Wie würdenSie Ihre Anfrage am besten formulieren?

(A) Schickt mir doch bitte mehr Informationenüber euer Klassenfahrtsprogramm!

(B) Könnten Sie mir bitte mehr Informationenüber Ihr Klassenfahrtsprogrammschicken?

(C) Ich möchte dich darum bitten, mir mehrInfos über dein Klassenfahrtsprogrammzu schicken.

(D) Schicken Sie mir sofort mehrInformationen über IhrKlassenfahrtsprogramm!

01443-87558 APSP German Practice Exam • InDS4 • Fonts: Minion Pro Serifa, Helvetica, Times, Universal • D1 2/23/11 RI64612 • D1a 2/28/11 RI64612 • D1b 3/1/11 RI64612 • D1c 3/11/11 RI64612 • D2 3/15/11 RI64612 • D2a 3/16/11 RI64612 • D3 3/17/11 RI64612

German Language and Culture 15

GO ON TO THE NEXT PAGE.

A A A A A A A A A A A A A A A Auswahl 2Thema: Naturwissenschaft und Technologie

Übersicht

In diesem Text geht es um den Naturforscher Alexander von Humboldt, einen der bedeutendsten deutschenGelehrten des 19. Jahrhunderts. Der ursprüngliche Roman „Die Vermessung der Welt“ wurde 2005 in Deutschlandvon dem Autor Daniel Kehlmann veröffentlicht. Dieser Ausschnitt des Romans präsentiert einen Rückblick aufAlexander von Humboldts Kindheit im späten 18. Jahrhundert.

Die Vermessung der WeltAlexander von Humboldt war in ganz Europa berühmt wegen einer Expedition in die Tropen, die er

fünfundzwanzig Jahre zuvor unternommen hatte. Er war in Neuspanien, Neugranada, Neubarcelona, Neuandalusienund den Vereinigten Staaten gewesen, hatte den natürlichen Kanal zwischen Orinoko und Amazonas entdeckt, denhöchsten Berg der bekannten Welt bestiegen, Tausende Pflanzen und Tiere, manche lebend, die meisten tot,gesammelt, hatte mit Papageien gesprochen, Leichen ausgegraben, jeden Fluss, Berg und See auf seinem Wegvermessen, war in jedes Erdloch gekrochen und hatte mehr Beeren gekostet und Bäume erklettert, als sichirgendjemand vorstellen mochte.

Er war der jüngere von zwei Brüdern. Ihr Vater, ein wohlhabender Mann von niederem Adel, war früh gestorben.Seine Mutter hatte sich bei niemand anderem als Goethe erkundigt, wie sie ihre Söhne ausbilden solle.

Ein Brüderpaar, antwortete dieser, in welchem sich so recht die Vielfalt menschlicher Bestrebungen ausdrücke, woalso die reichen Möglichkeiten zu Tat und Genuss auf das vorbildlichste Wirklichkeit geworden, das sei in der Tat einSchauspiel, angetan, den Sinn mit Hoffnung und den Geist mit mancherlei Überlegungen zu erfüllen.

Diesen Satz verstand keiner. Nicht die Mutter, nicht ihr Majordomus Kunth, ein magerer Herr mit großen Ohren.Er meine zu begreifen, sagte Kunth schließlich, es handle sich um ein Experiment. Der eine solle zum Mann derKultur ausgebildet werden, der andere zum Mann der Wissenschaft.

Und welcher wozu?Kunth überlegte. Dann zuckte er die Schultern und schlug vor, eine Münze zu werfen.Fünfzehn hochbezahlte Experten hielten ihnen Vorlesungen auf Universitätsniveau. Für den jüngeren Bruder

Chemie, Physik, Mathematik, für den älteren Sprachen und Literatur, für beide Griechisch, Latein und Philosophie.Zwölf Stunden am Tag, jeden Tag der Woche, ohne Pause oder Ferien.

Der jüngere Bruder, Alexander, war wortkarg und schwächlich, man musste ihn zu allem ermutigen, seine Notenwaren mittelmäßig. Wenn man ihn sich selbst überließ, strich er durch die Wälder, sammelte Käfer und ordnete sienach selbsterdachten Systemen. Mit neun Jahren baute er den von Benjamin Franklin erfundenen Blitzableiter nachund befestigte ihn auf dem Dach des Schlosses, das sie nahe der Hauptstadt bewohnten. Es war der zweite inDeutschland überhaupt; der andere stand in Göttingen auf dem Dach des Physikprofessors Lichtenberg. Nur andiesen zwei Orten war man vor dem Himmel sicher.

Der ältere Bruder sah aus wie ein Engel. Er konnte reden wie ein Dichter und schrieb früh altkluge Briefe an dieberühmtesten Männer des Landes. Wer immer ihn traf, wusste sich vor Begeisterung kaum zu fassen. Mit dreizehnbeherrschte er zwei Sprachen, mit vierzehn vier, mit fünfzehn sieben. Er war noch nie bestraft worden, keiner konntesich erinnern, dass er je etwas falsch gemacht hätte.

Zeile

5

10

15

20

25

30

01443-87558 APSP German Practice Exam • InDS4 • Fonts: Minion Pro Serifa, Helvetica, Times, Universal • D1 2/23/11 RI64612 • D1a 2/28/11 RI64612 • D1b 3/1/11 RI64612 • D1c 3/11/11 RI64612 • D2 3/15/11 RI64612 • D2a 3/16/11 RI64612 • D3 3/17/11 RI64612

German Language and Culture16

GO ON TO THE NEXT PAGE.

A A A A A A A A A A A A A A A 6. Wodurch wurde Alexander von Humboldt

berühmt?

(A) Er baute den ersten Blitzableiter in Europa.

(B) Er hielt Vorlesungen an einer Universität.

(C) Er reiste fünfundzwanzig Jahre lang umdie Welt.

(D) Er erforschte die tropischen RegionenAmerikas.

7. Was hatte Goethe mit der Familie von Humboldtzu tun?

(A) Er bildete die Brüder aus.

(B) Die Mutter fragte ihn um Rat.

(C) Er war mit der Familie verwandt.

(D) Der Vater war mit ihm befreundet.

8. Wie interpretierte Majordomus Kunthschließlich Goethes Satz?

(A) Beide Brüder sollten Experimente machen.

(B) Nur einer der Brüder sollte Wissenschaftlerwerden.

(C) Die Brüder sollten hoch bezahlt werden.

(D) Einer der Brüder sollte Literatur undBiologie studieren.

9. Welche Art von Ausbildung bekamen dieBrüder?

(A) Privatlehrer gaben ihnen Unterricht.

(B) Sie studierten an einer Universität.

(C) Majordomus Kunth lehrte sie persönlich.

(D) Sie lernten nach selbsterdachten Systemen.

10. Welche der folgenden Bedeutungen hat das Wort„wortkarg“ (Zeile 21), so wie es an dieser Stelleim Text benutzt wird?

(A) Jemand, der nicht freundlich redet

(B) Jemand, der nur ganz wenig spricht

(C) Jemand, der nicht sprechen kann

(D) Jemand, der ohne Emotion redet

11. Was zeichnete Alexander von Humboldt inseiner Jugend aus?

(A) Seine Schulnoten

(B) Seine Gesundheit

(C) Sein Interesse an der Natur

(D) Sein Interesse an Sprachen

12. Sie werden gebeten, den Inhalt diesesTextausschnittes in Stichpunktenzusammenzufassen. Welche der folgendenSequenzen würden Sie wählen?

(A)

(B)

(C)

(D)

01443-87558 APSP German Practice Exam • InDS4 • Fonts: Minion Pro Serifa, Helvetica, Times, Universal • D1 2/23/11 RI64612 • D1a 2/28/11 RI64612 • D1b 3/1/11 RI64612 • D1c 3/11/11 RI64612 • D2 3/15/11 RI64612 • D2a 3/16/11 RI64612 • D3 3/17/11 RI64612

German Language and Culture 17

Auswahl 3Thema: Globalisierung

Quellenmaterial 1Übersicht

In diesem Text geht es um den Status des Waldverlustes in der Welt. Der ursprüngliche Artikel wurde am 7.September 2009 in Deutschland von dem deutschen Journalisten Michael Miersch veröffentlicht.

Zeile

5

10

15

20

25

30

GO ON TO THE NEXT PAGE.

A A A A A A A A A A A A A A A

WELT Online/Michael Miersch

Und der Wald hat doch noch eine ZukunftNeue Studien liefern Ergebnisse, mit denen wohl niemand mehr gerechnet hätte: In der Sahara,Südamerika und den Alpen erobern wieder Bäume das Land. Gründe sind der Klimawandel,Aufforstung und ein nachhaltiger Umgang mit der Natur. Dennoch sind sich die Experten nichteinig, was genau Wald eigentlich ist.

Statistiken zeigen, dass Wälder durch Rodungen weiterhin schwinden. Doch neue Studien gebenEntwarnung: In vielen Gebieten wird der Verlust durch natürliche Prozesse verlangsamt.

Im Jahr 2007 stand ein Buch des amerikanischen Journalisten Alan Weisman auf denBestsellerlisten: „Die Welt ohne Menschen“. Weismann beschreibt darin ein Gedankenspiel: Waswürde auf der Welt geschehen, wäre die Menschheit plötzlich verschwunden? Einer der Effektewäre das Vorrücken der Wälder auf die verwaisten Felder und Weiden und in die Städte.

Weismans Vision könnte Wirklichkeit werden, auch ohne dass der Mensch von der Bildflächeverschwindet. Aus den verschiedensten Gebieten der Welt berichten Forstleute, Botaniker undÖkologen, dass die Wälder sich ausdehnen. Wird der Globus grüner?

Entwaldung bleibt im wahrsten Sinne des Wortes ein brennendes globales Problem. Umsoerfreulicher, dass in letzter Zeit aus den verschiedensten Gegenden der Welt gute Nachrichten überein verstärktes Baumwachstum bekannt werden. Teilweise aus Regionen, wo man es am wenigstenerwartet hätte, zum Beispiel der Sahara. Die Sahel-Region am Südrand der großen Wüste wirdimmer grüner. Satellitenbilder, Flugbilder und Fotos, die über Jahrzehnte dieVegetationsentwicklung dokumentieren, zeigen ein Vorrücken von Büschen und Bäumen nachNorden. Gazellen und andere Wildtiere kommen zurück, sogar Frösche.

Überraschend sind auch aktuelle Studien von Wissenschaftlern, die sich mit dem Tropenwald inLateinamerika befassen. Sie legen nahe, dass die dortigen Verlustraten womöglich überschätztwurden.

Alan Grainger, Geograf von der Universität Leeds und einer der führenden Experten zum ThemaEntwaldung in den Tropen, hält die dramatischen Zahlen über globale Regenwaldverluste fürübertrieben. Er verglich über drei Jahre lang alle teilweise sehr widersprüchlichen Statistiken undfand keine Bestätigung für einen erheblichen Nettoverlust. „Das Gesamtbild ist komplizierter alswir früher dachten“, sagt er. Grainger nimmt an, dass bisher zu wenig Augenmerk auf die natürlicheAusbreitung der Wälder gelegt wurde. Es gibt immer noch abgelegene Gegenden, wo die Wäldergrößer werden, ohne dass dies jemand registriert.

01443-87558 APSP German Practice Exam • InDS4 • Fonts: Minion Pro Serifa, Helvetica, Times, Universal • D1 2/23/11 RI64612 • D1a 2/28/11 RI64612 • D1b 3/1/11 RI64612 • D1c 3/11/11 RI64612 • D2 3/15/11 RI64612 • D2a 3/16/11 RI64612 • D3 3/17/11 RI64612

German Language and Culture18

GO ON TO THE NEXT PAGE.

A A A A A A A A A A A A A A A Quellenmaterial 2Übersicht

Diese Auswahl handelt von der weltweiten Ab- und Zunahme der Waldflächen. Die ursprüngliche Karte wurde am24. Februar 2006 in Deutschland von Waldundklima.net veröffentlicht.

Weltweite Ab- und Zunahme der Waldflächen

Je größer das Quadrat, umso größer die Zu- oder Abnahme in den jeweiligen Ländern.

01443-87558 APSP German Practice Exam • InDS4 • Fonts: Minion Pro Serifa, Helvetica, Times, Universal • D1 2/23/11 RI64612 • D1a 2/28/11 RI64612 • D1b 3/1/11 RI64612 • D1c 3/11/11 RI64612 • D2 3/15/11 RI64612 • D2a 3/16/11 RI64612 • D3 3/17/11 RI64612

German Language and Culture 19

GO ON TO THE NEXT PAGE.

A A A A A A A A A A A A A A A 13. Wer würde sich am meisten für diesen Artikel

interessieren?

(A) Leute, die sich für Reisen interessieren

(B) Leute, die sich für Holzmöbel interessieren

(C) Leute, die sich für die Umweltinteressieren

(D) Leute, die sich für Geschichte interessieren

14. Was zeigen neue Studien über Wälder?

(A) Das Waldsterben ist in den letzten Jahrenkatastrophaler geworden.

(B) Der Wald ist nicht so wichtig für dieZukunft wie man früher dachte.

(C) Der Wald verschwindet nicht so schnellwie man früher dachte.

(D) Die Wälder haben sich in den letztenJahren kaum verändert.

15. Was ist die Funktion des ersten Absatzes desArtikels?

(A) Er stellt die Meinung des Autors vor.

(B) Er beschreibt gegensätzliche Argumente.

(C) Er geht auf die Quellen des Artikels ein.

(D) Er gibt einen Überblick über den Artikel.

16. Was bedeutet wohl das Wort „Rodungen“(Zeile 5)?

(A) Das Transportieren von Bäumen

(B) Das Pflanzen von Bäumen

(C) Das Fällen von Bäumen

(D) Das Untersuchen von Bäumen

17. Warum erwähnt der Autor des Artikels das Buchvon Alan Weisman, „Die Welt ohne Menschen“(Zeile 8) ?

(A) Prozesse, die in diesem Buch beschriebenwerden, passieren zum Teil schon.

(B) Das Buch widerspricht den neuestenStudien der Wissenschaftler.

(C) Das Buch beschreibt die Ursachen für dieaktuellen Probleme der Wälder.

(D) Die Wälder sind nur zu retten, wenn eskeine Menschen gäbe.

18. Warum kommen Tiere wieder in die Saharazurück?

(A) Das Klima ändert sich.

(B) Es gibt mehr Vegetarier.

(C) Die Wüste wird immer grüner.

(D) Es gibt weniger Menschen.

19. Was fand Alan Grainger heraus?

(A) Weltweit nehmen alle Wälderdramatisch zu.

(B) Die Statistiken zeigen nicht das ganzeBild.

(C) Der Verlust der Regenwälder wird immergrößer.

(D) In den Tropen findet nicht genugForschung statt.

01443-87558 APSP German Practice Exam • InDS4 • Fonts: Minion Pro Serifa, Helvetica, Times, Universal • D1 2/23/11 RI64612 • D1a 2/28/11 RI64612 • D1b 3/1/11 RI64612 • D1c 3/11/11 RI64612 • D2 3/15/11 RI64612 • D2a 3/16/11 RI64612 • D3 3/17/11 RI64612

German Language and Culture20

GO ON TO THE NEXT PAGE.

A A A A A A A A A A A A A A A 20. Was soll die Grafik zeigen?

(A) Wie sich die Wälder der Erde in Zukunftweiterentwickeln werden

(B) Wo es auf der Erde noch größereWaldflächen gibt

(C) Wo die Wälder der Erde eher zu- oderabgenommen haben

(D) Warum heutzutage immer mehr Wald aufder Erde verschwindet

21. In welchem Land nahm der Wald laut dieserGrafik am meisten ab?

(A) In China

(B) In Brasilien

(C) In den USA

(D) In Australien

22. Sie haben die Aufgabe bekommen, einenAufsatz zum Thema des obigen Artikels zuschreiben. Welches Buch würden Sie für diesenAufsatz in Ihre Bibliographie aufnehmen?

(A) „Wieso, Weshalb, Warum: Wir entdeckenden Wald“

(B) „Der Wald: Eine literarische Wanderung“

(C) „Zustand und Zukunft des Waldes“

(D) „Naturführer Bayerischer Wald“

23. Im Geografieunterricht lernen Sie etwas über dieWüste. Sie haben auch den Text „Und der Waldhat doch noch eine Zukunft“ gelesen undmöchten nun ein Poster über die Wüstezusammenstellen. Welcher Punkt aus dem Textsteht im Zentrum Ihres Posters?

(A) Satellitenbilder zeigen, dass es in derSahara oft Waldbrände gibt.

(B) In der Sahara wachsen die Städte immerschneller.

(C) Satellitenbilder zeigen, dass die Wüstegrößer wird.

(D) Pflanzen und Tiere kehren langsam in dieWüste zurück.

01443-87558 APSP German Practice Exam • InDS4 • Fonts: Minion Pro Serifa, Helvetica, Times, Universal • D1 2/23/11 RI64612 • D1a 2/28/11 RI64612 • D1b 3/1/11 RI64612 • D1c 3/11/11 RI64612 • D2 3/15/11 RI64612 • D2a 3/16/11 RI64612 • D3 3/17/11 RI64612

German Language and Culture 21

GO ON TO THE NEXT PAGE.

A A A A A A A A A A A A A A A Auswahl 4Thema: Schönheit und Ästhetik

Übersicht

In diesem Text geht es um eine Katalogbestellung. Es ist ein Brief von Petra Zimmer an die Kundin, SonjaRosenberger.

Sehr geehrte Frau Rosenberger,

vielen Dank für Ihre Bestellung vom 21.03. Leider können wir Ihnen Ihre Bestellung nicht komplettzustellen, da auf Grund einer starken Nachfrage die Hose, Modell-Nr. 3457 in Schwarz, Größe 38im Moment ausverkauft ist. Allerdings werden wir in 4 Wochen eine weitere Lieferung erhalten.Erhältlich ist dieses Modell in Größe 38 in Blau und Grau. Ihre bestellten Oberteile wären am 04.04.lieferungsbereit.

Könnten Sie uns bitte benachrichtigen, ob Sie Ihre Bestellung auf einmal oder in zwei Teilengeliefert haben möchten. Anbei ist ein Bestellschein, den Sie ausgefüllt an uns zurückschickenkönnen. Sollten Sie die Hose stornieren oder durch eine andere Farbe ersetzen, markieren Sie dasbitte auch auf dem beigefügten Bestellschein.

Vielen Dank für Ihr Verständnis. Sollten Sie weitere Fragen haben, können Sie uns bequem jederzeitzwischen 9 und 19 Uhr unter der Nummer 0853 / 0 63 04 66 erreichen (Sonntage ausgenommen).Wir freuen uns darauf, Ihnen auch in Zukunft bei Ihrer Modewahl behilflich zu sein.

Mit freundlichen Grüßen

Petra ZimmerSchmidt Versand

Zeile

5

10

15

01443-87558 APSP German Practice Exam • InDS4 • Fonts: Minion Pro Serifa, Helvetica, Times, Universal • D1 2/23/11 RI64612 • D1a 2/28/11 RI64612 • D1b 3/1/11 RI64612 • D1c 3/11/11 RI64612 • D2 3/15/11 RI64612 • D2a 3/16/11 RI64612 • D3 3/17/11 RI64612

German Language and Culture22

GO ON TO THE NEXT PAGE.

A A A A A A A A A A A A A A A 24. Warum wurde dieser Brief an die Kundin

abgeschickt?

(A) Um die Ergebnisse einer Kundenumfragemitzuteilen

(B) Um über ein Problem mit der Bestellungzu informieren

(C) Um ein günstiges Angebot zu machen

(D) Um über neue Mode zu informieren

25. In welchem Stil ist dieser Brief geschrieben?

(A) Sachlich

(B) Aggressiv

(C) Emotional

(D) Persönlich

26. Was soll Frau Rosenberger machen, falls sie dieHose nicht möchte?

(A) Sie soll die Firma besuchen.

(B) Sie soll der Firma persönlich einen Briefschicken.

(C) Sie soll ein ausgefülltes Formularzurückschicken.

(D) Sie soll vier Wochen abwarten.

27. Wann kann man mit jemandem im Versandtelefonieren?

(A) Rund um die Uhr

(B) Für 12 Stunden täglich

(C) Bis 9 Uhr abends

(D) Sechs Tage in der Woche

28. Was bedeutet der Ausdruck „einer starkenNachfrage“ (Zeile 3) ?

(A) Die Leute finden die Qualität nicht gut.

(B) Viele Leute kaufen dieses Produkt.

(C) Das Produkt wird nicht mehr hergestellt.

(D) Dieses Produkt wird überall angeboten.

29. Welche Aussage wäre passend für eineschriftliche Antwort auf den Brief?

(A) Ist ja wieder typisch! Ich muss mir wohleinen anderen Versand suchen.

(B) Lassen Sie mal. Auf so ein Hin- und Herhabe ich überhaupt keine Lust!

(C) Da mir die Hose wirklich sehr gut gefällt,möchte ich auf jeden Fall darauf warten.

(D) Tja, ich weiß nicht so recht, aber die Hosehätte ich schon gerne. Aber warten? Nein,das möchte ich nicht.

30. Sonja wollte die Hose schon nächstesWochenende auf einer Party anziehen und mussnun neue Pläne machen. Sie will nun ihrerbesten Freundin darüber eine E-Mail schicken.Welches der folgenden Beispiele wäre dafür amangemessensten?

(A) Sehr geehrte Klara, gestern habe ich einenBrief bekommen. Die Hose, die ichwollte, ist ausverkauft. Hast du Lust,morgen mit mir shoppen zu gehen?

(B) Guten Tag, meine allerliebste Klara,gestern habe ich einen Brief bekommen.Die Hose, die ich wollte, ist ausverkauft.Haben Sie Lust, morgen mit mir shoppenzu gehen?

(C) Hallo Klara, gestern habe ich einen Briefbekommen. Die Hose, die ich wollte, istausverkauft. Hast du Lust, morgen mitmir shoppen zu gehen?

(D) Liebe Frau Klara Schmitt, gestern habe icheinen Brief bekommen. Die Hose, die ichwollte, ist ausverkauft. Haben Sie Lust,morgen mit mir shoppen zu gehen?

01443-87558 APSP German Practice Exam • InDS4 • Fonts: Minion Pro Serifa, Helvetica, Times, Universal • D1 2/23/11 RI64612 • D1a 2/28/11 RI64612 • D1b 3/1/11 RI64612 • D1c 3/11/11 RI64612 • D2 3/15/11 RI64612 • D2a 3/16/11 RI64612 • D3 3/17/11 RI64612

German Language and Culture 23

END OF PART A

If you finish before time is called, you may check your work on Part A only. Do not go on to Part B until you are told to do so.

STOP

Sollten Sie vor der Ansage, dass die Prüfung beendet ist, fertig werden, dürfen Sie Ihre Antworten im Teil A überprüfen. Blättern Sie nicht weiter, bis Sie dazu aufgefordert werden!

A A A A A A A A A A A A A A A

01443-87558 APSP German Practice Exam • InDS4 • Fonts: Minion Pro Serifa, Helvetica, Times, Universal • D1 2/23/11 RI64612 • D1a 2/28/11 RI64612 • D1b 3/1/11 RI64612 • D1c 3/11/11 RI64612 • D2 3/15/11 RI64612 • D2a 3/16/11 RI64612 • D3 3/17/11 RI64612

German Language and Culture24

You will listen to several audio selections. The firsttwo audio selections are accompanied by readingselections. When there is a reading selection, youwill have a designated amount of time to read it.

For each audio selection, first you will have adesignated amount of time to read a preview of theselection as well as to skim the questions that youwill be asked. Each selection will be played twice.As you listen to each selection, you may take notes.Your notes will not be scored.

Sie werden einige Audioauszüge hören. Die erstenbeiden Audioauszüge sind mit Lesetextengekoppelt. In diesem Falle steht Ihnen einevorgegebene Zeit zum Lesen dieser Texte zurVerfügung.

Vor dem Hören jeder Auswahl bekommen Sieetwas Zeit, um sich die Übersicht der Auswahlanzuschauen und die Fragen zu überfliegen. Siehören jeden Auszug zweimal. Während Siezuhören, können Sie sich Notizen machen. IhreNotizen werden nicht benotet.

GO ON TO THE NEXT PAGE.

You have 1 minute to read the directions for this part.

Sie haben 1 Minute Zeit, um die Anweisungen für den folgenden Teil zu lesen.

You will now begin this part. Sie werden jetzt mit diesem Teil beginnen.

Part B Time — Approximately 55 minutes

B B B B B B B B B B B B B B B B

After listening to each selection the first time, you will have 1 minute to begin answering the questions; after listening to each selection the second time, you will have 15 seconds per question to finish answering the questions. For each question, choose the response that is best according to the audio and/or reading selection and mark your answer on your answer sheet.

Nach dem ersten Anhören jeder Auswahl haben Sie 1 Minute Zeit, um mit dem Beantworten der Fragen zu beginnen; nach dem zweiten Anhören jeder Auswahl haben Sie pro Frage 15 Sekunden Zeit, um die Fragen fertig zu beantworten. Wählen Sie für jede Frage die Antwort, die am besten mit der vorgegebenen Auswahl übereinstimmt! Markieren Sie Ihre Antwort auf dem Antwortbogen!

01443-87558 APSP German Practice Exam • InDS4 • Fonts: Minion Pro Serifa, Helvetica, Times, Universal • D1 2/23/11 RI64612 • D1a 2/28/11 RI64612 • D1b 3/1/11 RI64612 • D1c 3/11/11 RI64612 • D2 3/15/11 RI64612 • D2a 3/16/11 RI64612 • D3 3/17/11 RI64612

German Language and Culture 25

GO ON TO THE NEXT PAGE.

B B B B B B B B B B B B B B B B Auswahl 1Thema: Naturwissenschaft und Technologie

Quellenmaterial 1

Zuerst haben Sie 4 Minuten Zeit, um das Quellenmaterial 1 zu lesen.

Übersicht

In diesem Text geht es um eine technische Neuerung. Der ursprüngliche Artikel wurde am 20. Februar 2009 inDeutschland von dem deutschen Journalisten Felix Rehwald veröffentlicht.

Dritte Tanne rechts

FußgängernavigationWer im Auto ein Navi benutzt, möchte oft auch zu

Fuß nicht auf Orientierungs- und Routentippsverzichten. Doch hält die Fußgängernavigation das,was sie verspricht?

Ein Navigationssystem gehört inzwischen fast zurStandardausstattung im Auto. Viele Fahrer möchtenden elektronischen Lotsen nicht mehr missen undwürden seine Orientierungs- und Routentipps auchgerne nutzen, wenn sie zu Fuß unterwegs sind.Manche mobilen Navis haben daher bereits einenFußgängermodus, und einige Anbieter liefern sogarKartenmaterial, auf dem auch Fuß- und Wanderwegeverzeichnet sind. Doch ganz ausgereift ist dieFußgängernavigation noch nicht.

Bislang hatten mobile Endgeräte wie Smartphonesoder Personal Navigation Assistants (PNA) dasProblem, dass sie außerhalb des Autos kaum zuverwenden waren. Das lag zum einen amKartenmaterial, das auf die Bedürfnisse derAutofahrer zugeschnitten war. Verzeichnet waren nursolche Straßen, die auch mit dem Auto zu befahrensind. Gassen, Wege und Pfade waren nicht aufgeführt– der Maßstab für Fußgänger war also zu groß, alsdass er für einen geführten Stadtrundgang getaugthätte.

Zum anderen gibt es Probleme mit der Ortung, sagtHelmut Schmaler, Navigations-Experte beimADAC-Technikzentrum in Landsberg (Bayern): „DasHauptproblem dieser Geräte ist, dass das Gerät nichtweiß, wo Norden ist. Als Fußgänger sind Sie zulangsam, als dass das Gerät sagen könnte, Sie gehenda oder da lang.“ Denn die Positionsdaten liegen zudicht beieinander. Im Auto ist wegen des höherenTempos ein größerer Abstand dazwischen, weshalbdie Fahrtrichtung erkannt wird.

Manche Smartphone-Anbieter spendieren ihrenModellen daher inzwischen einen Kompass.

Der Navigations-Anbieter Garmin hat seinemNavigationsgerät nüvi 550 schon eineKompassanzeige mitgegeben. Das Unternehmen ausGräfelfing (Bayern) bewirbt es als „Allround“-Modellfür die Verwendung im Auto, beim Wandern oderWassersport. Um das Gerät im Outdoor-Modus nutzenzu können, muss sich der Anwender allerdings einezusätzliche topografische Freizeit- oder Wanderkartekaufen.

Professionelle Outdoor-Geräte haben diese bereitsan Bord. Über das übliche Straßennetz sei in derDarstellung gewissermaßen eine topografische Kartegelegt, erklärt ADAC-Experte Schmaler. Darinwürden Höhenlinien, Flüsse und Brücken angezeigt.Die Ortungsgenauigkeit liege bei zwei bis fünfMetern. „Diese Dinger funktionieren recht gut. Aberman muss sich beim Wandern den Weg noch selbersuchen. Das Gerät sagt nicht: An der dritten Tannerechts.“

Laut ADAC-Experte Schmaler sollten Verbraucherderzeit von der Fußgängernavigation aber auch nochnicht zu viel erwarten. Die Geräte seien im Prinzip ok,aber noch weit entfernt von der Perfektion. „ZurOrientierung in einer fremden Stadt ist esausreichend“, sagt Schmaler – und die Geräte seienallemal besser als ein Stadtplan aus Papier, bei demder Besucher auch nicht weiß, wo er ist.

Felix Rehwald, Sueddeutsche Zeitung/ www.sueddeutsche.de, 22-02-2009

Zeile

5

10

15

20

25

30

35

40

45

50

55

60

01443-87558 APSP German Practice Exam • InDS4 • Fonts: Minion Pro Serifa, Helvetica, Times, Universal • D1 2/23/11 RI64612 • D1a 2/28/11 RI64612 • D1b 3/1/11 RI64612 • D1c 3/11/11 RI64612 • D2 3/15/11 RI64612 • D2a 3/16/11 RI64612 • D3 3/17/11 RI64612

German Language and Culture26

GO ON TO THE NEXT PAGE.

B B B B B B B B B B B B B B B B Quellenmaterial 2

Sie haben 2 Minuten Zeit, um die Übersicht zu lesen und die Fragen zu überfliegen.

Übersicht

In diesem Hörtext geht es um eine neue Erfindung. Der ursprüngliche Podcast „Gewusst, wo“ wurde am 16. Februar2009 in Deutschland von der deutschen Fraunhofer-Gesellschaft veröffentlicht. Ein Mitarbeiter testet ein neues Gerätmit der IIS-Mitarbeiterin Karin Loidl. Der Hörtext ist ca. zwei Minuten lang.�Gewusst, wo� from www.fraunhofer.de, copyright © 2009 by Fraunhofer. Used by permission.

31. Wen will der Artikel (Quellenmaterial 1)speziell ansprechen?

(A) Techniker und Wissenschaftler

(B) Firmen und Institute

(C) Kinder und Jugendliche

(D) Autofahrer und Fußgänger

32. Was ist ein „Navi“ (Zeile 1)(Quellenmaterial 1) ?

(A) Ein Kompass

(B) Ein GPS-Gerät

(C) Ein Verkehrsmittel

(D) Ein Autoatlas

33. Was können professionelle Outdoor-Geräte jetztschon anzeigen?

(A) Gewässer und Hügel

(B) Wanderrouten

(C) Freizeitmöglichkeiten

(D) Bäume und Büsche

34. Zu welchem Schluss kommt der Artikel(Quellenmaterial 1) über Geräte zurFußgängernavigation?

(A) In der Stadt kommt man ohne sie nicht aus.

(B) Sie werden von fast allen Wanderernbenutzt.

(C) Sie brauchen noch viele Verbesserungen.

(D) Ihre Technologie ist schon heuterevolutionär.

35. Was ist der Zweck des Podcasts(Quellenmaterial 2) ?

(A) Besuchern bei der Orientierung inNürnberg zu helfen

(B) Die Vorteile einer neuen Technologie zubeschreiben

(C) Die Arbeitsbereiche desFraunhofer-Instituts vorzustellen

(D) Für eine neue Art von Mobiltelefon zuwerben

36. Wie ist der Stil des Podcasts(Quellenmaterial 2) ?

(A) Kritisch

(B) Euphorisch

(C) Informativ

(D) Humorvoll

37. Was ist laut des Podcasts (Quellenmaterial 2)eine Schwäche von älterenFußgängernavigationsgeräten?

(A) Sie zeigen nicht an, wo man sich befindet.

(B) Man kann sie nur im Sommer benutzen.

(C) Man muss sich die Software extra kaufen.

(D) Sie funktionieren nicht gut zwischenGebäuden.

01443-87558 APSP German Practice Exam • InDS4 • Fonts: Minion Pro Serifa, Helvetica, Times, Universal • D1 2/23/11 RI64612 • D1a 2/28/11 RI64612 • D1b 3/1/11 RI64612 • D1c 3/11/11 RI64612 • D2 3/15/11 RI64612 • D2a 3/16/11 RI64612 • D3 3/17/11 RI64612

German Language and Culture 27

GO ON TO THE NEXT PAGE.

B B B B B B B B B B B B B B B B 38. Was ist ein Mobile Locator?

(A) Eine Navigationssoftware für Fußgänger

(B) Das Forschungsprojekt einesAutoherstellers

(C) Ein Signalgerät für den Stadtverkehr

(D) Ein Satellit zur Übertragung vonHandysignalen

39. Was haben der Artikel und der Podcastgemeinsam?

(A) Beide Texte behandeln die Bedeutung vonKompassen.

(B) Beide Autoren benutzen lieber traditionelleStadtpläne.

(C) Beide Texte handeln vonOrientierungshilfen für Fußgänger.

(D) Beide Autoren beschreiben interessanteStadtrundgänge.

40. Wie unterscheiden sich die beiden Texte?

(A) Der Artikel beschreibt die neuen Gerätesehr positiv, aber der Podcastkritisiert sie.

(B) Der Artikel gibt einen Überblick überverschiedene Geräte, aber der Podcastkonzentriert sich auf eine bestimmteErfindung.

(C) Der Artikel ist aus der Sicht einesindustriellen Herstellers geschrieben, aberder Podcast gibt eine unabhängigeMeinung.

(D) Der Artikel gibt konkrete Empfehlungen,aber der Podcast lässt den Hörer selbstentscheiden.

01443-87558 APSP German Practice Exam • InDS4 • Fonts: Minion Pro Serifa, Helvetica, Times, Universal • D1 2/23/11 RI64612 • D1a 2/28/11 RI64612 • D1b 3/1/11 RI64612 • D1c 3/11/11 RI64612 • D2 3/15/11 RI64612 • D2a 3/16/11 RI64612 • D3 3/17/11 RI64612

German Language and Culture28

GO ON TO THE NEXT PAGE.

B B B B B B B B B B B B B B B B Auswahl 2Thema: Alltag

Quellenmaterial 1

Zuerst haben Sie 1 Minute Zeit, um das Quellenmaterial 1 zu lesen.

Übersicht

Diese Auswahl handelt von Schülerinnen und Schülern, die 2008 und 2009 in Deutschland eine Lehrstelle suchten.Die ursprüngliche Grafik wurde 2009 in Deutschland von Trendence Schülerbarometer veröffentlicht.

Pro Sieben Sat 1 Media ist bei Schülern angesagt

Bei welchen Arbeitgebern Schulabgänger arbeiten wollen *

01443-87558 APSP German Practice Exam • InDS4 • Fonts: Minion Pro Serifa, Helvetica, Times, Universal • D1 2/23/11 RI64612 • D1a 2/28/11 RI64612 • D1b 3/1/11 RI64612 • D1c 3/11/11 RI64612 • D2 3/15/11 RI64612 • D2a 3/16/11 RI64612 • D3 3/17/11 RI64612

German Language and Culture 29

GO ON TO THE NEXT PAGE.

B B B B B B B B B B B B B B B B Quellenmaterial 2

Zuerst haben Sie 1 Minute Zeit, um die Übersicht zu lesen und die Fragen zu überfliegen.

Übersicht

In diesem Hörtext geht es um die Zukunftspläne von zwei Freunden. Es ist ein Gespräch zwischen Anna undBenjamin, die sich von der Schule her kennen. Der Hörtext ist ca. zwei Minuten lang.

41. Wer wurde in dieser Umfrage(Quellenmaterial 1) befragt?

(A) Wunscharbeitgeber

(B) Schulabgänger

(C) Lehrer

(D) Unternehmer

42. Worüber gibt die Grafik (Quellenmaterial 1)Auskunft?

(A) Was sich Arbeitgeber wünschen

(B) Wo man viel Geld verdienen kann

(C) Welche Firmen Profit machen

(D) Wo sich Schüler bewerben möchten

43. Bei welchem Unternehmen kann man 2009 eingrößeres Interesse als 2008 feststellen(Quellenmaterial 1) ?

(A) Porsche

(B) Adidas

(C) BMW

(D) Hilton Hotels

44. Warum hatte Anna Probleme, eine Stelle zufinden?

(A) Sie konnte sich nicht entscheiden.

(B) Ihre Praktika sind nicht gut verlaufen.

(C) Es gab zu wenig gute Stellen.

(D) Ihr Zeugnis war schlecht.

45. Wie erklärt Anna, dass sie schließlich eine Stellebekommen hat?

(A) Sie hat gute Englischkenntnisse.

(B) Sie hat gute Bewerbungen geschrieben.

(C) Sie hatte ein super Abschlusszeugnis.

(D) Sie hat zwei erfolgreiche Praktikagemacht.

46. Benjamin und Anna sprechen über Praktika.Was bedeutet das Wort „Praktikum“?

(A) Ein Kurs, der die Schüler auf das Abiturvorbereitet

(B) Eine bestimmte Zeit, in der ein SchülerHausaufgaben macht

(C) Eine bestimmte Zeit, in der Schüler ineinem Betrieb arbeiten

(D) Ein Kurs, in dem Schüler Sprachkenntnisseverbessern können

47. Wie würde sich Anna am Ende des Gesprächsvon Benjamin verabschieden?

(A) Schön, Sie wiederzusehen!

(B) Ich danke Ihnen für das Gespräch.

(C) Viel Spaß bei deinem neuen Job!

(D) Ich hoffe, du findest bald eine Stelle.

01443-87558 APSP German Practice Exam • InDS4 • Fonts: Minion Pro Serifa, Helvetica, Times, Universal • D1 2/23/11 RI64612 • D1a 2/28/11 RI64612 • D1b 3/1/11 RI64612 • D1c 3/11/11 RI64612 • D2 3/15/11 RI64612 • D2a 3/16/11 RI64612 • D3 3/17/11 RI64612

German Language and Culture30

GO ON TO THE NEXT PAGE.

B B B B B B B B B B B B B B B B Auswahl 3Thema: Persönliche und Öffentliche Identität

Übersicht

Zuerst haben Sie 1 Minute Zeit, um die Übersicht zu lesen und die Fragen zu überfliegen.

In diesem Hörtext geht es um die deutsche Schauspielerin Franka Potente. Das ursprüngliche Interview wurde am31. Mai 2009 in Deutschland von dem deutschen Moderator Jens Hübschen bei SWR Leute veröffentlicht. DerHörtext ist ca. drei Minuten lang.�Interview with Franka Potente,� from www.swr.de, copyright © 2008 by Südwestrundfunk. Used by permission.

48. Worüber spricht Franka Potente in demAusschnitt hauptsächlich?

(A) Was sie in ihrem Privatleben macht

(B) Wie ihre Eltern ihr Leben beeinflussen

(C) Was sie aus ihrer Kindheit gelernt hat

(D) Wie sie zur Schauspielerei gekommen ist

49. Was finden der Moderator und Franka Potentelustig?

(A) Was Franka Potente sich als Kind fürTheaterstücke ausgedacht hat

(B) Wie Franka Potente schon als Kindversucht hat, Geld zu machen

(C) Wie Franka Potente ihre Eltern von ihrerBerufswahl überzeugte

(D) Was Franka Potente früher für eigenartigeIdeen von ihrer Zukunft hatte

50. Wie reagierten Franka Potentes Eltern auf ihreLiebe zur Schauspielerei?

(A) Sie motivierten sie dazu, eineSchauspielschule zu besuchen.

(B) Sie hielten sich zuerst zurück, aberunterstützten sie dann später.

(C) Sie unterstützten die frühen Versuche,wurden aber immer kritischer.

(D) Sie haben alles getan, sie von diesemInteresse abzubringen.

51. Was, meint der Interviewer, hätte für FrankaPotente eine alternative Karrieremöglichkeit seinkönnen?

(A) Lehrerin

(B) Biologin

(C) Profisportlerin

(D) Journalistin

52. Sie rufen den Radiosender an, um die nächsteFrage persönlich an Franka Potente zu stellen.Was wäre die nächste logische Frage für denInterviewauszug, den Sie gerade gehört haben?

(A) Hätten Sie vielleicht eine andere Karrieregewählt, wenn Sie in einer Großstadtaufgewachsen wären?

(B) Wie lange hast du denn schon in derPostproduktion von Filmen gearbeitet?

(C) Spielte der Sport damals nichteine ziemlich wichtige Rolle in IhremLeben?

(D) Hättest du vielleicht Lust, nächstenSommer in unserer Schule bei einemTheaterstück mitzumachen?

01443-87558 APSP German Practice Exam • InDS4 • Fonts: Minion Pro Serifa, Helvetica, Times, Universal • D1 2/23/11 RI64612 • D1a 2/28/11 RI64612 • D1b 3/1/11 RI64612 • D1c 3/11/11 RI64612 • D2 3/15/11 RI64612 • D2a 3/16/11 RI64612 • D3 3/17/11 RI64612

German Language and Culture 31

GO ON TO THE NEXT PAGE.

B B B B B B B B B B B B B B B B Auswahl 4Thema: Naturwissenschaft und Technologie

Übersicht

Zuerst haben Sie 1 Minute Zeit, um die Übersicht zu lesen und die Fragen zu überfliegen.

In diesem Hörtext geht es um den Gebrauch von Zahnbürsten. Das ursprüngliche Interview wurde am 1. April 2008in Deutschland von dem Zahnfischer Podcast unter dem Titel „Die Zahnbürste“ veröffentlicht. Ein Moderator sprichtmit Alexander Fischer. Der Hörtext ist ca. zwei Minuten lang.© Men in Text, Sven Oswald

53. Wer wird sich diesen Podcast am ehestenanhören?

(A) Jemand, der seine Zahnhygieneverbessern will

(B) Jemand, der Zahnschmerzen hat

(C) Jemand, der Zahnarzt werden will

(D) Jemand, der beruflich mit Zahnbürsten zutun hat

54. Warum spricht Herr Fischer wohl überZahnhygiene?

(A) Er ist Verkäufer.

(B) Er ist Journalist.

(C) Er ist Apotheker.

(D) Er ist Zahnarzt.

55. Was ist Herrn Fischers Meinung zu denverschiedenen Zahnbürsten?

(A) Er begrüßt die Entwicklung immer neuererZahnbürsten.

(B) Er kritisiert, dass billige Zahnbürsten sehrungesund sind.

(C) Er findet, dass es zu viele Zahnbürsten aufdem Markt gibt.

(D) Er denkt, dass jeder Mensch eine andereZahnbürste braucht.

56. Welchen Ratschlag gibt Herr Fischer Zuhörern,die eine Handzahnbürste bevorzugen?

(A) Lieber eine elektrische Zahnbürste zubenutzen

(B) Eine Zahnbürste mit weichen Borsten zubenutzen

(C) Die Zahnbürste nicht im Supermarkt zukaufen

(D) Eine besonders gute Zahnpasta zu kaufen

57. Wie bleibt eine Zahnbürste hygienisch?

(A) Man wechselt sie jeden Tag.

(B) Man benutzt sie ohne Zahnpasta.

(C) Man lässt sie trocknen.

(D) Man reinigt sie mit Seife.

01443-87558 APSP German Practice Exam • InDS4 • Fonts: Minion Pro Serifa, Helvetica, Times, Universal • D1 2/23/11 RI64612 • D1a 2/28/11 RI64612 • D1b 3/1/11 RI64612 • D1c 3/11/11 RI64612 • D2 3/15/11 RI64612 • D2a 3/16/11 RI64612 • D3 3/17/11 RI64612

German Language and Culture32

GO ON TO THE NEXT PAGE.

B B B B B B B B B B B B B B B B Auswahl 5Thema: Schönheit und Ästhetik

Übersicht

Zuerst haben Sie 1 Minute Zeit, um die Übersicht zu lesen und die Fragen zu überfliegen.

In diesem Hörtext geht es um die Kunstschule Bauhaus. Es handelt sich hier um einen Vortrag von einem Expertenauf diesem Gebiet. Der Hörtext ist ca. drei Minuten lang.

58. Wie wird das Bauhaus in diesem Vortragcharakterisiert?

(A) Als eine altmodische Kunstrichtung

(B) Als eine politisch bedeutende Vereinigung

(C) Als ein gescheitertes Ausbildungsmodell

(D) Als eine einflussreiche Kunstbewegung

61. Wie hat das Bauhaus die traditionelleAuffassung von Kunst verändert?

(A) Künstler sollten keine spezielle Ausbildungmehr erhalten.

(B) Die Definition von Kunst wurde erweitert.

(C) Die Architektur wurde nicht mehr alsKunst angesehen.

(D) Malerei und Skulptur sollten wenigerwichtig sein.

60. Wie sahen die Nationalsozialisten dieKunstschule Bauhaus?

(A) Als politisch links

(B) Als politisch rechts

(C) Als politisch uninteressant

(D) Als politisch gleichgesinnt

(A) Um dem politischen Druck zu entkommen

(B) Um größere Werkstätten zu bekommen

(C) Um mehr Einfluss auszuüben

(D) Um Kontakt mit anderen Künstlernaufzubauen

59. Warum ist das Bauhaus zwei Mal umgezogen?

01443-87558 APSP German Practice Exam • InDS4 • Fonts: Minion Pro Serifa, Helvetica, Times, Universal • D1 2/23/11 RI64612 • D1a 2/28/11 RI64612 • D1b 3/1/11 RI64612 • D1c 3/11/11 RI64612 • D2 3/15/11 RI64612 • D2a 3/16/11 RI64612 • D3 3/17/11 RI64612

German Language and Culture 33

B B B B B B B B B B B B B B B B

65. Sie müssen für einen Bauhausvortrag einvisuelles Beispiel auswählen. Was würde ambesten passen?

(A) Eine bunte Postkarte von Berlin

(B) Ein handgefertigter Stuhl

(C) Ein neues Buch über Gropius

(D) Ein traditionelles Ölgemälde

64. Am Ende des Vortrags wird das„Bauhausmanifest“ von Walter Gropius erwähnt.Warum zitiert die Sprecherin aus diesemManifest?

(A) Um ein Beispiel für ein Kunstwerk zugeben

(B) Um geschichtlichen Hintergrund zu geben.

(C) Um die zentrale Idee des Bauhauses zuillustrieren

(D) Um das Handwerk der Keramik zubeschreiben

63. Was macht das Bauhaus laut des Textes sobesonders?

(A) Walter Gropius gründete die Kunstschulein Weimar.

(B) Es gab keinen Unterschied zwischen Kunstund Handwerk.

(C) Die Schule bestand an drei Orten.

(D) Das Bauhaus war politisch sehr extrem.

von Bauhaus?

(A) Eine neue Universität

(B) Ein Verein für Handwerker

(C) Ein neues Kunstverständnis

(D) Eine politische Partei

62. Was bedeutet der Ausdruck „Schule“ im Kontext

01443-87558 APSP German Practice Exam • InDS4 • Fonts: Minion Pro Serifa, Helvetica, Times, Universal • D1 2/23/11 RI64612 • D1a 2/28/11 RI64612 • D1b 3/1/11 RI64612 • D1c 3/11/11 RI64612 • D2 3/15/11 RI64612 • D2a 3/16/11 RI64612 • D3 3/17/11 RI64612

German Language and Culture34

END OF PART B

B B B B B B B B B B B B B B B B

END OF SECTION I

01443-87558 APSP German Practice Exam • InDS4 • Fonts: Minion Pro Serifa, Helvetica, Times, Universal • D1 2/23/11 RI64612 • D1a 2/28/11 RI64612 • D1b 3/1/11 RI64612 • D1c 3/11/11 RI64612 • D2 3/15/11 RI64612 • D2a 3/16/11 RI64612 • D3 3/17/11 RI64612

German Language and Culture 35

DO NOT OPEN THIS BOOKLET OR BREAK THE SEAL ON THE INSERT UNTIL YOU ARE TOLD TO DO SO.

SECTION II: Free-Response Questions

AP® German Language and CulturePractice Exam

InstructionsPart A: The questions for Section II Part A are printed in the booklet.

This part is a test of your ability to write German. It consists of one InterpersonalCommunication task and one Presentational Communication task. All answers must bewritten in German. Write clearly and legibly. Write only in the lined spaces provided forthe answers. Cross out any errors you make; crossed-out work will not be scored.

Manage your time carefully. You have 15 minutes to complete Task 1. You may reviewyour response if you finish before the end of Task 1 is announced, but you may NOT goon to Task 2 until you are told to do so. The master audio recording will indicate thebeginning and end of Task 2. You may review your response for Task 2 if you finishbefore the end of the task is announced, but you may NOT go back to Task 1 nor go onto Part B.

Part B: The questions for Part B are printed in the insert. This part is a test of your abilityto speak German. It consists of one Interpersonal Communication task and onePresentational Communication task. You will be asked to record your responses. You willbe asked to start, pause, and stop your recorder several times. Carefully follow theinstructions spoken on the master recording.

At a GlanceTotal TimeApproximately 1 hour,25 minutes

Number of Tasks4

Percent of Total Score50%

Writing InstrumentPen with black or darkblue ink

Section II Part ATask 1: E-mail ReplyTime15 minutes

Task 2: Persuasive EssayTimeApproximately 55minutes

Section II Part BTask 3: ConversationTask 4: Cultural ComparisonTimeApproximately15 minutes

01443-87558 APSP German Practice Exam • InDS4 • Fonts: Minion Pro Serifa, Helvetica, Times, Universal • D1 2/23/11 RI64612 • D1a 2/28/11 RI64612 • D1b 3/1/11 RI64612 • D1c 3/11/11 RI64612 • D2 3/15/11 RI64612 • D2a 3/16/11 RI64612 • D3 3/17/11 RI64612

German Language and Culture36

GERMAN LANGUAGE AND CULTURE

Part A

You will write a reply to an e-mail message. Youhave 15 minutes to read the message and write yourreply.

Your reply should include a greeting and a closingand should respond to all the questions andrequests in the message. In your reply, you shouldalso ask for more details about somethingmentioned in the message. Also, you should use aformal form of address.

Sie werden eine E-Mail beantworten. Sie haben 15Minuten Zeit, um die Nachricht zu lesen und IhreAntwort zu schreiben.

Ihre Antwort sollte eine Begrüßungs- und eineAbschiedsformel beinhalten. Gehen Sie auf alleFragen und Anforderungen in der Nachricht ein! InIhrer Antwort sollten Sie auch nach weiterenDetails fragen, die sich auf etwas in der Nachrichtbeziehen! Zudem sollten Sie Ihre E-Mail formellgestalten.

GO ON TO THE NEXT PAGE.

Total Time — Approximately 1 hour and 25 minutes

Time — 15 minutes

Time — Approximately 1 hour and 10 minutes

1 1 1 1 1 1 1 1 1 1 1 1 1 1 1 1 1 1 1 SECTION II

01443-87558 APSP German Practice Exam • InDS4 • Fonts: Minion Pro Serifa, Helvetica, Times, Universal • D1 2/23/11 RI64612 • D1a 2/28/11 RI64612 • D1b 3/1/11 RI64612 • D1c 3/11/11 RI64612 • D2 3/15/11 RI64612 • D2a 3/16/11 RI64612 • D3 3/17/11 RI64612

German Language and Culture 37

GO ON TO THE NEXT PAGE.

1 1 1 1 1 1 1 1 1 1 1 1 1 1 1 1 1 1 1 Thema: Persönliche und Öffentliche Identität

Übersicht

Diese E-Mail ist von Maria Nollmann, Leiterin des Alten- und Pflegeheims in Dölitz. Sie bekommen dieseNachricht, weil Sie sich dafür interessieren, in dem Seniorenzentrum auszuhelfen.

Von Maria Nollmann

Betreff Ehrenamtliche Hilfe

Sehr geehrte Schülerinnen und Schüler der 11a,

vielen Dank dafür, dass Sie im Pflegezentrum Dölitz ehrenamtlich tätig werden wollen. Wirkönnen immer Hilfe gebrauchen. Besonders freuen wir uns über die Mitarbeit vonJugendlichen, da wir die Erfahrung gemacht haben, dass ein Austausch zwischen denGenerationen für beide Seiten – Senioren und Jugendliche – sehr bereichernd sein kann.

Es gibt bei uns verschiedene Möglichkeiten auszuhelfen, z.B. mit HeimbewohnernSpaziergänge zu unternehmen oder sie zum Einkauf oder zu Terminen zu begleiten. WennSie lieber flexibel sein wollen, können Sie auch einfach hin und wieder ein paar Stundenvorbeikommen und einzelne Senioren besuchen, um ein bisschen Zeit mit ihnen zuverbringen.

Am besten teilen Sie uns einfach mit, wann und wie oft Sie bei uns sein wollen und was Sieam meisten interessiert.

Wir freuen uns auf Ihre Antwort!

Mit freundlichen Grüßen

Maria Nollmann

Heimleitung, Alten- und Pflegeheim Dölitz

01443-87558 APSP German Practice Exam • InDS4 • Fonts: Minion Pro Serifa, Helvetica, Times, Universal • D1 2/23/11 RI64612 • D1a 2/28/11 RI64612 • D1b 3/1/11 RI64612 • D1c 3/11/11 RI64612 • D2 3/15/11 RI64612 • D2a 3/16/11 RI64612 • D3 3/17/11 RI64612

German Language and Culture38

Do

Not

Wri

teB

eyon

dT

his

Bor

der D

oN

otWrite

Beyond

This

Border

GO ON TO THE NEXT PAGE.

1 1 1 1 1 1 1 1 1 1 1 1 1 1 1 1 1 1 1

01443-87558 APSP German Practice Exam • InDS4 • Fonts: Minion Pro Serifa, Helvetica, Times, Universal • D1 2/23/11 RI64612 • D1a 2/28/11 RI64612 • D1b 3/1/11 RI64612 • D1c 3/11/11 RI64612 • D2 3/15/11 RI64612 • D2a 3/16/11 RI64612 • D3 3/17/11 RI64612

German Language and Culture 39

Do

Not

Wri

teB

eyon

dT

his

Bor

der D

oN

otWrite

Beyond

This

Border

1 1 1 1 1 1 1 1 1 1 1 1 1 1 1 1 1 1 1

01443-87558 APSP German Practice Exam • InDS4 • Fonts: Minion Pro Serifa, Helvetica, Times, Universal • D1 2/23/11 RI64612 • D1a 2/28/11 RI64612 • D1b 3/1/11 RI64612 • D1c 3/11/11 RI64612 • D2 3/15/11 RI64612 • D2a 3/16/11 RI64612 • D3 3/17/11 RI64612

German Language and Culture40

STOP

Do not go on until you are told to do so. Blättern Sie nicht weiter, bis Sie dazu aufgefordert werden!

1 1 1 1 1 1 1 1 1 1 1 1 1 1 1 1 1 1 1

01443-87558 APSP German Practice Exam • InDS4 • Fonts: Minion Pro Serifa, Helvetica, Times, Universal • D1 2/23/11 RI64612 • D1a 2/28/11 RI64612 • D1b 3/1/11 RI64612 • D1c 3/11/11 RI64612 • D2 3/15/11 RI64612 • D2a 3/16/11 RI64612 • D3 3/17/11 RI64612

German Language and Culture 41

You will write a persuasive essay to submit to aGerman writing contest. The essay topic is basedon three accompanying sources, which presentdifferent viewpoints on the topic and include bothprint and audio material. First, you will have 6minutes to read the essay topic and the printedmaterial. Afterward, you will hear the audiomaterial twice; you should take notes while youlisten. Then, you will have 40 minutes to prepareand write your essay.

In your persuasive essay, you should present thesources’ different viewpoints on the topic and alsoclearly indicate your own viewpoint and defend itthoroughly. Use information from all of the sourcesto support your essay. As you refer to the sources,identify them appropriately. Also, organize youressay into clear paragraphs.

Sie werden an einem deutschen Schreibwettbewerbteilnehmen und reichen einen Aufsatz ein, in demSie überzeugend und klar Ihre Argumentedarstellen. Das Thema des Aufsatzes basiert aufdrei Quellen, die jeweils einen anderen Aspektdieses Themas darstellen. Diese Quellen bestehenjeweils aus Hör- und Lesetexten. Zuerst haben Sie6 Minuten Zeit, um das Aufsatzthema und diezusätzlichen Informationen zu lesen. Danachwerden Sie den Hörtext zweimal hören. Dabeisollten Sie sich Notizen machen. Dann haben Sie40 Minuten Zeit, um den Aufsatz zu organisierenund zu schreiben.

Ihr Aufsatz sollte unterschiedliche Meinungen derQuellen zu dem Thema behandeln und Ihre eigeneMeinung dazu klar ausdrücken und verteidigen.Benutzen Sie die Informationen, die Ihnen durchdas Quellenmaterial zur Verfügung gestellt wurden,um Ihre Meinung zu begründen! Wenn Sie auf dasQuellenmaterial verweisen, identifizieren Siedieses entsprechend. Zudem sollte der Aufsatzübersichtlich in Absätze gegliedert sein.

GO ON TO THE NEXT PAGE.

Time — Approximately 55 minutes

2 2 2 2 2 2 2 2 2 2 2 2 2 2 2 2 2 2 2

Thema: Alltag

Sie haben 6 Minuten Zeit, um das Aufsatzthema, das Quellenmaterial 1 und das Quellenmaterial 2 zu lesen.

Aufsatzthema:

Wie soll man heutzutage mit Sonnenstrahlen umgehen?

01443-87558 APSP German Practice Exam • InDS4 • Fonts: Minion Pro Serifa, Helvetica, Times, Universal • D1 2/23/11 RI64612 • D1a 2/28/11 RI64612 • D1b 3/1/11 RI64612 • D1c 3/11/11 RI64612 • D2 3/15/11 RI64612 • D2a 3/16/11 RI64612 • D3 3/17/11 RI64612

German Language and Culture42

GO ON TO THE NEXT PAGE.

2 2 2 2 2 2 2 2 2 2 2 2 2 2 2 2 2 2 2 Quellenmaterial 1Übersicht

In diesem Text geht es um die Wichtigkeit des Sonnenbadens. Der ursprüngliche Artikel wurde am 7. Juli 2009 inDeutschland von der deutschen Journalistin Christine Naefeke veröffentlicht.

Sonnenbad schützt vor Krankheiten

Vitamin D – Quelle SonneDosiertes Sonnenbaden ohne Sonnencreme soll vor Herz- und Gefäßkrankheiten, vor Diabetes undKrebs schützen

„Pflanzen ohne Licht gehen ein – Menschen auch“, sagt Nicolai Worm, Professor an der DeutschenHochschule für Prävention und Gesundheitsmanagement. In seinem neuen Buch „Heilkraft D“(systemed Verlag) fasst er neue, internationale Studien zusammen.

Seine These: Wir haben die Sonne aus unserem Leben verbannt. Wir verbringen unsere Tage imBüro, im Auto und in neonbeleuchteten Fitnesscentern. Und wenn wir mal draußen sind, benutzenwir Sonnencremes mit hohem Lichtschutzfaktor. Doch ohne die UVB-Strahlen der Sonne kannunsere Haut nicht genug Vitamin D bilden. Damit nehmen wir freiwillig eine massiveUnterversorgung mit diesem gesundheitsnotwendigen Vitamin in Kauf – und riskieren nicht nurschlechte Knochen, sondern auch ein geschwächtes Immunsystem und eine ganze Reihe vonZivilisationskrankheiten von Herzinfarkt über Diabetes und Depressionen bis hin zu Krebs.

Sein Sonnen-Tipp: Setzten Sie sich im Sommerhalbjahr mindestens zweimal pro Woche ohneSonnenschutzmittel mit Gesicht, Armen und Händen – je nach Hauttyp – zwischen 10 und 30Minuten der Sonne aus. Berufstätigen würde dafür ein Spaziergang in der Mittagspause ausreichen.Bleiben Sie aber nur so kurz in der Sonne, dass Sie keinesfalls einen Sonnenbrand oder auch nureine sichtbare Hautrötung riskieren. Richtwert: Die Hälfte der Zeit, die nötig wäre, um einenSonnenbrand auszulösen, ist die sinnvollste Sonnendosis, um einen Vitamin D-Spiegel im Körperaufzubauen, der vor vielen Zivilisationskrankheiten schützt und zugleich kein Hautkrebsrisikodarstellt.

Copyrighted 2010 Rodale Inc. 69012-umas.

Zeile

5

10

15

20

01443-87558 APSP German Practice Exam • InDS4 • Fonts: Minion Pro Serifa, Helvetica, Times, Universal • D1 2/23/11 RI64612 • D1a 2/28/11 RI64612 • D1b 3/1/11 RI64612 • D1c 3/11/11 RI64612 • D2 3/15/11 RI64612 • D2a 3/16/11 RI64612 • D3 3/17/11 RI64612

German Language and Culture 43

GO ON TO THE NEXT PAGE.

2 2 2 2 2 2 2 2 2 2 2 2 2 2 2 2 2 2 2 Quellenmaterial 2Übersicht

Diese Auswahl handelt von den Aussagen Jugendlicher zum Sonnenbaden. Das ursprüngliche Diagramm wurde am14. August 2008 in Deutschland von der GfK Marktforschung veröffentlicht.

Aussagen von Jugendlichen zum Sonnenbaden und zu gebräunter Haut

Bei der Umfrage wurden 2010 in Deutschland Personen zwischen 14 und 19 Jahrenbefragt.

01443-87558 APSP German Practice Exam • InDS4 • Fonts: Minion Pro Serifa, Helvetica, Times, Universal • D1 2/23/11 RI64612 • D1a 2/28/11 RI64612 • D1b 3/1/11 RI64612 • D1c 3/11/11 RI64612 • D2 3/15/11 RI64612 • D2a 3/16/11 RI64612 • D3 3/17/11 RI64612

German Language and Culture44

GO ON TO THE NEXT PAGE.

2 2 2 2 2 2 2 2 2 2 2 2 2 2 2 2 2 2 2 Quellenmaterial 3

Sie haben 30 Sekunden Zeit, um die Übersicht zu lesen.

Übersicht

In diesem Hörtext geht es um die Gefahren des Sonnenbadens. Der ursprüngliche Bericht wurde am 18. August 2009in Deutschland von der deutschen Radiojournalistin Renate Währisch vom Bayerischen Rundfunk unter dem Titel„Sonnenbaden fördert Hautkrebs: Wie sie sich schützen können“ veröffentlicht. Sie hören die Moderatorin RenateWährisch und den Hautarzt Professor Klaus Degitz. Der Hörtext ist ca. zweieinhalb Minuten lang.

Produktionen des Bayrischen Rundfunks

01443-87558 APSP German Practice Exam • InDS4 • Fonts: Minion Pro Serifa, Helvetica, Times, Universal • D1 2/23/11 RI64612 • D1a 2/28/11 RI64612 • D1b 3/1/11 RI64612 • D1c 3/11/11 RI64612 • D2 3/15/11 RI64612 • D2a 3/16/11 RI64612 • D3 3/17/11 RI64612

German Language and Culture 45

Do

Not

Wri

teB

eyon

dT

his

Bor

der D

oN

otWrite

Beyond

This

Border

GO ON TO THE NEXT PAGE.

2 2 2 2 2 2 2 2 2 2 2 2 2 2 2 2 2 2 2

01443-87558 APSP German Practice Exam • InDS4 • Fonts: Minion Pro Serifa, Helvetica, Times, Universal • D1 2/23/11 RI64612 • D1a 2/28/11 RI64612 • D1b 3/1/11 RI64612 • D1c 3/11/11 RI64612 • D2 3/15/11 RI64612 • D2a 3/16/11 RI64612 • D3 3/17/11 RI64612

German Language and Culture46

Do

Not

Wri

teB

eyon

dT

his

Bor

der D

oN

otWrite

Beyond

This

Border

GO ON TO THE NEXT PAGE.

2 2 2 2 2 2 2 2 2 2 2 2 2 2 2 2 2 2 2

01443-87558 APSP German Practice Exam • InDS4 • Fonts: Minion Pro Serifa, Helvetica, Times, Universal • D1 2/23/11 RI64612 • D1a 2/28/11 RI64612 • D1b 3/1/11 RI64612 • D1c 3/11/11 RI64612 • D2 3/15/11 RI64612 • D2a 3/16/11 RI64612 • D3 3/17/11 RI64612

German Language and Culture 47

Do

Not

Wri

teB

eyon

dT

his

Bor

der D

oN

otWrite

Beyond

This

Border

GO ON TO THE NEXT PAGE.

2 2 2 2 2 2 2 2 2 2 2 2 2 2 2 2 2 2 2

01443-87558 APSP German Practice Exam • InDS4 • Fonts: Minion Pro Serifa, Helvetica, Times, Universal • D1 2/23/11 RI64612 • D1a 2/28/11 RI64612 • D1b 3/1/11 RI64612 • D1c 3/11/11 RI64612 • D2 3/15/11 RI64612 • D2a 3/16/11 RI64612 • D3 3/17/11 RI64612

German Language and Culture48

Do

Not

Wri

teB

eyon

dT

his

Bor

der D

oN

otWrite

Beyond

This

Border

2 2 2 2 2 2 2 2 2 2 2 2 2 2 2 2 2 2 2

GO ON TO THE NEXT PAGE.

01443-87558 APSP German Practice Exam • InDS4 • Fonts: Minion Pro Serifa, Helvetica, Times, Universal • D1 2/23/11 RI64612 • D1a 2/28/11 RI64612 • D1b 3/1/11 RI64612 • D1c 3/11/11 RI64612 • D2 3/15/11 RI64612 • D2a 3/16/11 RI64612 • D3 3/17/11 RI64612

German Language and Culture 49

Do

Not

Wri

teB

eyon

dT

his

Bor

der D

oN

otWrite

Beyond

This

Border

2 2 2 2 2 2 2 2 2 2 2 2 2 2 2 2 2 2 2

01443-87558 APSP German Practice Exam • InDS4 • Fonts: Minion Pro Serifa, Helvetica, Times, Universal • D1 2/23/11 RI64612 • D1a 2/28/11 RI64612 • D1b 3/1/11 RI64612 • D1c 3/11/11 RI64612 • D2 3/15/11 RI64612 • D2a 3/16/11 RI64612 • D3 3/17/11 RI64612

German Language and Culture50

END OF PART A

If you finish before time is called, you may check your work on Part A only. Do not go on to Part B until you are told to do so.

STOP

Sollten Sie vor der Ansage, dass die Prüfung beendet ist, fertig werden, dürfen Sie Ihre Antworten im Teil A überprüfen. Blättern Sie nicht weiter, bis Sie dazu aufgefordert werden!

2 2 2 2 2 2 2 2 2 2 2 2 2 2 2 2 2 2 2

01443-87558 APSP German Practice Exam • InDS4 • Fonts: Minion Pro Serifa, Helvetica, Times, Universal • D1 2/23/11 RI64612 • D1a 2/28/11 RI64612 • D1b 3/1/11 RI64612 • D1c 3/11/11 RI64612 • D2 3/15/11 RI64612 • D2a 3/16/11 RI64612 • D3 3/17/11 RI64612

German Language and Culture 51

SECTION II, Part B

DO NOT BREAK THE SEAL ON THIS INSERT UNTIL YOU ARE TOLD TO DO SO.

AP® German Language and CulturePractice Exam

At a GlanceTotal TimeApproximately15 minutes

Number of Tasks2

Section II Part BTask 3: ConversationTimeApproximately 5minutes

Task 4: Cultural ComparisonTimeApproximately 7minutes

InstructionsDirections for speaking will be given to you by a master recording, and you will be toldwhen to open this insert. This art consists of one Interpersonal Communication task andone Presentational Communication task. Your responses will be recorded. All responsesmust be spoken in German. Carefully follow the directions for starting, pausing, andstopping your recording equipment. Make sure that the machine is in the “Record”position when you are recording. At the completion of the exam, you should verify thatyour voice has been recorded.

p

01443-87558 APSP German Practice Exam • InDS4 • Fonts: Minion Pro Serifa, Helvetica, Times, Universal • D1 2/23/11 RI64612 • D1a 2/28/11 RI64612 • D1b 3/1/11 RI64612 • D1c 3/11/11 RI64612 • D2 3/15/11 RI64612 • D2a 3/16/11 RI64612 • D3 3/17/11 RI64612

German Language and Culture52

Part B

GO ON TO THE NEXT PAGE.

Time — Approximately 15 minutes

This part requires spoken responses. Your cue to start or stop speaking will always be this tone.

In dem folgenden Teil müssen Sie sprechen. Ihr Signal, um anzufangen oder aufzuhören, wird immer dieser Ton sein.

You have 1 minute to read the directions for this part.

Sie haben 1 Minute Zeit, um die Anweisungen für den folgenden Teil zu lesen.

Your spoken responses will be recorded. Your score will be based on what you record. It is important that you speak loudly enough and clearly enough for the machine to record what you say. You will be asked to start, pause, and stop your recorder at various points during the exam. Follow the directions and start, pause, or stop the recorder only when you are told to do so. Remember that the tone is a cue only to start or stop speaking—not to start or stop the recorder.

3 3 3 3 3 3 3 3 3 3 3 3 3 3 3 3 3 3 3

Ihre gesprochenen Antworten werden aufgenommen. Ihre Note basiert darauf, was Sie aufgenommen haben. Es ist wichtig, dass Sie laut und deutlich genug sprechen, damit die Aufnahme ganz klar ist. Sie werden an bestimmten Stellen gebeten, den Rekorder zu starten, auf Pause zu drücken oder zu stoppen. Folgen Sie den Anweisungen und starten Sie den Rekorder, drücken Sie auf Pause und stoppen Sie den Rekorder nur dann, wenn Sie explizit dazu aufgefordert werden. Denken Sie daran, dass der Ton nur ein Signal für das Anfangen und das Aufhören des Sprechens ist --- nicht dafür, wann man den Rekorder startet oder stoppt.

01443-87558 APSP German Practice Exam • InDS4 • Fonts: Minion Pro Serifa, Helvetica, Times, Universal • D1 2/23/11 RI64612 • D1a 2/28/11 RI64612 • D1b 3/1/11 RI64612 • D1c 3/11/11 RI64612 • D2 3/15/11 RI64612 • D2a 3/16/11 RI64612 • D3 3/17/11 RI64612

German Language and Culture 53

You will participate in a conversation. First, youwill have 1 minute to read a preview of theconversation, including an outline of each turn inthe conversation. Afterward, the conversation willbegin, following the outline. Each time it is yourturn to speak, you will have 20 seconds to recordyour response.

You should participate in the conversation as fullyand appropriately as possible.

Sie nehmen an einem Gespräch teil. Zuerst habenSie 1 Minute Zeit, um die Übersicht für dasGespräch zu lesen. Sie sehen auch einen Plan, dereinen Überblick jedes Austauschs zeigt. Danachbeginnt das Gespräch, welches dem Plan folgt.Jedes Mal, wenn Sie sprechen sollen, haben Sie 20Sekunden Zeit, um Ihre Antwort aufzunehmen.

Sie sollten Ihre Antworten so komplett undangemessen wie möglich gestalten.

You will now begin this task. Sie werden jetzt mit dieser Aufgabe beginnen.

You have 1 minute to read the directions for this task.

Sie haben 1 Minute Zeit, um die Anweisungen für diese Aufgabe zu lesen.

You will now begin this part. Sie werden jetzt mit diesem Teil beginnen.

GO ON TO THE NEXT PAGE.

3 3 3 3 3 3 3 3 3 3 3 3 3 3 3 3 3 3 3

01443-87558 APSP German Practice Exam • InDS4 • Fonts: Minion Pro Serifa, Helvetica, Times, Universal • D1 2/23/11 RI64612 • D1a 2/28/11 RI64612 • D1b 3/1/11 RI64612 • D1c 3/11/11 RI64612 • D2 3/15/11 RI64612 • D2a 3/16/11 RI64612 • D3 3/17/11 RI64612

German Language and Culture54

3 3 3 3 3 3 3 3 3 3 3 3 3 3 3 3 3 3 3 Thema: Persönliche und Öffentliche Identität

Sie haben 1 Minute Zeit, um die Übersicht zu lesen.

Übersicht

Dies ist ein Gespräch mit Ihrem guten Freund Martin. Sie führen dieses Gespräch, weil Martin nicht zu Ihrer Partykommen kann.

Martin • stellt Ihnen eine Frage.

Sie • antworten ihm.

Martin • spricht über ein Problem.

Sie • kommentieren und fragen warum.

Martin • sagt Ihnen einen Grund.

Sie • geben ihm einen Rat.

Martin • bittet Sie um einen Ratschlag.

Sie • machen ihm einen Vorschlag.

Martin • bedankt sich und macht einen Vorschlag.

Sie • akzeptieren und machen einen Vorschlag.

01443-87558 APSP German Practice Exam • InDS4 • Fonts: Minion Pro Serifa, Helvetica, Times, Universal • D1 2/23/11 RI64612 • D1a 2/28/11 RI64612 • D1b 3/1/11 RI64612 • D1c 3/11/11 RI64612 • D2 3/15/11 RI64612 • D2a 3/16/11 RI64612 • D3 3/17/11 RI64612

German Language and Culture 55

STOP

Do not go on until you are told to do so. Blättern Sie nicht weiter, bis Sie dazu aufgefordert werden!

3 3 3 3 3 3 3 3 3 3 3 3 3 3 3 3 3 3 3

01443-87558 APSP German Practice Exam • InDS4 • Fonts: Minion Pro Serifa, Helvetica, Times, Universal • D1 2/23/11 RI64612 • D1a 2/28/11 RI64612 • D1b 3/1/11 RI64612 • D1c 3/11/11 RI64612 • D2 3/15/11 RI64612 • D2a 3/16/11 RI64612 • D3 3/17/11 RI64612

German Language and Culture56

You will make an oral presentation on a specifictopic to your class. You will have 4 minutes to readthe presentation topic and prepare yourpresentation. Then you will have 2 minutes torecord your presentation.

In your presentation, compare your owncommunity to an area of the German-speakingworld with which you are familiar. You shoulddemonstrate your understanding of cultural featuresof the German-speaking world. You should alsoorganize your presentation clearly.

Sie halten vor Ihrer Klasse einen Vortrag über einbestimmtes Thema. Sie haben 4 Minuten Zeit, umdas Vortragsthema zu lesen und Ihren Vortragvorzubereiten. Dann haben Sie 2 Minuten Zeit, umIhren Vortrag aufzunehmen.

In Ihrem Vortrag vergleichen Sie Ihr eigenessoziales Umfeld mit einer Gegend derdeutschsprachigen Welt, mit der Sie bekannt sind.Sie sollen Ihr Verständnis der kulturellenEigenschaften der deutschsprachigen Weltbeweisen. Sie sollten Ihren Vortrag übersichtlichgliedern.

You have 1 minute to read the directions for this task.

Sie haben 1 Minute Zeit, um die Anweisungen für diese Aufgabe zu lesen.

You will now begin this task. Sie werden jetzt mit dieser Aufgabe beginnen.

4 4 4 4 4 4 4 4 4 4 4 4 4 4 4 4 4 4 4

Thema: Naturwissenschaft und Technologie

Thema des Vortrags:

Wie haben soziale Netzwerke im Internet in den letzten Jahren das Leben der Menschen in Ihrer Umgebungbeeinflusst? Vergleichen Sie Perspektiven, wo Sie wohnen, mit Perspektiven in deutschsprachigen Regionen! Siekönnen in Ihrem Vortrag Beobachtungen, Erfahrungen oder das, was Sie gelernt haben, beschreiben.

01443-87558 APSP German Practice Exam • InDS4 • Fonts: Minion Pro Serifa, Helvetica, Times, Universal • D1 2/23/11 RI64612 • D1a 2/28/11 RI64612 • D1b 3/1/11 RI64612 • D1c 3/11/11 RI64612 • D2 3/15/11 RI64612 • D2a 3/16/11 RI64612 • D3 3/17/11 RI64612

German Language and Culture 57

END OF PART B

END OF EXAM

STOP

4 4 4 4 4 4 4 4 4 4 4 4 4 4 4 4 4 4 4

01443-87558 APSP German Practice Exam • InDS4 • Fonts: Minion Pro Serifa, Helvetica, Times, Universal • D1 2/23/11 RI64612 • D1a 2/28/11 RI64612 • D1b 3/1/11 RI64612 • D1c 3/11/11 RI64612 • D2 3/15/11 RI64612 • D2a 3/16/11 RI64612 • D3 3/17/11 RI64612

German Language and Culture58

Section 1, Part B, Selection 1

Script(N) Zuerst haben Sie vier Minuten Zeit, um das Quellenmaterial eins zu lesen.

(4minutes)

(N) Die Zeit zum Lesen ist jetzt beendet. Wenden Sie sich dem Quellenmaterial zwei zu! Sie haben zwei Minuten Zeit, um die Übersicht zu lesen und die Fragen zu überfliegen.

(2minutes)

(N) Hören Sie sich jetzt das Quellenmaterial zwei an!

(MA) Nie mehr nach dem Weg fragen müssen, immer wissen, wo’s lang geht, davon konnten die meisten Fußgänger bisher nur träumen. Während immer mehr Autofahrer ein GPS-Gerät nutzen, um auf dem schnellsten Weg ans Ziel zu kommen, tun sich Fußgänger in fremden Städten oft schwer mit der Orientierung. Zwar gibt es bereits heute Mobiltelefone mit GPS-Funktion, doch die sind nicht ganz billig; außerdem liefern sie zwischen hohen Häusern oft nur eine ungenaue und im Inneren von Gebäuden gar keine Position, weil die GPS-Signale, die von Satelliten ausgesandt werden, Wände, Decken und Dächer nicht durchdringen können.

(MA) Doch jetzt gibt es eine Alternative: der „Mobile Locator“ wurde von Forschern in der Nürnberger Außenstelle des Fraunhofer-Instituts für integrierte Schaltungen (IIS) entwickelt. Er liefert exakte Koordinaten, egal ob draußen oder drinnen. Um ihn nutzen zu können, benötigt man nicht einmal ein neues Gerät. Die Software kann aufs Handy geladen werden.

(MA) [Traffıc sounds] Nürnberg ist die erste Stadt der Welt, in der man sich mit dem Mobile Locator jederzeit und überall orientieren kann. Wie das funktioniert zeigt Karin Loidl vom IIS bei einem Spaziergang vom Bahnhof zum Kinocenter Cinecitta.

(MA) [Steps] Es regnet in Strömen. Ein ideales Wetter, um die Vorteile der neuen Technik zu demonstrieren. Auf Karin Loidls Smartphone sehen wir jederzeit den richtigen Stadtplanausschnitt, unsere Position und wir können auf dem kürzesten Weg zum Kino gehen. Ohne den Mobile Locator, der auch in den verwinkelten Gässchen, die wir passiern, genau die Position anzeigt, hätten wir sicherlich länger gebraucht. Wir hätten nach dem Weg fragen müssen, im Regen den Stadtplan aus- und eingepackt und wir wären am Ende weitaus nässer gewesen.

(N) Sie haben jetzt eine Minute Zeit, um mit den Fragen für diesen Text zu beginnen. Nach einer Minute werden Sie diesen Text noch einmal hören.

(1minute)

(N) Hören Sie das noch einmal!

Repeat

(N) Sie können jetzt mit dem Beantworten der Fragen für diesen Hörtext fortfahren.

(2minutesand30seconds)

01443-87558 APSP German Practice Exam • InDS4 • Fonts: Minion Pro Serifa, Helvetica, Times, Universal • D1 2/23/11 RI64612 • D1a 2/28/11 RI64612 • D1b 3/1/11 RI64612 • D1c 3/11/11 RI64612 • D2 3/15/11 RI64612 • D2a 3/16/11 RI64612 • D3 3/17/11 RI64612

German Language and Culture 59

Section 1, Part B, Selection 2

Script(N) Zuerst haben Sie eine Minute Zeit, um das Quellenmaterial eins zu lesen.

(1minute)

(N) Die Zeit zum Lesen ist jetzt beendet. Wenden Sie sich dem Quellenmaterial zwei zu! Sie haben eine Minute Zeit, um die Übersicht zu lesen und die Fragen zu überfliegen.

(1minute)

(N) Hören Sie sich jetzt das Quellenmaterial zwei an!

(WA) Hallo Benjamin, wie läuft’s denn bei dir? Wir ham uns ja schon lange nich’ mehr gesehen. Was gibt’s denn Neues bei dir?

(MA) Oh, hi Anna. Tja, du, wie du weißt, bin ich ja endlich in die 10. gekommen und jetzt heißt es Entscheidungen zu treffen. Du weißt schon, ob Gymi oder Lehre ... was machst du denn jetzt so?

(WA) Na, ich musste mir nach meinem Abschluss letztes Jahr erst einmal eine Stelle suchen. War gar nicht so einfach. Die in der Politik sagen zwar immer, dass es genügend Lehrstellen gibt, aber die guten sind oft schnell vergeben. Ich wollte ja Bürokauffrau lernen und habe dann auch noch Glück gehabt und doch noch eine Stelle bei einem Unternehmen in der Stadt bekommen.

(MA) Wie hast du das denn geschafft?

(WA) Naja, es war gut, dass ich schon zwei Praktika gemacht hatte, die wirklich toll gelaufen sind. Dadurch hatte ich gute Empfehlungsschreiben. Mein Abschlusszeugnis war auch ganz ok. Und das hilft natürlich auch, um was zu finden. Leider weiß man nie so genau. Guck dir mal diese Stastitik von vor ein paar Jahren an. Da kann man ja wirklich sehen, wie viele sich bei großen Unternehmen beworben haben. Aber man sollte die kleinen nicht ignorieren.

(MA) Da hast du bestimmt Recht. Trotzdem sollte ich mich mal mit der Tabelle auseinander setzen.

(WA) Warum nicht? Vielleicht gibt sie dir ja den einen oder anderen Denkanstoß.

(MA) ... Jaa, du — vielleicht sollte ich mich mal bei Lufthansa bewerben, denn Autos oder Technik sind gar nicht meins. Wenn ich nur wüsste, was ich da machen könnte. ... Da kämen mir bestimmt meine Englischkenntnisse zu Gute. In dem Fach habe ich nämlich eine 1. Du, das gibt mir eine Idee. Ich schaue mal, ob ich da als Bürokaufmann ein Praktikum machen könnte. Das wäre eine echte Herausforderung. Hoffentlich stellen die Azubis ein. In den letzten Jahren waren die Stellen richtig knapp.

(N) Sie haben jetzt eine Minute Zeit, um mit den Fragen für diesen Text zu beginnen. Nach einer Minute werden Sie diesen Text noch einmal hören.

(1minute)

(N) Hören Sie das noch einmal!

Repeat

(N) Sie können jetzt mit dem Beantworten der Fragen für diesen Hörtext fortfahren.

(2minutesand30seconds)

01443-87558 APSP German Practice Exam • InDS4 • Fonts: Minion Pro Serifa, Helvetica, Times, Universal • D1 2/23/11 RI64612 • D1a 2/28/11 RI64612 • D1b 3/1/11 RI64612 • D1c 3/11/11 RI64612 • D2 3/15/11 RI64612 • D2a 3/16/11 RI64612 • D3 3/17/11 RI64612

German Language and Culture60

Section 1, Part B, Selection 3

Script(N) Zuerst haben Sie eine Minute Zeit, um die Übersicht zu lesen und die Fragen zu überfliegen.

(1minute)

(N) Hören Sie sich jetzt den Text an!

(MA) Von Dülmen im Münsterland bis nach Hollywood: So sieht die Schauspielkarriere von Franka Potente aus, die heute hier in SWR1 Leute zu Gast ist. Und diese Karriere, Franka Potente, begann im Wäschekeller Ihrer Eltern.

(WA) [Laughs] Ja, oder auch in meinem Zimmer. Also – oder auch gern im Garten, mit den Nachbarn im Sommer mal, ja.

(MA) War die Resonanz immer positiv?

(WA) Ich glaube schon, weil ich hab das immer wieder gemacht. Ich hab das A aus –, zum Geldverdienen gemacht; man musste auch Tickets erwerben für diverse Shows, die ich veranstaltet habe . . .

(MA) Wenn die Nachbarn kamen, dann mussten die dafür Geld bezahlen?

(WA) Ja, natürlich! [Laughs] Manchmal war da auch ein kleiner spontaner, äh, Flohmarkt dran gekoppelt, weil ich dachte, da kann ich noch mehr Geld machen, irgendwie. Ähm, mein Bruder, oder wahlweise Freundinnen mussten auch mitmachen. Mmh, weiß nicht mehr so genau, aber ich glaub’ das kam relativ gut an; also zumindest hab ich das so abgespeichert – aus meiner Erinnerung. Also die grundsätzliche Erfahrung war, man macht was vor und mer, es kommt gut an, man wird geliebt dafür.

(MA) Haben Ihre Eltern das immer unterstützt? Ihr Vater war Lehrer, Ihre Mutter medizinisch-technische Assistentin – ähm, haben die das immer unterstützt, die Schauspielerei?

(WA) Ach irgendwie schon, also die sind zumindest immer gekommen, wenn’s so Schulaufführungen gab, und dann irgendwann hab’ ich ja auch angefangen mich an Schauspielschulen zu bewerben – das haben die zumindest nicht unterbunden, also die – meine Eltern sind immer sehr gern auch ins Theater gegangen, ähm, und ich bin ja dann auch mal ‘ne Runde weiter gekommen, also das fanden sie dann schon irgendwie ganz spannend. Ähm, meine Mutter vorher hat immer irgendwie gesagt, „ach werd’ doch Logopädin“ aus irgend ‘nem Grund, und äh ich – bei mir hat’s zum Glück alles total pünktlich zum Abitur geklappt, also ich glaub, wenn ich erst Abi gemacht hätte, und dann noch so zwei Jahre immer rumgetingelt wäre, dann machen Eltern sich Sorgen. Aber ich wusste vorm Abitur schon, ich gehe an die Otto-Falckenberg-Schule München, äh, und deswegen entstand dieses Vakuum gar nicht, wo die irgendwie dann, “häh, was machst du denn, geh doch erst mal auf die Uni”, oder so, und . . .

(MA) Also ‘ne andere Karriere wär’ nie irgendwie denkbar gewesen? Ich meine, auch Sport spielte damals schon ‘ne Rolle in Ihrem Leben. Sie waren Torfrau bei, äh, Greven 09 – wäre das auch ‘ne Möglichkeit gewesen?

(WA) Ähm – ich hab’ das ganz gerne gemacht, aber ich weiß nur, dass ich am Ende irgendwie, weiß ich nicht. Als Torwart is’ man schon auch immer so die Person alleine hinten im Tor – also ich hab’ Handball ganz gerne gespielt, ich spiel auch gern Basketball und so – also, beruflich was Sportliches bestimmt nicht. Also das glaub’ ich nicht; da fand ich immer andere Leute auch sportlicher. Also, ich hab mich nie als sportlich –, besondere Sportskanone gesehen; also das –, so’n Feedback hab’ ich nie gekriegt. Ähm, ich glaube ich hätte Biologie studiert, das hat mich irgendwie interessiert, Journalismus – alles was mer in so ’nem – wie soll ich sagen – so ’nem provinziellen kleinstädtischen Raum sich so vorstellen kann – also der war unheimlich limitiert für mich. Heute zum Beispiel finde ich, wo ich mit Filmen zu tun habe, die Postproduktion von Filmen wahnsinnig spannend, also alles was Soundbearbeitung ist und so weiter, ähm – kannte ich gar nicht den Beruf.

(N) Sie haben jetzt eine Minute Zeit, um mit den Fragen für diesen Text zu beginnen. Nach einer Minute werden Sie diesen Text noch einmal hören.

(1minute)

(N) Hören Sie das noch einmal!

Repeat

(N) Sie können jetzt mit dem Beantworten der Fragen für diesen Hörtext fortfahren.

(1minuteand15s econds)

01443-87558 APSP German Practice Exam • InDS4 • Fonts: Minion Pro Serifa, Helvetica, Times, Universal • D1 2/23/11 RI64612 • D1a 2/28/11 RI64612 • D1b 3/1/11 RI64612 • D1c 3/11/11 RI64612 • D2 3/15/11 RI64612 • D2a 3/16/11 RI64612 • D3 3/17/11 RI64612

German Language and Culture 61

Section 1, Part B, Selection 4

Script(N) Zuerst haben Sie eine Minute Zeit, um die Übersicht zu lesen und die Fragen zu überfliegen.

(1minute)

(N) Hören Sie sich jetzt den Text an!

(MA) Herzlich willkommen zum Zahnfischer Podcast! Und wenn man jetzt mal durch den normalen Drogeriemarkt oder Supermarkt geht, dann sagt man sich schon: Ist ja alles so schön bunt hier. Da steht man vor kilometerlangen Regalen voll mit Zahnbürsten — Millionen verschiedene — so kommt es einem zumindest vor. Aber was ist jetzt die richtige Zahnbürste, Alexander Fischer?

(MB) Ja, Hallo. Gute Frage. Also, ich denke, die meisten Zahnbürsten, die auf dem Markt sind, sind einfach nur aus einem Grund auf dem Markt: damit man was auf den Markt bringt.

(MA) Wie, Moment: das heißt also, ich bräuchte den ganzen Schnick-Schnack und Schnuddelfurz gar nicht?

(MB) Ich denke, dass die Leute sich auch früher die Zähne anständig geputzt haben. Wichtig ist für mich, ähm — Natürlich gibt’s ’ne Entwicklung in vielfacher Richtung, ähm, der ist dann aber meistens nicht auf dem normalen Handzahnbürstenmarkt. Ich möcht’ mal ganz gern unterscheiden zwischen der elektrischen und der Handzahnbürste. Bei der Handzahnbürste, oder bei Patienten, die dies bevorzugen, ähm, ist es für mich wichtig, ihnen nahezulegen, dass sie sich möglichst eine Zahnbürste kaufen sollten mit weichen Borsten. Nicht mit zu groben, nicht mit zu harten Borsten, weil man kann damit ’ne Menge falsch machen, ’ne Menge kaputt machen. Das heißt, mit harten Borsten kann man sich tatsächlich das Zahnfleisch verletzen ...

(MA) Wie hygienisch ist denn meine Zahnbürste? Wenn ich mir jetzt morgens und abends so im Normalfall damit die Zähne putze und die über Tag in meinem Zahnbecher stehen lasse?

(MB) Also im Normalfall, wenn Sie sie richtig ausspülen, danach — wie gesagt — in dem Moment wo eine Zahnbürste abtrocknet, sterben die meisten Keime einfach auch mal ab. Und das sollte auch nicht das Problem ergeben in dem Moment, dass da Keime durch die Zahnpasta kommen-, durch die Zahnbürste kommen. Außerdem mach mer ja ’ne Zahnpasta drauf, die ja auch noch mal ein gewisses Maß an-, reinigend, desinfizierend wirkt, so dass sich das Problem nicht ergibt, dass man sich — was weiß ich — krank machen kann mit ’ner Zahnbürste ...

(N) Sie haben jetzt eine Minute Zeit, um mit den Fragen für diesen Text zu beginnen. Nach einer Minute werden Sie diesen Text noch einmal hören.

(1minute)

(N) Hören Sie das noch einmal!

Repeat

(N) Sie können jetzt mit dem Beantworten der Fragen für diesen Hörtext fortfahren.

(1minuteand15seconds)

01443-87558 APSP German Practice Exam • InDS4 • Fonts: Minion Pro Serifa, Helvetica, Times, Universal • D1 2/23/11 RI64612 • D1a 2/28/11 RI64612 • D1b 3/1/11 RI64612 • D1c 3/11/11 RI64612 • D2 3/15/11 RI64612 • D2a 3/16/11 RI64612 • D3 3/17/11 RI64612

German Language and Culture62

Section 1, Part B, Selection 5

Script(N) Zuerst haben Sie eine Minute Zeit, um die Übersicht zu lesen und die Fragen zu überfliegen.(1minute)(N) Hören Sie sich jetzt den Text an!(WA) Heute soll es also um das Bauhaus gehen. Das Bauhaus war, denk’ ich, ohne Zweifel die einflussreichste Kunstschule des 20. Jahrhunderts – und ihre

Bedeutung auf Design, Architektur und bildende Kunst war so bedeutend, dass sie noch bis heute anhält. Die Schule bestand an drei Orten – zuerst Weimar von 1919 bis 1925, dann Dessau von 1925 bis 1932 und dann

aber nur ganz kurz in Berlin von 1932 bis 1933. Der zweimalige Umzug geschah jeweils auf politischen Druck der rechten Parteien, die in der Weimarer Republik der 20er Jahre ja immer stärker an Einfluss gewannen. Da das Bauhaus als politisch links galt, war es denen von Anfang an ein Dorn im Auge. Daher wurde das Bauhaus 1933 dann auf Druck der nationalsozialistischen Partei auch aufgelöst. Die Nationalsozialisten waren moderner Kunst im Allgemeinen gegenüber feindlich eingestellt, und das Bauhaus verkörperte nun mal die moderne Kunst zu dieser Zeit.

So, was war also nun das besondere am Bauhaus? Wie ich schon gesagt habe, wurde das Bauhaus also 1919 ursprünglich in Weimar als Kunstschule gegründet, und

zwar von dem Architekten Walter Gropius. Dabei war das Bauhaus von Anfang an keine traditionelle Kunstschule im herkömmlichen Sinn.

Denn zu der damaligen Zeit sah die Kunstausbildung in der Regel so aus, dass die verschiedenen Kunstrichtungen ziemlich strikt von einander getrennt waren und es gab da eine ganz klare Hierarchie. Auf der einen Seite standen dabei die Bildenden Künste – d.h. Malerei, Skulptur sowie die Architektur, und auf der anderen Seite das Kunsthandwerk, also die Herstellung von Gebrauchsgegenständen wie Möbel, Geschirr, Textilien usw., letzteres galt vorrangig als Handwerk, und war der Kunst und Architektur ganz klar untergeordnet. Es gab also grob gesagt eine Unterscheidung zwischen dem Künstler, der sich den „höheren“ Dingen widmete, und dem Handwerker, der die praktischen Dinge des Alltags herstellte.

Im Bauhaus dagegen war diese Unterscheidung von Anfang an aufgehoben. Die traditionellen Künste wurden hier zusammen mit dem Handwerk unterrichtet, und es gab auch keine Unterscheidung mehr zwischen Künstler und Handwerker, und zu den Fächern, die unterrichtet wurden, gehörten neben Architektur und Malerei dann zum Beispiel auch Keramik, Möbelherstellung, graphisches und industrielles Design, Fotografie und Textildesign, etc. Walter Gropius legte diesen Grundsatz anfangs in einer Schrift nieder, die er das „Bauhausmanifest“ nannte. Darin schreibt er: „Architekten, Bildhauer, Maler, wir alle müssen zum Handwerk zurück! Denn es gibt keine „Kunst von Beruf “. Es gibt keinen Wesensunterschied zwischen dem Künstler und dem Handwerker. Der Künstler ist eine Steigerung des Handwerkers.“

(N) Sie haben jetzt eine Minute Zeit, um mit den Fragen für diesen Text zu beginnen. Nach einer Minute werden Sie diesen Text noch einmal hören.

(1minute)(N) Hören Sie das noch einmal!Repeat(N) Sie können jetzt mit dem Beantworten der Fragen für diesen Hörtext fortfahren.

(2minutes)

01443-87558 APSP German Practice Exam • InDS4 • Fonts: Minion Pro Serifa, Helvetica, Times, Universal • D1 2/23/11 RI64612 • D1a 2/28/11 RI64612 • D1b 3/1/11 RI64612 • D1c 3/11/11 RI64612 • D2 3/15/11 RI64612 • D2a 3/16/11 RI64612 • D3 3/17/11 RI64612

German Language and Culture 63

Persuasive Essay

Script(N) Sie haben eine Minute Zeit, um die Anweisungen für diese Aufgabe zu lesen.(1minute)(N) Sie werden jetzt mit dieser Aufgabe beginnen.(N) Sie haben sechs Minuten Zeit, um das Aufsatzthema, das Quellenmaterial eins und das Quellenmaterial zwei zu

lesen.(6minutes)(N) Die Zeit zum Lesen ist jetzt beendet. Wenden Sie sich jetzt dem Quellenmaterial drei zu. Sie haben dreißig

Sekunden Zeit, um die Übersicht zu lesen.(30seconds)(N) Hören Sie sich jetzt das Quellenmaterial drei an!(MA) Man weiß heute, dass die Hautkrebsrate durch das Sonnenbaden in der Freizeit gesteigert wird, und dass

deshalb auch die Hautkrebsrate in Deutschland immer noch zunimmt. In Deutschland ist geschätzt jeder fünfte Krebs mittlerweile ein Hautkrebs, und das Basalzellkarzinom ist vermutlich der häufigste Krebs in Deutschland überhaupt.

(WA) Diese Krebsart tritt vorwiegend an der Nase, an der Stirn, an der Oberlippe und, bei spärlichem Haarbewuchs, an der Glatze auf. Immerhin streut er nur selten Metastasen, doch eine Operation ist erforderlich. Wie entsteht dieses Basalzellkarzinom?

(MA) Die Sonne sendet neben Wärmestrahlung, also Infrarot, dem sichtbaren Licht, das wir mit den Augen wahrnehmen, auch die nicht wahrnehmbare UV-Strahlung, die kurzwellig und biologisch aggressiv ist und die Haut schädigen kann. Es gibt die UVB-Strahlung, die besonders kurzwellig ist, und die langwelligere UVA-Strahlung. Die UVB-Strahlung schädigt die Oberhaut und fördert Hautkrebs, die UVA-Strahlung macht dasselbe, ist aber auch noch in der Lage, das Bindegewebe der Haut zu zerstören und eine Hautalterung zu beschleunigen.

(WA) Daher rät der Dermatologe, die Sonne zu meiden und entsprechend Sonnencreme zu nutzen. Blonde, helläugige Menschen brauchen einen besonders hohen Lichtschutzfaktor.

(MA) Der Lichtschutz muss für den eigenen Hauttyp, also die Empfindlichkeit der Haut, gewählt werden. Beim Sonnenschutz ist wichtig, dass man nicht das Ziel hat, möglichst lange in der Sonne zu sein ohne Sonnenbrand, sondern dass man sich weit vom Sonnenbrand entfernt.

(WA) Die agressive Mittagssonne sollte man dringend meiden und Sonnencremes frühzeitig auftragen, also mindestens eine Viertelstunde bevor man sich den Sonnenstrahlen aussetzt. Und wenn man wasserfeste Sonnencreme wählt, erklärt der Hautarzt, dann...

(MA) dann kann man auch einige Zeit im Wasser verbringen und davon ausgehen, dass das Sonnenschutzmittel noch auf der Haut wirkt. Allerdings wird das Sonnenschutzmittel durch das Abfrottieren wieder von der Haut genommen und man muss sich dann doch wieder erneut eincremen.

(WA) Eine neue Untersuchung der Stiftung Wahrentest hat kürzlich erbracht, dass keineswegs die teuersten Sonnencremes auch die besten sind. Eine Studie ergab, dass die preiswerten Hausmarken aus dem Discounter meist die höchste Wirkung erzielen.

(N) Hören Sie das noch einmal!Repeat(N) Jetzt haben Sie vierzig Minuten Zeit, um Ihren Aufsatz vorzubereiten und zu schreiben.(40minutes)

01443-87558 APSP German Practice Exam • InDS4 • Fonts: Minion Pro Serifa, Helvetica, Times, Universal • D1 2/23/11 RI64612 • D1a 2/28/11 RI64612 • D1b 3/1/11 RI64612 • D1c 3/11/11 RI64612 • D2 3/15/11 RI64612 • D2a 3/16/11 RI64612 • D3 3/17/11 RI64612

German Language and Culture64

Conversation

Script(N) Sie haben eine Minute Zeit, um die Anweisungen für diese Aufgabe zu lesen.

(1minute)

(N) Sie werden jetzt mit dieser Aufgabe beginnen.

(N) Sie haben eine Minute Zeit, um die Übersicht zu lesen.

(1minute)

(N) Jetzt wird das Gespräch beginnen. Drücken Sie jetzt auf die Record-Taste!

(MA) Na endlich. Wird aber auch Zeit, dass du mal wieder anrufst. Was machst du denn so die ganze Zeit?

TONE

(20seconds)

TONE

(MA) Leider habe ich schlechte Nachrichten. Ich kann am Samstagabend doch nicht zu deiner Party kommen.

TONE

(20seconds)

TONE

(MA) Meine Oma wird dann 70 und wie du weißt, wird das ja immer groß gefeiert. Das wird bestimmt super langweilig. Die ganze Familie kommt; die kritisieren bestimmt wieder meine Klamotten oder meinen Nasenring — wie soll ich das nur aushalten?

TONE

(20seconds)

TONE

(MA) Oh, das mache ich. Danke. Ach — da fällt mir ein: Ich brauche doch für meine Großmutter unbedingt auch noch ein Geschenk. Hast du vielleicht eine Idee, was ich ihr kaufen könnte?

TONE

(20seconds)

TONE

(MA) Danke, das ist eine prima Idee! Jetzt fühl’ ich mich schon besser. Ich werde dich am Wochenende auf jeden Fall nochmal anrufen. Wann passt es dir denn?

TONE

(20seconds)

TONE

01443-87558 APSP German Practice Exam • InDS4 • Fonts: Minion Pro Serifa, Helvetica, Times, Universal • D1 2/23/11 RI64612 • D1a 2/28/11 RI64612 • D1b 3/1/11 RI64612 • D1c 3/11/11 RI64612 • D2 3/15/11 RI64612 • D2a 3/16/11 RI64612 • D3 3/17/11 RI64612

German Language and Culture 65

Cultural Comparison

Script(N) Sie haben eine Minute Zeit, um die Anweisungen für diese Aufgabe zu lesen.

(1minute)

(N) Sie werden jetzt mit dieser Aufgabe beginnen.

(N) Sie haben vier Minuten Zeit, um das Vortragsthema zu lesen und Ihren Vortrag vorzubereiten.

(4minutes)

(N) Sie haben zwei Minuten Zeit, um Ihren Vortrag aufzunehmen. Drücken Sie jetzt auf die Record-Taste oder wieder auf die Pause-Taste, um Ihre Aufnahme fortzusetzen! Fangen Sie an zu sprechen, sobald Sie den Signalton hören!

TONE

(2minutes)

TONE

01443-87558 APSP German Practice Exam • InDS4 • Fonts: Minion Pro Serifa, Helvetica, Times, Universal • D1 2/23/11 RI64612 • D1a 2/28/11 RI64612 • D1b 3/1/11 RI64612 • D1c 3/11/11 RI64612 • D2 3/15/11 RI64612 • D2a 3/16/11 RI64612 • D3 3/17/11 RI64612

German Language and Culture66

Notes on the Practice Exam

Introduction This section provides a detailed description of how the questions in the Practice Exam correspond to the components of the Curriculum Framework included in the AP German Language and Culture Course and Exam Description. For all questions in the Practice Exam, the course theme and the targeted learning objectives from the Curriculum Framework are indicated, and the source materials that serve as the basis for the questions are described.

For each set of multiple-choice questions, there is also an overview of the focus of the questions that lists specific aspects of communication that are assessed. For each question within a set, the correct response is justified and an explanation is provided as to why the other responses are incorrect.

For each free-response question, there is a description of the task and of the source materials on which it is based. Scoring Guidelines, as well as a summary of the characteristics of strong, good, and fair student responses, are provided for each free-response question.

Section NumberofQuestions

PercentofFinal

ScoreTime

SectionI:MultipleChoice Approx.95minutes

Part A Interpretive Communication: Print Texts 30 questions

50%

Approx. 40 minutes

Part B

Interpretive Communication: Print and Audio Texts (combined) 35 questions Approx. 55

minutesInterpretive Communication: Audio Texts

SectionII:FreeResponse Approx.85minutes

Interpersonal Writing: E-mail Reply 1 prompt

50%

15 minutes

Presentational Writing: Persuasive Essay 1 prompt Approx. 55 minutes

Interpersonal Speaking: Conversation 5 prompts20 seconds

for each response

Presentational Speaking: Cultural Comparison 1 prompt 2 minutes to

respond

01443-87558 APSP German Practice Exam • InDS4 • Fonts: Minion Pro Serifa, Helvetica, Times, Universal • D1 2/23/11 RI64612 • D1a 2/28/11 RI64612 • D1b 3/1/11 RI64612 • D1c 3/11/11 RI64612 • D2 3/15/11 RI64612 • D2a 3/16/11 RI64612 • D3 3/17/11 RI64612

German Language and Culture 67

Multiple-Choice SectionSection I of the AP Exam, the multiple-choice section, primarily assesses proficiency in the Interpretive mode of communication. Students are asked to identify main points, significant details, purpose, and intended audience of a variety of authentic print and audio texts; they are also asked to make inferences and predictions based on them. Some questions require students to show understanding of cultural or interdisciplinary information contained in a text. Each set of questions is accompanied by a preview that provides contextual information (e.g., text type, country and date of publication, statement of topic, names of characters, or speakers in the text).

Questions 1 through 30 are based on a variety of authentic print texts (e.g., journalistic and literary texts, announcements, advertisements, letters, maps, and tables).

Questions 31 through 65 are based on a variety of authentic audio materials, including interviews, podcasts, public service announcements, conversations, and brief presentations. Students will have time to read the preview and skim the questions before listening to the audio. All audio texts will be played twice. In this part of the multiple-choice section, questions 31 through 47 are based on audio texts that are paired with print texts; questions 48 through 65 are based solely on audio texts.

In total, the multiple-choice section contains 65 questions, lasts approximately 95 minutes, and accounts for 50 percent of the student’s overall AP Exam score.

01443-87558 APSP German Practice Exam • InDS4 • Fonts: Minion Pro Serifa, Helvetica, Times, Universal • D1 2/23/11 RI64612 • D1a 2/28/11 RI64612 • D1b 3/1/11 RI64612 • D1c 3/11/11 RI64612 • D2 3/15/11 RI64612 • D2a 3/16/11 RI64612 • D3 3/17/11 RI64612

German Language and Culture68

Information for Questions 1 – 5CourseTheme Contemporary Life / Alltag

Source

This advertisement was published in Germany in September 2009 by a youth hostel association. It highlights a youth hostel in Bad Marienberg, Germany. The advertisement provides information for teachers who organize field trips for their students. It contains information on programming, including a three-day adventure program with details on activities for school classes, meals, costs, and contact information.

FocusofQuestions

The following aspects of Interpretive Communication are assessed in this task:Questions 1 and 2 assess students’ ability to identify the purpose of the advertisement and the intended audience. Questions 3 and 4 assess students’ comprehension and identification of significant details mentioned in the text.Question 5 assesses the students’ ability to identify an appropriate request in order to get more information about the youth hostel.

Question 1

TargetedLearningObjective:• The student understands the purpose of a message and point of view of its

author.

(A) This option is incorrect because the text has almost none of the practical information that students need to prepare for a multiday excursion.

(B)

This option is incorrect because the text is not oriented toward recruiting individual participants for a competitive event. It does mention that there is a team competition on Day 2, but it provides no details on what that competition might be.

(C) This option is incorrect because the text is not a factual report about a past event or any experiences in the past.

(D) Thisoptioniscorrectbecauseitconveystheoverridingpurposeofthetext.Itisanadvertisement(Werbung)foractivitiesattheyouthhostel.

01443-87558 APSP German Practice Exam • InDS4 • Fonts: Minion Pro Serifa, Helvetica, Times, Universal • D1 2/23/11 RI64612 • D1a 2/28/11 RI64612 • D1b 3/1/11 RI64612 • D1c 3/11/11 RI64612 • D2 3/15/11 RI64612 • D2a 3/16/11 RI64612 • D3 3/17/11 RI64612

German Language and Culture 69

Question 2

TargetedLearningObjective:• The student identifies the distinguishing features (e.g., type of resource, intended

audience, purpose) of authentic written and print resources.(A) This option is incorrect because the advertisement focuses on school classes.

(B)

This option is incorrect because the advertisement does not address its readers as parents (e.g., “your child will be able to . . .”); instead it concentrates on telling the readers that the excursion will increase group cohesion, a concern more of teachers than of individual parents.

(C)

ThisoptioniscorrectbecausetheadvertisementtargetsapersonwhoisinapositiontoorganizeactivitiesforaGermanschoolclass(Klasse)andwhoisinterestedinteambuilding(forinstance,theprominentlyplacedKlassengemeinschaft).

(D)This option is incorrect because the text does not address college students (Studenten), nor is the term Klasse(n)gemeinschaft used in reference to college students.

Question 3

TargetedLearningObjective:• The student demonstrates comprehension of content from authentic written and

print resources.

(A) This option is incorrect because the advertisement says nothing specific about the food that will be offered when referring to meals.

(B)Thisoptioniscorrectbecausethegoalofteambuilding(Klasse(n)gemeinschaft)isfeaturedprominentlyatthetopoftheadvertisement.

(C) This option is incorrect because the advertisement makes no references to earning money.

(D)This option is incorrect because the advertisement does not indicate time for discussions. Instead, the emphasis is activities (Bau der Laubhütte, actionreiche Zeit).

Question 4

TargetedLearningObjective:• The student demonstrates comprehension of content from authentic written and

print resources.

(A) This option is incorrect because it includes breakfast only and does not reflect the meaning of Vollpension (three meals a day).

(B) This option is incorrect because the named price does include meals.

(C)

Thisoptioniscorrectbecausethepriceof€55,80refersto3 Tage mit Vollpension.Inthestandardizedlanguageofhotelsandaccommodations,thismeansthreedaysofroomingplusbreakfast,lunch,anddinner.

(D) This option is incorrect because it makes no mention of the meals that are included in the price of €55,80.

01443-87558 APSP German Practice Exam • InDS4 • Fonts: Minion Pro Serifa, Helvetica, Times, Universal • D1 2/23/11 RI64612 • D1a 2/28/11 RI64612 • D1b 3/1/11 RI64612 • D1c 3/11/11 RI64612 • D2 3/15/11 RI64612 • D2a 3/16/11 RI64612 • D3 3/17/11 RI64612

German Language and Culture70

Question 5

TargetedLearningObjective:• The student engages in the written exchange of information, opinions, and ideas

in a variety of time frames in formal situations.

(A)

This option is incorrect because the register is informal, and the request is phrased as a mild demand: the operative verb is Schickt, the ihr form of the imperative, which corresponds loosely to “Hey guys! Send me X!” This is inappropriate for a business communication.

(B)

Thisoptioniscorrectbecausetheinquiryhereisphrasedinthemostappropriate,politemanner:thewriterhasusedtheformalSieformofaddress,andtheinformationhasbeenrequestedusingasubjunctiveKönnten Sieplusbitte.

(C)This option is incorrect because it is inappropriately informal, using the du form of address. The use of modal verb möchte . . . plus bitten does not make up for the error in register.

(D)This option is incorrect because it is phrased as a demand. Although the formal Sie is used, the abruptness of the imperative combined with the equally abrupt sofort makes this inappropriate for business communications.

Information for Questions 6 – 12

CourseTheme Science and Technology / Naturwissenschaft und Technologie

Source

This text is an excerpt from the bestselling novel Die Vermessung der Welt by Daniel Kehlmann, which was published in Germany in 2005. The novel deals with the life of the scientist Alexander von Humboldt, one of the most important German scholars in the natural sciences during the 19th century. In this particular excerpt, students read about his childhood and education in the late 18th century.

FocusofQuestions

The following aspects of Interpretive Communication are assessed in this task:Questions 6 and 11 assess students’ comprehension and identification of significant details mentioned in the text. Questions 7 and 10 assess students’ comprehension of vocabulary, identifying the meaning of a word or phrase in context as it pertains to the story.Question 8 assesses students’ ability to think critically and differentiate fact from opinion. Question 9 assesses the extent to which students engage in critical reading, following the steps in logic to identify the relationship between what is stated in the text and what this statement presumably meant in von Humboldt’s real life. Question 12 assesses students’ identification of the organizational structure of the text.

01443-87558 APSP German Practice Exam • InDS4 • Fonts: Minion Pro Serifa, Helvetica, Times, Universal • D1 2/23/11 RI64612 • D1a 2/28/11 RI64612 • D1b 3/1/11 RI64612 • D1c 3/11/11 RI64612 • D2 3/15/11 RI64612 • D2a 3/16/11 RI64612 • D3 3/17/11 RI64612

German Language and Culture 71

Question 6

TargetedLearningObjective:• The student demonstrates comprehension of content from authentic written and

print resources.

(A) This option is incorrect because the text states that Humboldt was the second person in Germany to install a lightning rod.

(B) This option is incorrect because nothing in the text indicates that Humboldt lectured at a university.

(C)This option is incorrect because the text does not state that Humboldt’s journeys around the world lasted for 25 years. The text instead talks about Humboldt at a point in his life 25 years after his journeys in the tropics.

(D)

ThisoptioniscorrectbecauseitisclearlystatedinthefirstsentenceofthetextthatAlexandervonHumboldtwasfamousbecauseofhisexpeditiontothetropics(berühmt wegen einer Expedition in die Tropen).

Question 7

TargetedLearningObjective:• The student demonstrates understanding of a variety of vocabulary, including

idiomatic and culturally authentic expressions.

(A) This option is incorrect because the text does not state that Goethe was the boys’ tutor.

(B)Thisoptioniscorrectbecause,accordingtothetext,themotherdidindeedaskGoetheforadvice(Seine Mutter hatte sich bei niemand anderem als Goethe erkundigt, wie sie ihre Söhne ausbilden solle).

(C) This option is incorrect because the text does not state that Goethe was related to the Humboldts.

(D)This option is incorrect because the text does not indicate that the father of the Humboldt brothers was a friend of Goethe’s or that he had had any dealings with Goethe.

01443-87558 APSP German Practice Exam • InDS4 • Fonts: Minion Pro Serifa, Helvetica, Times, Universal • D1 2/23/11 RI64612 • D1a 2/28/11 RI64612 • D1b 3/1/11 RI64612 • D1c 3/11/11 RI64612 • D2 3/15/11 RI64612 • D2a 3/16/11 RI64612 • D3 3/17/11 RI64612

German Language and Culture72

Question 8

TargetedLearningObjective:• The student demonstrates critical reading of authentic written and print

resources in the target cultural context.

(A)

This option is incorrect because the word Experimente does not refer to something that the brothers should do, but rather to what the mother should do, namely, to educate her sons in two different ways, thereby conducting a kind of educational experiment.

(B)

Thisoptioniscorrectbecauseaccordingtothetextonebrotherwastostudyculture(Mann der Kultur),whiletheotherwastobeeducatedinthesciences(Mann der Wissenschaft).Theuseoftheone(der eine)andtheother(der andere)impliesder eine Bruder/der andere Bruder;thus,onlyonebrotherwastobecomeascientist.

(C) This option is incorrect because the phrase hoch bezahlt refers to the tutors, not the brothers.

(D) This option is incorrect because neither brother was supposed to study both literature (Kultur) and biology (Wissenschaft).

Question 9

TargetedLearningObjective:• The student demonstrates critical reading of authentic written and print

resources in the target cultural context.

(A)

Thisoptioniscorrectbecausethetextstatesthatmanyexperts(fünfzehn hochbezahlte Experten)providedinstructionfortheboys.Therigorousscheduleoftwelvehoursofinstructionperdaywithoutbreaksdoesnotdescribeatraditionalschool.Thetextstatesthatthelectureswereuniversitylevel,notatauniversity.

(B) This option is incorrect because the text does not state that they studied at a university, only that the lectures were university level (Universitätsniveau).

(C) This option is incorrect because the text does not indicate that Kunth provided instruction.

(D)This option is incorrect because the boys received instruction from experts. The text does indicate that Alexander organized beetles using his own system, but this did not apply to Wilhelm.

01443-87558 APSP German Practice Exam • InDS4 • Fonts: Minion Pro Serifa, Helvetica, Times, Universal • D1 2/23/11 RI64612 • D1a 2/28/11 RI64612 • D1b 3/1/11 RI64612 • D1c 3/11/11 RI64612 • D2 3/15/11 RI64612 • D2a 3/16/11 RI64612 • D3 3/17/11 RI64612

German Language and Culture 73

Question 10

TargetedLearningObjective:• The student demonstrates understanding of a variety of vocabulary, including

idiomatic and culturally authentic expressions.

(A)This option is incorrect because the modifying element karg does not contain any meaning relating to friendliness; just because Alexander did not speak often does not mean that he was unfriendly.

(B)ThisoptioniscorrectbecausetheadjectivewortkargisunderstoodtomeanthatAlexander’swordswerekarg(i.e.,scarce,sparse,hardtocomeby).

(C) This option is incorrect because the modifying element karg does not imply that Alexander’s words were nonexistent, merely that they were rare.

(D) This option is incorrect because the modifying element karg does not imply that Alexander’s words were without feeling or emotion.

Question 11

TargetedLearningObjective:• The student demonstrates comprehension of content from authentic written and

print resources.

(A) This option is incorrect because the text states that Alexander’s grades were fair (mittelmäßig).

(B) This option is incorrect because the text states the exact opposite: Alexander was schwächlich (weak, prone to infirmity).

(C)ThisoptioniscorrectbecausethetextportraysAlexanderasaboywho,whengivenachance(wenn man ihn sich selbst überließ),chosewalksinthewoods,collectingbugs,etc.

(D)This option is incorrect because Alexander is not identified in the text as having interest in languages. It was his brother, Wilhelm, who studied multiple languages at an early age.

Question 12

TargetedLearningObjectives:• The student demonstrates comprehension of content from authentic written and

print resources.• The student retells or summarizes information in narrative form, demonstrating

a consideration of audience.(A) This option is incorrect because the order of the three themes is reversed.

(B)Thisoptioniscorrectbecauseitaccuratelyreflectsthesequenceofmainthemesinthetext:Alexander von Humboldts Exkursionen und Erfolge, Rückblick auf Familie und Ausbildung, Charakteristika der Brüder.

(C) This option is incorrect because Rückblick auf Familie und Ausbildung is the second of the three main themes in the text, not the first.

(D) This option is incorrect because Charakteristika der Brüder is the third of the three main themes in the text, not the second.

01443-87558 APSP German Practice Exam • InDS4 • Fonts: Minion Pro Serifa, Helvetica, Times, Universal • D1 2/23/11 RI64612 • D1a 2/28/11 RI64612 • D1b 3/1/11 RI64612 • D1c 3/11/11 RI64612 • D2 3/15/11 RI64612 • D2a 3/16/11 RI64612 • D3 3/17/11 RI64612

German Language and Culture74

Information for Questions 13 – 23CourseTheme Global Challenges / Globalisierung

Source#1

This text is an excerpt from an article about the status of forests in the world. The original text was published by the German journalist Michael Miersch in 2009 in Germany. The article addresses the issue of whether or not forests are indeed disappearing and discusses the reasons why they are expanding in some regions of the world.

Source#2

This source is a map that depicts the increase and decrease of forested regions of the world. The map originally appeared on the website waldundlkima.net on February 24, 2006. It shows the relative increase or decrease of forests by means of the size of squares associated with each country.

FocusofQuestions

The following aspects of Interpretive Communication are assessed in this task:Questions 13, 15, and 20 assess ability to distinguish details of the sources, including the intended audience, the organizing principles of the first paragraph of the article, and the main purpose of the article and map.Questions 14, 19, and 20 assess comprehension of details that are presented in the article and map.Questions 16 and 22 assess the extent to which students comprehend vocabulary words or phrases in context. Questions 17 and 21 assess ability to critically read text or visual displays like a map. Question 17 evaluates whether students can find commonalities between the text and other sources that might support the author’s main point. Question 21 asks students to use the information from the visual display and the graphic’s linguistic cues to draw a conclusion about the information. Questions 18 and 23 assess general understanding of content in other disciplines, including biology and geography. Students are asked to think in general terms about why animals might be returning to areas of the desert or what the relationship is between the information in the sources here and their own understanding of geography.Question 22 also assesses the extent to which students can reflect on the article and map and the perspective of the authors in order to infer similarities with other possible sources from the target culture.

01443-87558 APSP German Practice Exam • InDS4 • Fonts: Minion Pro Serifa, Helvetica, Times, Universal • D1 2/23/11 RI64612 • D1a 2/28/11 RI64612 • D1b 3/1/11 RI64612 • D1c 3/11/11 RI64612 • D2 3/15/11 RI64612 • D2a 3/16/11 RI64612 • D3 3/17/11 RI64612

German Language and Culture 75

Question 13

TargetedLearningObjectives:• The student understands the purpose of a message and point of view of its

author. • The student identifies the distinguishing features (e.g., type of resource, intended

audience, purpose) of authentic written and print resources.

(A) This option is incorrect because someone primarily interested in traveling is not necessarily interested in the overall state of the world’s forests.

(B)This option is incorrect because someone primarily interested in the form and function of wood furniture is not necessarily interested in the trends of global forest coverage.

(C)Thisoptioniscorrectbecauseforestlossonaglobalscaleisanissuethatdirectlyspeakstopeoplewhoareinterestedintheenvironment(Umwelt).

(D) This option is incorrect because someone primarily interested in history is not necessarily interested in the current trends in forest coverage.

Question 14

TargetedLearningObjective:• The student demonstrates comprehension of content from authentic written and

print resources.

(A) This option is incorrect because the text makes it clear that the rate of decline is not as bad as it was once thought to be.

(B) This option is incorrect because there are no statements or claims in the text about the importance of forest.

(C)

Thisoptioniscorrectbecausethearticleacknowledgesadeclineinoverallforestedareasoftheworld(Wälder durch Rodungen weiterhin schwinden);ontheotherhand,therateoflossisslowing(In vielen Gebieten wird der Verlust . . . verlangsamt).

(D) This option is incorrect because the article clearly states that forest areas continue to shrink.

Question 15

TargetedLearningObjective:• The student identifies the distinguishing features (e.g., type of resource, intended

audience, purpose) of authentic written and print resources.

(A) This option is incorrect because the paragraph is phrased as a neutral and factual style that does not convey an individual’s subjective point of view.

(B) This option is incorrect because there is no contrast of views or conflicting sets of facts; the paragraph conveys a single, consistent finding.

(C) This option is incorrect because there is no mention of any sources in the paragraph.

(D)Thisoptioniscorrectbecausethefirstparagraphgivesafactualoverviewofwhatthearticleisabout:theunexpectedresilienceandevenregrowthofforests.

01443-87558 APSP German Practice Exam • InDS4 • Fonts: Minion Pro Serifa, Helvetica, Times, Universal • D1 2/23/11 RI64612 • D1a 2/28/11 RI64612 • D1b 3/1/11 RI64612 • D1c 3/11/11 RI64612 • D2 3/15/11 RI64612 • D2a 3/16/11 RI64612 • D3 3/17/11 RI64612

German Language and Culture76

Question 16

TargetedLearningObjective:• The student demonstrates understanding of a variety of vocabulary, including

idiomatic and culturally authentic expressions.

(A) This option is incorrect because the transportation of trees does not fully convey the idea that trees are cleared.

(B) This option is incorrect because tree planting results in the opposite of forest destruction and loss.

(C)Thisoptioniscorrectbecausecuttingdowntrees(fällen)istheactivitythatbestexplainsthedwindling(schwinden)referredtointhesentenceWälder schwinden weiterhin durch X.

(D) This option is incorrect because merely examining (untersuchen) trees does not necessarily result in forest loss.

Question 17

TargetedLearningObjectives:• The student demonstrates critical reading of authentic written and print

resources in the target cultural content.• The student demonstrates knowledge and understanding of content across

disciplines.

(A)ThisoptioniscorrectbecauseitaccuratelystatesthatthereisacommonelementbetweenthearticleandWeisman’sbook(howforestsnaturallyspread).

(B) This option is incorrect because the book actually agrees with the latest scientific studies; thus, it is not contradicting (widersprechen).

(C) This option is incorrect because the article does not extract any clear causal explanations of current forest loss from Weisman’s book.

(D)This option is incorrect because neither the article nor Weisman’s book adopts the position that forests will survive only if humans disappear from Earth.

Question 18

TargetedLearningObjective:• The student demonstrates knowledge and understanding of content across

disciplines.

(A)This option is incorrect because although one of three possible factors that have indirectly supported the animals’ return, climate change is not the best answer to the question.

(B) This option is incorrect because no link is made to vegetarianism and increased animal populations.

(C)ThisoptioniscorrectbecausethetextstatesthatanimalssuchasgazellesandevenfrogsarereturningtotheSahel,apartoftheSahara;moreover,theSahelisbecomingimmer grüner.

(D) This option is incorrect because no statement is made in the text about population density in the Sahara.

01443-87558 APSP German Practice Exam • InDS4 • Fonts: Minion Pro Serifa, Helvetica, Times, Universal • D1 2/23/11 RI64612 • D1a 2/28/11 RI64612 • D1b 3/1/11 RI64612 • D1c 3/11/11 RI64612 • D2 3/15/11 RI64612 • D2a 3/16/11 RI64612 • D3 3/17/11 RI64612

German Language and Culture 77

Question 19

TargetedLearningObjective:• The student demonstrates comprehension of content from authentic written and

print resources.

(A)

This option is incorrect because Grainger did not find evidence of a dramatic net growth of forests on a global scale but rather a much more limited set of evidence indicating that tropical rain-forest loss was possibly not as bad as once thought.

(B)

ThisoptioniscorrectbecauseGraingerdidfindthestatisticstobeincompletebecause,forinstance,forestshavebeengrowinginremoteareas(abgelegenen Gegenden)wherenoonenoticed(ohne dass dies jemand registriert).

(C) This option is incorrect because Grainger’s evidence showed that rain forests are not faring as badly as once thought.

(D)This option is incorrect because Grainger does not say that not enough research is being conducted on tropical forests. He believes that the statistics on tropical forests are not complete.

Question 20

TargetedLearningObjectives:• The student demonstrates comprehension of content from authentic visual

resources.• The student identifies the distinguishing features (e.g., type of resource, intended

audience, purpose) of authentic audio, visual, and audiovisual resources.

(A)

This option is incorrect because the graphic merely portrays documented trends in forest loss or gain over an unspecified number of years. It is not a prediction of the future (wie sich die Wälder weiterentwickeln werden is a future tense).

(B) This option is incorrect because the graphic gives an account of both forest growth and loss with the dynamic verbs of zu- and abnehmen.

(C)Thisoptioniscorrectbecausethegraphicpresentsrecenttrendsinforestarealossorforestareagrowth,brokendownbygeographiclocation(wo).

(D) This option is incorrect because the graphic is exclusively descriptive: it does not provide an explanation as to why.

01443-87558 APSP German Practice Exam • InDS4 • Fonts: Minion Pro Serifa, Helvetica, Times, Universal • D1 2/23/11 RI64612 • D1a 2/28/11 RI64612 • D1b 3/1/11 RI64612 • D1c 3/11/11 RI64612 • D2 3/15/11 RI64612 • D2a 3/16/11 RI64612 • D3 3/17/11 RI64612

German Language and Culture78

Question 21

TargetedLearningObjective:• The student demonstrates critical listening of authentic audio resources in the

target cultural context.

(A)This option is incorrect because China shows a large forest gain—it has a very large dark-shaded square, which the key tells us indicates forest growth.

(B)

Thisoptioniscorrectbecausethecaptionunderthegraphicexplainsthatthesizeofthesquareisanindicationofthesizeofforestlossorgain.Thelargestsquareforforestloss(indicatedwithdots)istheonelabeledBrasilien.

(C) This option is incorrect because the United States is shown with a small dark-shaded square, which means an increase in forest.

(D) This option is incorrect because Australia is shown with a small dotted square indicating forest loss.

Question 22

TargetedLearningObjectives:• The student demonstrates understanding of a variety of vocabulary, including

idiomatic and culturally authentic expressions.• The student examines, compares, and reflects on products, practices, and

perspectives of the target culture(s).• The student uses reference tools, acknowledges sources, and cites them

appropriately.

(A) This option is incorrect because the book title does not correspond to the dynamics of forest loss and growth that are the focus of the article.

(B)This option is incorrect because the book title indicates that it will discuss literary texts that focus on the forest. This is a poor match with the more factual approach of the article.

(C)

Thisoptioniscorrectbecausethebook Zustand und Zukunft des Waldesislikelytobeafactualandevenscientificaccountofhowmuchforestthereisnow(Zustand),itwillcontainpredictionsaboutthefuture(Zukunft),anditwillprobablynotbelimitedtoaparticulararea.

(D) This option is incorrect because the title indicates that the book will probably be a tourist guide to a specific forest, the Bayerischer Wald.

01443-87558 APSP German Practice Exam • InDS4 • Fonts: Minion Pro Serifa, Helvetica, Times, Universal • D1 2/23/11 RI64612 • D1a 2/28/11 RI64612 • D1b 3/1/11 RI64612 • D1c 3/11/11 RI64612 • D2 3/15/11 RI64612 • D2a 3/16/11 RI64612 • D3 3/17/11 RI64612

German Language and Culture 79

Question 23

TargetedLearningObjectives:• The student retells or summarizes information in narrative form, demonstrating

a consideration of audience.• The student demonstrates knowledge and understanding of content across

disciplines.

(A)This option is incorrect because the article does not state that fires are common in the Sahara or that they had been documented by satellite photos.

(B) This option is incorrect because the article does not give any facts about the rate of growth of cities anywhere.

(C)This option is incorrect because the article conveys the impression that deserts are getting smaller. The satellite pictures that are mentioned more likely document a reduction in desert size.

(D)Thisoptioniscorrectbecauseitsummarizesanimportantandstrikingfindingfromthearticle,namely,thatthedesert(oratleastaportionoftheSahara)isslowlybeingrepopulatedwithanimalsandplants.

Information for Questions 24 – 30CourseTheme Beauty and Aesthetics / Schönheit und Ästhetik

Source

This source is a formal letter written by Petra Zimmer to Sonja Rosenberger regarding a catalog order placed by Ms. Rosenberger. The letter explains that one of the items ordered is temporarily out of stock and asks Ms. Rosenberger if she would prefer to cancel or change her order and what her preference is regarding shipping the order.

FocusofQuestions

The following aspects of Interpretive Communication are assessed in this task:Question 24 assesses understanding of the purpose of the letter.Question 25 assesses ability to identify the writing style and tone of the letter.Questions 26 and 27 assess comprehension and identification of significant details mentioned in the text. Question 28 assesses comprehension of vocabulary, identifying the meaning of a phrase in context as it pertains to the letter.Question 29 assesses the ability to identify an appropriate written response to a formal business letter.Question 30 assesses the ability to identify an appropriate written message relaying contents of the letter to a friend.

01443-87558 APSP German Practice Exam • InDS4 • Fonts: Minion Pro Serifa, Helvetica, Times, Universal • D1 2/23/11 RI64612 • D1a 2/28/11 RI64612 • D1b 3/1/11 RI64612 • D1c 3/11/11 RI64612 • D2 3/15/11 RI64612 • D2a 3/16/11 RI64612 • D3 3/17/11 RI64612

German Language and Culture80

Question 24

TargetedLearningObjective:• The student understands the purpose of a message and point of view of its

author.

(A) This option is incorrect because the letter does not mention anything about the results of a customer survey (Kundenumfrage).

(B)Thisoptioniscorrectbecausethetextofthelettersignalsaproblemearlyon(byusingLeider)andthenexplainsthatapairofpantsissoldoutandthusunavailable(ausverkauft).

(C) This option is incorrect because the company is not advertising a special sale or new offer to Ms. Rosenberger.

(D) This option is incorrect because the letter does not provide information on new trends in clothing.

Question 25

TargetedLearningObjective:• The student demonstrates comprehension of content from authentic written and

print resources.

(A)Thisoptioniscorrectbecausetheadjectivesachlichreferstoanunemotionalandfactualstylethatcorrespondstotheprofessionalandbusinessliketoneoftheletter.

(B) This option is incorrect because the polite and businesslike tone is not in any way aggressive or demanding.

(C) This option is incorrect because there is no hint of personal emotion in the letter’s businesslike tone.

(D)This option is incorrect because the standardized language of the letter is the same as what would be used in communications with any other customer.

Question 26

TargetedLearningObjective:• The student demonstrates comprehension of content from authentic written and

print resources.

(A) This option is incorrect because the company did not write anything that would indicate that Ms. Rosenberger should come to their headquarters.

(B) This option is incorrect because the company did not ask Ms. Rosenberger to send them a letter (persönlich . . . einen Brief schicken).

(C) ThisoptioniscorrectbecauseMs.Rosenbergerisaskedtoindicateherpreferenceonapreprintedorderform(Bestellschein).

(D) This option is incorrect because the company explained that canceling her order requires only that she mark a box on the enclosed order form.

01443-87558 APSP German Practice Exam • InDS4 • Fonts: Minion Pro Serifa, Helvetica, Times, Universal • D1 2/23/11 RI64612 • D1a 2/28/11 RI64612 • D1b 3/1/11 RI64612 • D1c 3/11/11 RI64612 • D2 3/15/11 RI64612 • D2a 3/16/11 RI64612 • D3 3/17/11 RI64612

German Language and Culture 81

Question 27

TargetedLearningObjective:• The student demonstrates comprehension of content from authentic written and

print resources.

(A) This option is incorrect because the customer service telephones are available only from 9-19 Uhr, not 24 hours a day.

(B)This option is incorrect because the customer service telephones are available only from 9-19 Uhr, which is a period of ten hours, not twelve hours.

(C) This option is incorrect because the phones are open until 19 Uhr , which is 7 pm, not 9 pm.

(D)ThisoptioniscorrectbecausethetextoftheletterstatesthatcustomerscancallthecompanyanytimewiththeexceptionofSundays(Sonntage ausgenommen).

Question 28

TargetedLearningObjectives:• The student demonstrates comprehension of content from authentic written and

print resources.• The student demonstrates understanding of a variety of vocabulary, including

idiomatic and culturally authentic expressions.

(A) This option is incorrect because it is illogical for the pants to be sold out if nobody likes the quality of the product.

(B) Thisoptioniscorrectbecausealotofpeoplebuyingtheproductistheonlyanswerthatexplainsthetemporaryshortageofthepants.

(C)This option is incorrect only because the letter indicates it is a temporary shortage (im Moment). The retailer will have additional stock in just four weeks.

(D)This option is incorrect because it is illogical for the pants to sell out for weeks at a time if people are able to obtain them from many different retailers (überall).

01443-87558 APSP German Practice Exam • InDS4 • Fonts: Minion Pro Serifa, Helvetica, Times, Universal • D1 2/23/11 RI64612 • D1a 2/28/11 RI64612 • D1b 3/1/11 RI64612 • D1c 3/11/11 RI64612 • D2 3/15/11 RI64612 • D2a 3/16/11 RI64612 • D3 3/17/11 RI64612

German Language and Culture82

Question 29

TargetedLearningObjectives:• The student writes formal correspondence in a variety of media using

appropriate formats and conventions.• The student evaluates similarities and differences in the perspectives of the target

culture(s) and his/her own culture(s) as found in authentic written and print resources.

• The student demonstrates an understanding of features of target culture communities (e.g., geographic, historical, social, or political).

(A) This option is incorrect because it has a threatening and emotional tone.

(B) This option is incorrect because it is inappropriate in register and has an emotional tone.

(C) Thisoptioniscorrectbecausetheresponseisinaneutraltonethatisappropriateforabusinesstransaction.

(D) This option is incorrect because it would be inappropriate to bother the recipient with one’s back-and-forth thinking.

Question 30

TargetedLearningObjective:• The student engages in the written exchange of information, opinions, and ideas

in a variety of time frames in informal situations.

(A)This option is incorrect because the greeting Sehr geehrte would be used with a title (Herr / Frau) and last name. The greeting is inappropriately formal in an e-mail between friends that uses du.

(B)This option is incorrect because the greeting allerliebste is a greeting more often reserved for love letters, and this level of endearment clashes with the formal Sie.

(C)ThisoptioniscorrectbecausethewriterhaschosenaninformaltonereflectedintheaddressHallo Klaraandtheuseofduthatisappropriateinane-mailbetweenfriends.

(D) This option is incorrect because the e-mail confusingly combines levels of familiarity (Liebe) with more formal elements (Frau Klara Schmitt and Sie).

01443-87558 APSP German Practice Exam • InDS4 • Fonts: Minion Pro Serifa, Helvetica, Times, Universal • D1 2/23/11 RI64612 • D1a 2/28/11 RI64612 • D1b 3/1/11 RI64612 • D1c 3/11/11 RI64612 • D2 3/15/11 RI64612 • D2a 3/16/11 RI64612 • D3 3/17/11 RI64612

German Language and Culture 83

Information for Questions 31 – 40

CourseTheme Science and Technology / Naturwissenschaft und Technologie

Source#1

This source is an article on GPS navigation systems for pedestrians. Authored by Felix Rehwald, the article was published in February 2009 in the Süddeutsche Zeitung. The author provides an overview of the devices, including limitations of the technology.

Source#2

This audio report is about new technology developed by the Fraunhofer-Gesselschaft, an application-oriented research organization in Germany. The podcast reports on a field test of a new device, the Mobile Locator conducted by Karin Loidl. The original podcast „Gewusst wo“ was released in February 2009 by the Fraunhofer-Gesselschaft. The audio report is approximately two minutes long.

FocusofQuestions

The following aspects of Interpretive Communication are assessed in this task:Question 31 assesses ability to identify the intended audience of the article.Question 32 assesses comprehension of vocabulary, identifying the meaning of a phrase in context as it pertains to the article.Question 33 assesses the extent to which students engage in critical reading.Question 34 assesses the extent to which students understand the purpose of a message and point of view of its author.Questions 35 and 36 assess the ability to identify distinguishing features (e.g., purpose, tone) of authentic audio.Question 37 and 38 assess the ability to comprehend the content of the audio. Questions 39 and 40 assess the ability to think critically by identifying similarities and differences between the two sources.

01443-87558 APSP German Practice Exam • InDS4 • Fonts: Minion Pro Serifa, Helvetica, Times, Universal • D1 2/23/11 RI64612 • D1a 2/28/11 RI64612 • D1b 3/1/11 RI64612 • D1c 3/11/11 RI64612 • D2 3/15/11 RI64612 • D2a 3/16/11 RI64612 • D3 3/17/11 RI64612

German Language and Culture84

Question 31

TargetedLearningObjective:• The student understands the purpose of a message and point of view of its

author.

(A)This option is incorrect because the article gives a general overview of different devices on the market; it does not contain the in-depth technical information that would mainly be of interest to scientists.

(B)This option is incorrect because the article does not focus on commercial firms or institutions. The devices are discussed exclusively in the context of how an individual could use them.

(C)This option is incorrect because the article is not written in a style that would indicate that children or young people are the primary audience. No mention is made of that demographic.

(D)

Thisoptioniscorrectbecausethearticledescribesadevicethatassistspedestriansinfindingtheirway;thetechnologybuildsonasimilardevicesfoundincars.ThismakesitofgreatinteresttoAutofahrer und Fußgänger.

Question 32

TargetedLearningObjective:• The student demonstrates understanding of a variety of vocabulary, including

idiomatic and culturally authentic expressions.

(A) This option is incorrect because a compass is only one of the functions built into some of the devices, not a navigation system.

(B) ThisoptioniscorrectbecausetheNaviisidentifiedinthetextasaNavigationssystem . . . im Auto.

(C)This option is incorrect because the device does not actually transport anyone or anything from one place to another; it is not a Verkehrsmittel such as a car or bicycle.

(D) This option is incorrect because an Autoatlas usually refers to collection of maps bound in a book.

01443-87558 APSP German Practice Exam • InDS4 • Fonts: Minion Pro Serifa, Helvetica, Times, Universal • D1 2/23/11 RI64612 • D1a 2/28/11 RI64612 • D1b 3/1/11 RI64612 • D1c 3/11/11 RI64612 • D2 3/15/11 RI64612 • D2a 3/16/11 RI64612 • D3 3/17/11 RI64612

German Language and Culture 85

Question 33

TargetedLearningObjectives:• The student demonstrates critical reading of authentic written and print

resources in the target cultural context.• The student demonstrates knowledge and understanding of content across

disciplines.

(A)Thisoptioniscorrectbecausethetextstatesthatprofessionelle Outdoor-Geräteallowuserstoseeatopographicmapofagivenarea:Darin würden Höhenlinien, Flüsse und Brücken angezeigt.

(B)This option is incorrect because the article quotes an expert, Helmut Schmaler, who says that “you still have to find the path yourself when you’re out hiking” (man muss sich beim Wandern den Weg noch selber suchen).

(C) This option is incorrect because the article does not suggest that GPS devices provide possible leisure-time activities.

(D) This option is incorrect because the article does not say that the maps are detailed enough to identify individual trees and bushes in the environment.

Question 34

TargetedLearningObjectives:• The student understands the purpose of a message and point of view of its

author.• The student demonstrates critical reading of authentic written and print

resources in the target cultural context.

(A)This option is incorrect because the article says only that the devices are sufficient (ausreichend), not essential for Orientierung in einer fremden Stadt.

(B) This option is incorrect because the article does not suggest that nearly all hikers use the devices.

(C)

Thisoptioniscorrectbecausethearticlestatesthatthetechnologyisnicht . . . ganz ausgereift andconcludeswithanexpert’sfindingthatthedevicesare“stillalongwayfrombeingperfect”(noch weit entfernt von der Perfektion).

(D)This option is incorrect because the article portrays the development of the technology as incremental progress on existing devices and not “revolutionary.”

01443-87558 APSP German Practice Exam • InDS4 • Fonts: Minion Pro Serifa, Helvetica, Times, Universal • D1 2/23/11 RI64612 • D1a 2/28/11 RI64612 • D1b 3/1/11 RI64612 • D1c 3/11/11 RI64612 • D2 3/15/11 RI64612 • D2a 3/16/11 RI64612 • D3 3/17/11 RI64612

German Language and Culture86

Question 35

TargetedLearningObjectives:• The student understands the purpose of a message and point of view of its

author.• The student identifies the distinguishing features (e.g., type of resource, intended

audience, purpose) of authentic audio resources.• The student demonstrates critical listening of authentic audio resources in the

target cultural context.

(A) This option is incorrect because the podcast informs the audience about new technology. It does not help them find their way around Nuremberg.

(B) Thisoptioniscorrectbecausethereportdescribestheadvantagesofanewtechnologyforpedestrians.

(C)This option is incorrect because the audio segment focuses on one new technology and gives little information about the overall spectrum of work at the Frauenhofer-Institut.

(D)

This option is incorrect because the mobile locator is not a device, but rather software that can be loaded onto a cell phone (um ihn benutzen zu können, benötigt man nicht einmal ein neues Gerät. Die Software kann aufs Handy geladen werden).

Question 36

TargetedLearningObjective:• The student identifies the distinguishing features (e.g., type of resource, intended

audience, purpose) of authentic audio resources.

(A) This option is incorrect because the report does not concentrate on shortcomings of the new technology.

(B) This option is incorrect because the speaker does not use exaggerated words of praise or the tone is rather neutral.

(C)Thisoptioniscorrectbecausethepodcastconcentratesalmostexclusivelyonprovidinginformationinafactualdescriptionofanewtechnology.

(D) This option is incorrect because there is no audible laughter and no attempt on the part of the speakers to portray anything in a humorous fashion.

01443-87558 APSP German Practice Exam • InDS4 • Fonts: Minion Pro Serifa, Helvetica, Times, Universal • D1 2/23/11 RI64612 • D1a 2/28/11 RI64612 • D1b 3/1/11 RI64612 • D1c 3/11/11 RI64612 • D2 3/15/11 RI64612 • D2a 3/16/11 RI64612 • D3 3/17/11 RI64612

German Language and Culture 87

Question 37

TargetedLearningObjective:• The student demonstrates comprehension of content from authentic audio

resources.

(A) This option is incorrect because the older devices are not described as not having the capability of displaying the user’s location.

(B) This option is incorrect because the older devices are not described as only functioning during part of the year.

(C) This option is incorrect because no mention is made of purchasing software for any devices.

(D)Thisoptioniscorrectbecausethepodcaststartsbydescribingaweaknessofpreviouslyexistingsystems,notfunctioningwellbetweenorinsidebuildings.

Question 38

TargetedLearningObjective:• The student demonstrates knowledge and understanding of content across

disciplines.

(A) ThisoptioniscorrectbecauseitcorrectlysummarizeswhattheMobile Locatorisandwhatitdoes:itisnavigationsoftwareforpedestrians.

(B) This option is incorrect because no mention is made of any automobile manufacturers.

(C) This option is incorrect because the software is not designed to send signals for city traffic.

(D) This option is incorrect because the software is loaded on a cell phone and is not a satellite.

Question 39

TargetedLearningObjectives:• The student demonstrates comprehension of content from authentic audio

resources.• The student demonstrates comprehension of content from authentic written

and print resources.(A) This option is incorrect because only the article mentions compasses.

(B)

This option is incorrect because the author of the article notes that a map does not show a user where he or she is, and the podcast reporter notes that the user would have been drenched if using a paper map in the pouring rain.

(C) Thisoptioniscorrectbecauseitcorrectlycapturesthefocusofboththearticleandthepodcast,whichisOrientierungshilfenforpedestrians.

(D) This option is incorrect because neither author describes sights on a walking tour of a city.

01443-87558 APSP German Practice Exam • InDS4 • Fonts: Minion Pro Serifa, Helvetica, Times, Universal • D1 2/23/11 RI64612 • D1a 2/28/11 RI64612 • D1b 3/1/11 RI64612 • D1c 3/11/11 RI64612 • D2 3/15/11 RI64612 • D2a 3/16/11 RI64612 • D3 3/17/11 RI64612

German Language and Culture88

Question 40

TargetedLearningObjectives:• The student demonstrates comprehension of content from authentic audio

resources.• The student demonstrates comprehension of content from authentic written

and print resources.

(A) This option is incorrect because it inaccurately characterizes the content of the article and the podcast.

(B)Thisoptioniscorrectbecauseitaccuratelydescribesthearticle,whichprovidesanoverviewofdevicesandthepodcast,whichfocusesonthesystemtestedinNuremberg.

(C)

This option is incorrect because the article is more consumer-oriented, presenting skepticism about the devices and not necessarily the manufacturer’s opinion. The podcast avoids saying anything critical about the software, which indicates that it might not be an independent opinion.

(D)This option is incorrect because the article gives no concrete recommendations about specific products, and the podcast provides a positive description of only one product and mentions no other options.

01443-87558 APSP German Practice Exam • InDS4 • Fonts: Minion Pro Serifa, Helvetica, Times, Universal • D1 2/23/11 RI64612 • D1a 2/28/11 RI64612 • D1b 3/1/11 RI64612 • D1c 3/11/11 RI64612 • D2 3/15/11 RI64612 • D2a 3/16/11 RI64612 • D3 3/17/11 RI64612

German Language and Culture 89

Information for Questions 41 – 47CourseTheme Contemporary Life / Alltag

Source#1

This is a chart published in 2009 in Germany by Trendence Intitut in Berlin, which conducts surveys of students for students. The chart provides a ranking of the 12 most popular companies among students looking for apprenticeships in 2008 and 2009.

Source#2

This is a conversation between two friends, Anna and Benjamin, who know each other from school, and haven’t seen each other for a while. Benjamin is now in 10th grade and needs to make a decision soon about his future. Anna finished school last year and reports on her successful efforts to land an internship. Both discuss the chart (Source #1), and Benjamin concludes the conversation by suggesting that he might apply for an internship with Lufthansa.

FocusofQuestions

The following aspects of Interpretive Communication are assessed in this task:Questions 41 and 46 assess comprehension of vocabulary, including idiomatic expressions, and culturally authentic expressions, such as the term “ist angesagt” from the title of the chart, and the understanding of the term “Praktikum.” Questions 42 and 43 assess ability to think critically by reading the chart, making connections among numbers, cultural information, and meaning of words. Question 43 also assesses comprehension and identification of significant details mentioned in the chart. Questions 44 and 45 assess comprehension and identification of significant details mentioned in the conversation. Question 47 assesses the ability to identify an appropriate ending of the conversation.

Question 41

TargetedLearningObjective:• The student demonstrates understanding of a variety of vocabulary, including

idiomatic and culturally authentic expressions.

(A) This option is incorrect because the survey is about preferred employers (Wunscharbeitgeber); employers were not surveyed.

(B) ThisoptioniscorrectbecausethetitleofthechartindicatesthatitisaboutwhatispopularwithSchüler.

(C) This option is incorrect because nothing indicates that teachers were involved.

(D) This option is incorrect because Unternehmer are employers. The survey gathered the opinions of potential employees.

01443-87558 APSP German Practice Exam • InDS4 • Fonts: Minion Pro Serifa, Helvetica, Times, Universal • D1 2/23/11 RI64612 • D1a 2/28/11 RI64612 • D1b 3/1/11 RI64612 • D1c 3/11/11 RI64612 • D2 3/15/11 RI64612 • D2a 3/16/11 RI64612 • D3 3/17/11 RI64612

German Language and Culture90

Question 42

TargetedLearningObjectives:• The student demonstrates critical viewing of authentic visual resources in the

target cultural context.• The student demonstrates an understanding of features of target culture

communities (e.g., geographic, historical, artistic, social, or political).

(A) This option is incorrect because the information provided is not the qualities that employers are looking for in new hires.

(B) This option is incorrect because no mention is made of wages or salaries.

(C) This option is incorrect because no information is provided about income or profit.

(D) Thisoptioniscorrectbecausethegraphgivesinformationaboutwheregraduateswouldliketowork.

Question 43

TargetedLearningObjective:• The student demonstrates critical viewing of authentic visual resources in the

target cultural context.

(A) This option is incorrect because the graph shows a higher interest in Porsche in 2008 than in 2009.

(B) ThisoptioniscorrectbecausethepercentagesindicatinginterestinAdidasaregreaterin2009(9,7%)thantheywerein2008(8,5%).

(C) This option is incorrect because the graph shows a higher interest in BMW in 2008 than in 2009.

(D) This option is incorrect because the graph for Hilton Hotels shows a higher interest in 2008 than in 2009.

Question 44

TargetedLearningObjectives:• The student demonstrates comprehension of content from authentic audio

resources.• The student demonstrates an understanding of features of target culture

communities (e.g., geographic, historical, artistic, social, or political).

(A)This option is incorrect because Anna states that she wanted to be a Bürokauffrau and does not give the impression that she could not make up her mind.

(B) This option is incorrect because Anna states that her two internships—Praktika—went very well ([sie] sind toll gelaufen).

(C)

ThisoptioniscorrectbecauseitreflectswhatAnnastates,namelythatgute Stellenarefilledquickly.Thisisconveyedinthestatementthatcontrastswhatpoliticianssay(es gibt genügend Lehrstellen)withherexperience(aber die guten sind schnell vergeben).

(D) This option is incorrect because Anna’s grades were ganz OK.

01443-87558 APSP German Practice Exam • InDS4 • Fonts: Minion Pro Serifa, Helvetica, Times, Universal • D1 2/23/11 RI64612 • D1a 2/28/11 RI64612 • D1b 3/1/11 RI64612 • D1c 3/11/11 RI64612 • D2 3/15/11 RI64612 • D2a 3/16/11 RI64612 • D3 3/17/11 RI64612

German Language and Culture 91

Question 45

TargetedLearningObjective:• The student demonstrates comprehension of content from authentic audio

resources.

(A)This option is incorrect because she does not mention her English skills. It is Benjamin who talks about how his English skills could help him get an apprenticeship.

(B) This option is incorrect because she does not say anything about her application letters.

(C) This option is incorrect because she says her report card, Abschlusszeugnis, was only ganz OK, not super.

(D)ThisoptioniscorrectbecauseAnnastatesthathertwoPraktikawentwell([sie] sind toll gelaufen)andresultedingoodlettersofrecommendation.

Question 46

TargetedLearningObjective:• The student demonstrates understanding of a variety of vocabulary, including

idiomatic and culturally authentic expressions.

(A) This option is incorrect because a Praktikum refers to work experience, not learning in school.

(B) This option is incorrect because a Praktikum does not refer to time set aside to complete homework.

(C) ThisoptioniscorrectbecauseaPraktikum,aninternship,isdesignedtoprovideworkexperience.

(D) This option is incorrect because a Praktikum is not primarily designed to improve language skills.

Question 47

TargetedLearningObjective:• The student engages in the oral exchange of information, opinions, and ideas in

a variety of time frames in informal situations.

(A) This option is incorrect because the formal Sie is inappropriate for a casual conversation in which the two people have been using du all along.

(B) This option is incorrect because it is the standard closing phrase for a news interview with a well-known person, and Ihnen is a formal form of address.

(C) This option is incorrect because Benjamin has not yet found a job; therefore, Anna could not wish him well at it.

(D)Thisoptioniscorrectbecauseithasaninformaltonethatisappropriateforaconversationbetweenyoungadults(du)andbecauseitbestreflectsthecontentoftheconversation.

01443-87558 APSP German Practice Exam • InDS4 • Fonts: Minion Pro Serifa, Helvetica, Times, Universal • D1 2/23/11 RI64612 • D1a 2/28/11 RI64612 • D1b 3/1/11 RI64612 • D1c 3/11/11 RI64612 • D2 3/15/11 RI64612 • D2a 3/16/11 RI64612 • D3 3/17/11 RI64612

German Language and Culture92

Information for Questions 48 – 52

CourseTheme Personal and Public Identities / Persönliche und Öffentliche Identität

Source

This interview with the actress Franka Potente was broadcast by Südwestrundfunk in May 2009. The interview with Franka Potente was conducted by Jens Hübschen. In the converstion, Potente talks about her early interest in theater and athletics.The excerpt is about three minutes long.

FocusofQuestions

The following aspects of Interpretive Communication are assessed in this task:Question 48 assesses identification of the main idea of this excerpt of the radio broadcast. Question 49 assesses critical listening to determine whether or not students understand the point of view expressed by the two speakers. Question 50 assesses comprehension of the details about the support Potente had from her parents in her career choice.Question 51 also assesses critical listening and identification of significant detail.Question 52 assesses whether students can respond orally in the context provided. They are asked to anticipate an appropriate follow-up question based on what they have heard.

Question 48

TargetedLearningObjective:• The student demonstrates comprehension of content from authentic audio

resources.

(A) This option is incorrect because the focus of the conversation is not on what she currently does in her private life.

(B)This option is incorrect because the portion of the interview about her parents’ role is clearly focused on the past and not how they currently influence her.

(C) This option is incorrect because she is not asked to give any general statements of lessons learned from childhood.

(D) Thisoptioniscorrectbecausethefocusoftheexcerptisindeedonherearlyinterestinactingandhowthatbecamehercareer.

01443-87558 APSP German Practice Exam • InDS4 • Fonts: Minion Pro Serifa, Helvetica, Times, Universal • D1 2/23/11 RI64612 • D1a 2/28/11 RI64612 • D1b 3/1/11 RI64612 • D1c 3/11/11 RI64612 • D2 3/15/11 RI64612 • D2a 3/16/11 RI64612 • D3 3/17/11 RI64612

German Language and Culture 93

Question 49

TargetedLearningObjective:• The student understands the purpose of a message and point of view of its

author.

(A) This option is incorrect because no details about the plays that she put on as a child are provided.

(B)

ThisoptioniscorrectbecausethereisaudiblelaughterwhenMs.Potentetalksabouttheshowsthatsheputonasachild—nottoindulgeherartisticfantasy,butrathertomakemoney:Ich hab’ das . . . zum Geldverdienen gemacht.

(C) This option is incorrect because the tone of the interview is more objective when she talks about her parents’ reaction to the idea of an acting career.

(D)This option is incorrect because she does not describe peculiar ideas she had about potential careers. Although she references a suggestion made by her mother, she indicates that her only career goal was acting.

Question 50

TargetedLearningObjective:• The student demonstrates critical listening of authentic audio resources in the

target cultural context.

(A)This option is incorrect because Potente says of her parents’ initial reaction to her attending acting school was “at least they didn’t stop it” (das haben die zumindest nicht unterbunden).

(B)Thisoptioniscorrectbecauseitaccuratelyreflectswhatwehearabouthowherparentswerelukewarmabouttheideabutthenlaterseemedtoaccepthercareerchoice.

(C) This option is incorrect because it inverts the actual progression of the parents’ attitudes as described by Potente in the interview.

(D) This option is incorrect because nothing in the interview indicates that her parents tried to stop her from pursuing an acting career.

Question 51

TargetedLearningObjective:• The student demonstrates critical listening of authentic audio resources in the

target cultural context.

(A) This option is incorrect because the interviewer does not suggest teaching as a career choice for Potente. He does state that her father was a teacher.

(B)This option is incorrect because the interviewer does not suggest biology. Potente mentions biology in response to his question about a possible sports career.

(C)ThisoptioniscorrectbecausetheinterviewerasksPotentewhethersporthadbeenacareeroptionforher:Sie waren Torfrau bei, äh, Greven 09 – wäre das auch ’ne Möglichkeit gewesen?

(D)This option is incorrect because the interviewer does not mention journalism. Potente in response to his question suggests she might have become a journalist were she not an actress.

01443-87558 APSP German Practice Exam • InDS4 • Fonts: Minion Pro Serifa, Helvetica, Times, Universal • D1 2/23/11 RI64612 • D1a 2/28/11 RI64612 • D1b 3/1/11 RI64612 • D1c 3/11/11 RI64612 • D2 3/15/11 RI64612 • D2a 3/16/11 RI64612 • D3 3/17/11 RI64612

German Language and Culture94

Question 52

TargetedLearningObjectives:• The student engages in the oral exchange of information, opinions, and ideas in a

variety of time frames in formal situations.• The student demonstrates critical listening of authentic audio resources in the

target cultural context.

(A)Thisoptioniscorrectbecauseitisformal(usingSie)andalogicalextensionofapointthatPotentemakesinherlastresponseoftheinterview,suggestingthattownsizemightlimitcareerchoices.

(B)This option is incorrect because it uses an informal form of address (du), and it is illogical because her last response indicates that she has not done that type of work.

(C) This option is incorrect because the topic was already addressed in the interview.

(D) This option is incorrect because it uses an informal form of address (du) and not a next logical question to ask.

Information for Questions 53 – 57

CourseTheme Science and Technology / Naturwissenschaft und Technologie

Source

This podcast is an episode of the Zahnfischer Podcast published on April 1, 2008, online at www.podcast.de. In this interview dentist Alexander Fischer gives listeners advice about buying and using toothbrushes. He makes the point that the great variety of toothbrushes on the market is superfluous and advises against hard-bristled brushes. Fischer also explains that rinsing and allowing the toothbrush to dry after use kills bacteria.

FocusofQuestions

The following aspects of Interpretive Communication are assessed in this task:Question 53 assesses the ability to identify the intended audience: students are to understand that the podcast is not addressing experts of the field but is addressing listeners who want to avoid dental problems. Question 54 assesses the ability to infer the profession of Alexander Fischer based on content from the interview.Question 55 assesses the ability to identify the point of view of the dentist: there are too many different toothbrushes on the market. Question 56 assesses the ability to comprehend the content of the audio, and the point of view of its author. Students need to understand that hard-bristled brushes can damage teeth. Question 57 assesses ability to listen critically for details regarding sanitation.

01443-87558 APSP German Practice Exam • InDS4 • Fonts: Minion Pro Serifa, Helvetica, Times, Universal • D1 2/23/11 RI64612 • D1a 2/28/11 RI64612 • D1b 3/1/11 RI64612 • D1c 3/11/11 RI64612 • D2 3/15/11 RI64612 • D2a 3/16/11 RI64612 • D3 3/17/11 RI64612

German Language and Culture 95

Question 53

TargetedLearningObjectives:• The student understands the purpose of a message and point of view of its

author.• The student identifies the distinguishing features (e.g., type of resource, intended

audience, purpose) of authentic audio resources.

(A) Thisoptioniscorrectbecausethebroadcastisclearlyaimedatageneralaudienceoflistenerswhowanttoknowalittlemoreabouttoothbrushes.

(B) This option is incorrect because the expert does not give any insight into tooth pain or provide any remedies for a toothache.

(C) This option is incorrect because the level of information is not detailed enough to be of most interest to prospective dentists.

(D) This option is incorrect because the level of information is not technical enough to be of interest to a dental professional.

Question 54

TargetedLearningObjectives:• The student understands the purpose of a message and point of view of its

author.• The student identifies the distinguishing features (e.g., type of resource, intended

audience, purpose) of authentic audio resources.

(A) This option is incorrect because Fischer does not attempt to sell any particular product during the interview.

(B) This option is incorrect because Fischer’s level of expertise puts him above the level of a journalist.

(C)

This option is incorrect because Fischer’s expertise as demonstrated here lies more in the area of dentistry and dental hygiene than in pharmaceuticals. Toothbrushes are usually sold in Drogerien and not necessarily in Apotheken.

(D) ThisoptioniscorrectbecauseFischerdemonstratesexperienceandexpertknowledgeaboutteeth,gums,anddentalhygiene.

Question 55

TargetedLearningObjective:• The student understands the purpose of a message and point of view of its

author.

(A) This option is incorrect because Fischer criticizes the number of toothbrushes available.

(B) This option is incorrect because Fischer does not say that cheap toothbrushes are unhealthy; he does not mention prices.

(C)ThisoptioniscorrectbecauseFischersaysthatmosttoothbrushesareontheshelfonlysothatmanufacturershavesomethingtosell(sie sind nur aus einem Grund auf dem Markt: damit man was auf den Markt bringt).

(D)This option is incorrect because Fischer does not say that every person should have a different kind of toothbrush; indeed, he suggests that there are too many different toothbrushes available.

01443-87558 APSP German Practice Exam • InDS4 • Fonts: Minion Pro Serifa, Helvetica, Times, Universal • D1 2/23/11 RI64612 • D1a 2/28/11 RI64612 • D1b 3/1/11 RI64612 • D1c 3/11/11 RI64612 • D2 3/15/11 RI64612 • D2a 3/16/11 RI64612 • D3 3/17/11 RI64612

German Language and Culture96

Question 56

TargetedLearningObjective:• The student demonstrates comprehension of content from authentic audio

resources.

(A) This option is incorrect because Fischer does not take a position on whether electric or manual toothbrushes are better.

(B) ThisoptioniscorrectbecauseFischerstatesthattoothbrusheswithbristlesthataretoohardcandoalotofdamage.

(C) This option is incorrect because Fischer discusses only the qualities of the toothbrush, not where to purchase a toothbrush.

(D) This option is incorrect because Fischer does not talk about the qualities of toothpaste.

Question 57

TargetedLearningObjective:• The student demonstrates critical listening of authentic audio resources in the

target cultural context.

(A) This option is incorrect because using a different toothbrush every day is not suggested in the interview.

(B) This option is incorrect because Fischer indicates that toothpaste has a disinfecting effect.

(C)ThisoptioniscorrectbecauseFischerstatesthatgermsdieoffassoonasthetoothbrushdries(in dem Moment wo eine Zahnbürste abtrocknet, sterben die meisten Keime…).

(D) This option is incorrect because no one in the audio segment suggests cleaning the brush with soap.

01443-87558 APSP German Practice Exam • InDS4 • Fonts: Minion Pro Serifa, Helvetica, Times, Universal • D1 2/23/11 RI64612 • D1a 2/28/11 RI64612 • D1b 3/1/11 RI64612 • D1c 3/11/11 RI64612 • D2 3/15/11 RI64612 • D2a 3/16/11 RI64612 • D3 3/17/11 RI64612

German Language and Culture 97

Information for Questions 58 – 65CourseTheme Beauty and Aesthetics / Schönheit und Ästhetik

Source

This is an excerpt from a lecture on the Bauhaus school. The presenter gives an overview and short history of the Bauhaus movement and provides listeners with information on the ideology of the movement and examples of some representative works. The excerpt is approximately 3 minutes long.

FocusofQuestions

The following aspects of Interpretive Communication are assessed in this task:Question 58 assesses the ability to identify the point of view of the speaker.Questions 59 and 63 assess the ability to identify main ideas and supporting details in the audio resource.Questions 60 and 61 assess the ability demonstrate knowledge and understanding of content across disciplines. Question 61 also assesses the ability to reflect on products and perspectives of the target culture.Question 62 assesses the ability to understanding a variety of vocabulary, including idiomatic and culturally authentic expressions.Question 64 assesses the ability to use references appropriately by asking students to identify why the speaker uses a quote from the Bauhausmanifest.Question 65 assesses the ability to expound on the topic Bauhaus by identifying a suitable example.

Question 58

TargetedLearningObjective:• The student understands the purpose of a message and point of view of

its author.

(A) This option is incorrect because Bauhaus is clearly presented as a modern direction and not old-fashioned (altmodisch).

(B) This option is incorrect because nothing is said about Bauhaus having any political influence or following any political goals.

(C) This option is incorrect because it is stated that Bauhaus had distinctive ideas about art education and that it had a large impact.

(D)ThisoptioniscorrectbecausethepresenteridentifiesBauhausasmostinfluentialandstatesthatitsinfluenceondesign,architecture,andvisualartsendures.

01443-87558 APSP German Practice Exam • InDS4 • Fonts: Minion Pro Serifa, Helvetica, Times, Universal • D1 2/23/11 RI64612 • D1a 2/28/11 RI64612 • D1b 3/1/11 RI64612 • D1c 3/11/11 RI64612 • D2 3/15/11 RI64612 • D2a 3/16/11 RI64612 • D3 3/17/11 RI64612

German Language and Culture98

Question 59

TargetedLearningObjective:• The student demonstrates comprehension of content from authentic audio

resources.

(A) Thisoptioniscorrectbecausethepresenterstatesthattheschoolmovedtwicebecauseofpoliticalpressure.

(B) This option is incorrect because nothing is said about the school needing larger studios.

(C) This option is incorrect because there is no indication in the presentation that the school was hoping to expand its influence by moving.

(D) This option is incorrect because there is no indication in the presentation that the school changed locations in hopes of connecting with more artists.

Question 60

TargetedLearningObjectives:• The student examines, compares, and reflects on products, practices, and

perspectives of the target culture(s).• The student demonstrates an understanding of features of target culture

communities (e.g., geographic, historical, artistic, social, or political).• The student demonstrates knowledge and understanding of content across

disciplines.

(A) ThisoptioniscorrectbecausethepresenterstatesthattheBauhausschoolwasregardedaspoliticallyleftoriented.

(B) This option is incorrect because the presenter clearly states that the Bauhaus school was regarded as politically left oriented.

(C)This option is incorrect because the presenter notes that the National Socialist party found the political nature of the Bauhaus school threatening enough to order the closing of the school.

(D)This option is incorrect because politisch gleichgesinnt means that the Bauhaus school shared the political ideas of the National Socialist party, and the presentation indicates this was not the case.

Question 61

TargetedLearningObjective:• The student demonstrates knowledge and understanding of content across

disciplines.

(A) This option is incorrect because as a school for artists, the Bauhaus school clearly believed that artists needed a specialized education.

(B)ThisoptioniscorrectbecausethepresenterstatesthattheBauhausschooldepartedfromthetraditionalconceptofartbyincludingtheareaofKunsthandwerk(artsandcrafts).

(C) This option is incorrect because nothing is said in the presentation about architecture no longer being regarded as art by the Bauhaus school.

(D)

This option is incorrect because the presenter does not say that the Bauhaus school regarded painting and sculpture as less important, only that other disciplines such as graphic design and photography were regarded as equally important.

01443-87558 APSP German Practice Exam • InDS4 • Fonts: Minion Pro Serifa, Helvetica, Times, Universal • D1 2/23/11 RI64612 • D1a 2/28/11 RI64612 • D1b 3/1/11 RI64612 • D1c 3/11/11 RI64612 • D2 3/15/11 RI64612 • D2a 3/16/11 RI64612 • D3 3/17/11 RI64612

German Language and Culture 99

Question 62

TargetedLearningObjective:• The student demonstrates understanding of a variety of vocabulary, including

idiomatic and culturally authentic expressions.

(A) This option is incorrect because Bauhaus cannot be called a university as it did not offer academic study in a wide variety of subjects.

(B) This option is incorrect because the Bauhaus school was not a club but rather a school that viewed Handwerk as an artistic discipline.

(C)

ThisoptioniscorrectbecauseeventhoughtheBauhausschoolwasanactualinstitutionforthestudyofart,italsopropagatedanewunderstandingofartthathadalastingimpactondesign,architecture,andothervisualarts.

(D) This option is incorrect because it is clear from the presentation that the Bauhaus school was an art school and not a political party.

Question 63

TargetedLearningObjective:• The student demonstrates comprehension of content from authentic audio

resources.

(A) This option is incorrect because even though Gropius did found the school in Weimar, the location is not what made the Bauhaus special.

(B)ThisoptioniscorrectbecausethepresenterexplainsthattheBauhausschoolmadenodistinctionbetweenKunsthandwerkandBildende Kunst.

(C) This option is incorrect because even though the school existed in three different locations this is not what made the Bauhaus school special.

(D)This option is incorrect because even though it is mentioned that the Bauhaus school was seen as politically oriented, the presenter does not indicate that political views made the school special.

Question 64

TargetedLearningObjective:• The student uses reference tools, acknowledges sources, and cites them

appropriately.

(A) This option is incorrect because the citation does not describe an exemplary piece of art.

(B) This option is incorrect because the quote does not provide any historical background on the foundation of the school.

(C)ThisoptioniscorrectbecausethepresentercitestheBauhausmanifesttoillustrateabasicprincipleoftheBauhausschool,theunionofKunstandHandwerk.

(D)This option is incorrect because Keramik is not mentioned in the quote. It is only mentioned as one of the subjects that were taught at the Bauhaus school.

01443-87558 APSP German Practice Exam • InDS4 • Fonts: Minion Pro Serifa, Helvetica, Times, Universal • D1 2/23/11 RI64612 • D1a 2/28/11 RI64612 • D1b 3/1/11 RI64612 • D1c 3/11/11 RI64612 • D2 3/15/11 RI64612 • D2a 3/16/11 RI64612 • D3 3/17/11 RI64612

German Language and Culture100

Question 65

TargetedLearningObjective:• The student expounds on familiar topics and those requiring research.

(A)This option is incorrect because there is no particularly strong connection between the Bauhaus school and the city of Berlin or postcards about Berlin.

(B)Thisoptioniscorrectbecausethepresenterstatesthattheschoolhadverymodernideasaboutartandthatitincludedtheproductionofeverydayitemsinitsunderstandingandteachingofart.

(C) This option is incorrect because a book would not be a visual example of the impact and importance of the Bauhaus school.

(D) This option is incorrect because a traditional oil painting is representative of the traditional view of art that the Bauhaus school was seeking to reform.

01443-87558 APSP German Practice Exam • InDS4 • Fonts: Minion Pro Serifa, Helvetica, Times, Universal • D1 2/23/11 RI64612 • D1a 2/28/11 RI64612 • D1b 3/1/11 RI64612 • D1c 3/11/11 RI64612 • D2 3/15/11 RI64612 • D2a 3/16/11 RI64612 • D3 3/17/11 RI64612

German Language and Culture 101

Answers to Multiple-Choice Questions

1 – D2 – C3 – B4 – C5 – B6 – D7 – B8 – B9 – A10 – B11 – C12 – B13 – C14 – C15 – D16 – C17 – A

18 – C 19 – B20 – C21 – B22 – C23 – D24 – B25 – A26 – C27 – D28 – B29 – C30 – C31 – D32 – B33 – A34 – C

35 – B36 – C 37 – D38 – A39 – C40 – B41 – B42 – D43 – B44 – C45 – D46 – C47 – D48 – D49 – B50 – B51 – C

52 – A53 – A54 – D 55 – C56 – B57 – C58 – D59 – A60 – A61 – B62 – C63 – B64 – C65 – B

01443-87558 APSP German Practice Exam • InDS4 • Fonts: Minion Pro Serifa, Helvetica, Times, Universal • D1 2/23/11 RI64612 • D1a 2/28/11 RI64612 • D1b 3/1/11 RI64612 • D1c 3/11/11 RI64612 • D2 3/15/11 RI64612 • D2a 3/16/11 RI64612 • D3 3/17/11 RI64612

German Language and Culture102

Free-Response Section Section II of the AP German Language and Culture Exam, the free-response section, assesses proficiency in the Interpersonal and Presentational modes of communication. Students are asked to produce written and spoken responses in which they state and support their ideas and opinions on a variety of topics in a manner appropriate for a specific audience and situation. They also demonstrate understanding of information in print and audio source materials as well as of cultural features of the German-speaking world.

Students demonstrate their ability to write in the Interpersonal mode by reading and replying to an e-mail message. Then, using the Presentational mode, they write a persuasive essay based on three sources that present different viewpoints on a topic. Students read an article, study a table or graphic, and listen twice to a related audio. Then they write an essay in response to a prompt using the information from all three sources to present and defend their own viewpoint. Students have access to the print sources and any notes they may take on the audio during the entire writing period.

The ability to speak in the Interpersonal mode is assessed by asking students to respond to questions as part of a simulated conversation. Students are provided a preview of the conversation, including an outline of each exchange. Finally, in the Presentational mode, students are asked to plan and produce a 2-minute presentation in response to a prompt on a cultural topic. In their presentation, students compare cultural features of their own community to those found in an area of the German-speaking world with which they are familiar. Students are encouraged to cite examples from materials they’ve read, viewed, and listened to as well as from personal experiences and observations.

In total, the free-response section contains 4 tasks, lasts approximately 85 minutes, and accounts for 50 percent of the student’s overall AP Exam score.

The notes for each free-response task include a description of the task and any source material(s), and a listing of the targeted learning objectives from the Curriculum Framework. Scoring Guidelines are provided for each free-response task and a description of the characteristics of strong, good, and fair responses.

01443-87558 APSP German Practice Exam • InDS4 • Fonts: Minion Pro Serifa, Helvetica, Times, Universal • D1 2/23/11 RI64612 • D1a 2/28/11 RI64612 • D1b 3/1/11 RI64612 • D1c 3/11/11 RI64612 • D2 3/15/11 RI64612 • D2a 3/16/11 RI64612 • D3 3/17/11 RI64612

German Language and Culture 103

Information for Interpersonal Writing: E-mail Reply

CourseTheme Personal and Public Identities / Persönliche und Öffentliche Identität

DescriptionofTask

The student writes a response to an e-mail message. In the response, the student should include a greeting and closing and respond to all questions and requests in the message. The student should also ask for additional details about something mentioned in the message and use the formal form of address.

TimingThe student has 15 minutes to read the message and write the reply. The student has access to the message during the entire 15 minutes.

Source

Maria Nollmann, the director of a nursing home in Dölitz, writes an e-mail to students in an 11th grade (11a) class about doing volunteer work in her institution. She notes the benefits of encounters between teenagers and the elderly. She provides examples of possible assignments, such as going for walks together or accompanying a resident to an appointment. In her message, Ms. Nollmann asks the student to provide information on when and how often he or she might volunteer and which task would interest the student most.

TargetedLearningObjectives

• The student engages in the written exchange of information, opinions, and ideas in a variety of time frames in formal situations.

• The student writes formal correspondence in a variety of media using appropriate formats and conventions.

• The student elicits information and clarifies meaning by using a variety of strategies.

• The student initiates and sustains interaction during written interpersonal communication in a variety of media.

• The student understands a variety of vocabulary, including idiomatic and culturally appropriate expressions.

• The student uses a variety of vocabulary, including idiomatic and culturally appropriate expressions on a variety of topics.

01443-87558 APSP German Practice Exam • InDS4 • Fonts: Minion Pro Serifa, Helvetica, Times, Universal • D1 2/23/11 RI64612 • D1a 2/28/11 RI64612 • D1b 3/1/11 RI64612 • D1c 3/11/11 RI64612 • D2 3/15/11 RI64612 • D2a 3/16/11 RI64612 • D3 3/17/11 RI64612

German Language and Culture104

CharacteristicsofaSTRONGResponse(5)

The response opens with an appropriate greeting and register: Sehr geehrte Frau Nollmann. The reply contains all the information that the message from Maria Nollmann asks for: the student states he or she could come dienstags, für zwei Stunden, einkaufen gehen und quatschen. The student demonstrates control of the language by including some complex sentences in addition to compound sentences: Ich freue mich schon darauf, mit älteren Menschen zu sprechen, da ich meine Großeltern nie kennen lernte. It ends with an appropriate closure: Mit freundlichen Grüßen.

CharacteristicsofaGOODResponse(4)

Overall, an appropriate response with some elaboration that starts with formal register: Sehr geehrte Maria Nollmann. Errors do not impede comprehension. There are occasional shifts of register, such as Vielen Dank für deine E-mail. The student responds to all questions, but could provide more elaboration. For instance, the student suggests Einkaufen as one of the activities to engage in with the nursing home residents, but does not explain why he or she has identified this particular activity. The e-mail ends with an appropriate closure.

CharacteristicsofaFAIRResponse(3)

An appropriate response with limited elaboration. The message has some errors that impede comprehensibility: Ich mochte zweimal Mittwoch bei er arbeiten. There is mostly accurate use of present tense and some accuracy in other time frames. The student uses an informal register, such as beginning the e-mail with Liebe Frau Nollmann, and closes with Dein Freund Holger.

01443-87558 APSP German Practice Exam • InDS4 • Fonts: Minion Pro Serifa, Helvetica, Times, Universal • D1 2/23/11 RI64612 • D1a 2/28/11 RI64612 • D1b 3/1/11 RI64612 • D1c 3/11/11 RI64612 • D2 3/15/11 RI64612 • D2a 3/16/11 RI64612 • D3 3/17/11 RI64612

German Language and Culture 105

Scoring Guidelines for Interpersonal Writing: E-mail Reply

5Strong

• Maintains the exchange with an appropriate response and elaboration

• Fully understandable, with ease and clarity of expression; occasional errors do not impede comprehensibility

• Varied and appropriate vocabulary and idiomatic expressions

• Control of time frames; accuracy and variety in grammar, syntax, and usage, with few errors

• Consistent use of register appropriate to the situation • Variety of simple and compound sentences, and some

complex sentences

4Good

• Maintains the exchange with an appropriate response and some elaboration

• Fully understandable, with some errors, which do not impede comprehensibility

• Generally appropriate vocabulary, including some idiomatic expressions

• Accurate use of present time and mostly accurate use of other time frames; general control of grammar, syntax, and usage

• Consistent use of register appropriate to the situation except for occasional shifts

• Simple, compound, and a few complex sentences

3Fair

• Maintains the exchange with an appropriate response and limited elaboration

• Generally understandable, with errors that may impede comprehensibility

• Sufficient vocabulary, including a few idiomatic expressions

• Mostly accurate use of present time and some accuracy in other time frames; some control of grammar, syntax, and usage

• Use of register may be inappropriate, several shifts occur • Simple and a few compound sentences

2Weak

• Maintains the exchange with a minimal response • Partially understandable, with errors that force

interpretation and cause confusion for the reader• Limited vocabulary and idiomatic expressions • Some accuracy in present time and little or no accuracy

in other time frames; limited control of grammar, syntax, and usage

• Use of register is generally inappropriate to the situation• Simple sentences and phrases

01443-87558 APSP German Practice Exam • InDS4 • Fonts: Minion Pro Serifa, Helvetica, Times, Universal • D1 2/23/11 RI64612 • D1a 2/28/11 RI64612 • D1b 3/1/11 RI64612 • D1c 3/11/11 RI64612 • D2 3/15/11 RI64612 • D2a 3/16/11 RI64612 • D3 3/17/11 RI64612

German Language and Culture106

1Poor

• Unsuccessfully attempts to maintain the exchange by providing an inappropriate response

• Barely understandable, with frequent or significant errors that impede comprehensibility

• Very few vocabulary resources• Little or no control of grammar, syntax, usage, and time

frames• Little or no control of register • Simple sentences or fragments

0Unacceptable

• Mere restatement of language from the stimulus• Completely irrelevant to the stimulus• “Ich weiß nicht.” “Ich verstehe nicht.” or equivalent • Not in the language of the exam • Blank

01443-87558 APSP German Practice Exam • InDS4 • Fonts: Minion Pro Serifa, Helvetica, Times, Universal • D1 2/23/11 RI64612 • D1a 2/28/11 RI64612 • D1b 3/1/11 RI64612 • D1c 3/11/11 RI64612 • D2 3/15/11 RI64612 • D2a 3/16/11 RI64612 • D3 3/17/11 RI64612

German Language and Culture 107

Information for Presentational Writing: Persuasive EssayCourseTheme Contemporary Life / Alltag

DescriptionofTask

The student writes a persuasive essay for a German-language writing contest on the topic is exposure to the sun good or bad for one’s health? In the essay, the student presents the sources’ different viewpoints on the topic and then states and supports his or her own viewpoint.

Timing

The student has 6 minutes to read Source #1 and Source #2 (printed material). Source #3 (audio) is then played twice; the audio is approximately 2 ½ minutes long. After the second playing of the audio, the student has 40 minutes to prepare and write the essay. The student has access to the printed material and any notes taken on the audio during the entire writing period.

Source#1

This article is entitled “Sonnenbad schützt vor Krankheiten” and deals with the possible health benefits of sun exposure. The original article appeared in Germany on July 7, 2009 and was written by the German journalist Christina Naefeke.

Source#2

This graph is entitled “Aussagen von Jugendlichen zum Sonnenbaden und zu gebräunter Haut” and was put together by the GfK Martkforschung and published in Germany on August 14, 2008. The graph illustrates the opinions of German teenagers on the topic of sunbathing and tanning.

Source#3

This source is a recording by the Bayerischer Runkfunk about the possible dangers of sun exposure. The broadcast took place on August 18, 2009 in Germany and is entitled “Sonnenbaden fördert Hautkrebs: Wie sie sich schützen können.” This excerpt is about 2 1/2 minutes long and includes part of the conversation between the radio journalist Renate Währisch and the dermatologist Klaus Degitz.

TargetedLearningObjectives

• The student plans and produces written presentational communications.

• The student produces persuasive essays.• The student demonstrates comprehension of content

from authentic audio resources.• The student demonstrates comprehension of content

from authentic visual resources.• The student demonstrates comprehension of content

from written and print resources.• The student uses reference tools, acknowledges sources,

and cites them appropriately.

01443-87558 APSP German Practice Exam • InDS4 • Fonts: Minion Pro Serifa, Helvetica, Times, Universal • D1 2/23/11 RI64612 • D1a 2/28/11 RI64612 • D1b 3/1/11 RI64612 • D1c 3/11/11 RI64612 • D2 3/15/11 RI64612 • D2a 3/16/11 RI64612 • D3 3/17/11 RI64612

German Language and Culture108

CharacteristicsofaSTRONGResponse(5)

A strong response is organized into paragraphs with effective transitions and multiple references to the source materials. The student integrates some of the details of the sources into his/her argument and might use phrases such as Der Autor weist darauf hin, dass... to provide cohesion, or express an opinion with phrases such as Meiner Meinung nach kann man nicht behaupten, dass....The student uses a range of simple and compound sentences, and also complex sentences such as relative clauses or um...zu or dass or wenn. Although there may be some inaccuracy in structure, especially in complex sentences, the errors do not impede comprehensibility. The student can move beyond topics of personal interest and familiarity and discuss in more detail the physical effects of the sun on the skin. The student might make additional connections to aspects of sunbathing and health issues in German-speaking culture beyond that which is discussed in the source material. The response reflects awareness of the more formal register appropriate for a discussion on science and health issues. The student also demonstrates familiarity with most rules of capitalization and spelling in German. Errors do not impede the reader’s ability to understand the essay.

01443-87558 APSP German Practice Exam • InDS4 • Fonts: Minion Pro Serifa, Helvetica, Times, Universal • D1 2/23/11 RI64612 • D1a 2/28/11 RI64612 • D1b 3/1/11 RI64612 • D1c 3/11/11 RI64612 • D2 3/15/11 RI64612 • D2a 3/16/11 RI64612 • D3 3/17/11 RI64612

German Language and Culture 109

CharacteristicsofaGOODResponse(4)

A good response has paragraph structure and contains some transitional phrases that link ideas and main points. The student is able to summarize the source material but might not integrate the information into his/her own argument. A good response will contain some cohesive devices, such as In dem ersten Text geht es um..., or phrases such as Meiner Meinung nach to express an opinion. The student uses vocabulary suitable for discussing topics of personal interest and familiarity and, at times, culturally appropriate idioms. The student might attempt a few complex sentences, although most of the essay will contain simple and compound sentences; errors don’t impede comprehensibility. A good response demonstrates that the student is aware of the more formal type of writing that is appropriate for this kind of situation, although the writing might shift to more colloquial, informal expressions at times. The student generally pays attention to spelling and capitalization rules, and errors don’t impede comprehensibility. The student might be able to discuss, in some detail, the customs of German speakers relevant to being out in the sun or going on vacation. However, the student might describe less accurately views or attitudes associated with sun exposure, for example health risks.

CharacteristicsofaFAIRResponse(3)

A fair response indicates that a student can summarize source material, although the essay might be structured like a string of separate sentences rather than a cohesive paragraph. The student uses a few cohesive devices, including transition words like Zuerst or Zuletzt to connect sentences, or basic phrases like Ich meine or Das stimmt to express an opinion. In a fair response the student writes simple and compound sentences but rarely attempts complex sentences. Errors in structure might impede the comprehensibility of the essay. Vocabulary is typically limited to familiar thematic word groups with few idiomatic expressions. The student might display inconsistencies in capitalization and spelling, which might lead to confusion, and might shift between more informal and formal writing. The student might be able to identify some practices of German-speaking culture with respect to sun bathing or vacationing but might not describe as accurately some of the attitudes of German speakers regarding sun exposure or their views on sun tanning and its health risks.

01443-87558 APSP German Practice Exam • InDS4 • Fonts: Minion Pro Serifa, Helvetica, Times, Universal • D1 2/23/11 RI64612 • D1a 2/28/11 RI64612 • D1b 3/1/11 RI64612 • D1c 3/11/11 RI64612 • D2 3/15/11 RI64612 • D2a 3/16/11 RI64612 • D3 3/17/11 RI64612

German Language and Culture110

Scoring Guidelines for Presentational Writing: Persuasive Essay

5Strong

• Thorough and effective treatment of topic, including supporting details and relevant references to all sources

• Fully understandable, with ease and clarity of expression; occasional errors do not impede comprehensibility

• Varied and appropriate vocabulary and idiomatic expressions

• Control of time frames; accuracy and variety in grammar, syntax and usage, with few errors

• Organized essay; effective use of transitional elements or cohesive devices

• Variety of simple and compound sentences, and some complex sentences

4Good

• Effective treatment of topic, including some supporting details and mostly relevant references to all sources

• Fully understandable, with some errors, which do not impede comprehensibility

• Generally appropriate vocabulary, including some idiomatic expressions

• Accurate use of present time and mostly accurate use of other time frames; general control of grammar, syntax, and usage

• Organized essay; some effective use of transitional elements or cohesive devices

• Simple, compound, and a few complex sentences

3Fair

• Competent treatment of topic, including a few supporting details and references to all sources

• Generally understandable, with errors that may impede comprehensibility

• Sufficient vocabulary, including a few idiomatic expressions

• Mostly accurate use of present time and some accuracy in other time frames; some control of grammar, syntax, and usage

• Some organization; limited use of transitional elements or cohesive devices

• Simple and a few compound sentences

01443-87558 APSP German Practice Exam • InDS4 • Fonts: Minion Pro Serifa, Helvetica, Times, Universal • D1 2/23/11 RI64612 • D1a 2/28/11 RI64612 • D1b 3/1/11 RI64612 • D1c 3/11/11 RI64612 • D2 3/15/11 RI64612 • D2a 3/16/11 RI64612 • D3 3/17/11 RI64612

German Language and Culture 111

2Weak

• Inadequate treatment of topic, consisting mostly of statements with no development; references to sources may be inaccurate; may not refer to all sources

• Partially understandable, with errors that force interpretation and cause confusion for the reader

• Limited vocabulary and idiomatic expressions • Some accuracy in present time and little or no accuracy

in other time frames; limited control of grammar, syntax, and usage

• Inadequate organization; ineffective use of transitional elements or cohesive devices

• Simple sentences and phrases

1Poor

• Little or no treatment of topic; may consist mostly of repetition of sources or may not refer to any sources

• Barely understandable, with frequent or significant errors that impede comprehensibility

• Very few vocabulary resources • Little or no control of grammar, syntax, usage, and time

frames • Little or no organization; absence of transitional

elements and cohesive devices• Simple sentences or fragments

0Unacceptable

• Mere restatement of language from the prompt• Clearly does not respond to the prompt; completely

irrelevant to the topic • “Ich weiß nicht.” “Ich verstehe nicht.” or equivalent • Not in the language of the exam • Blank

01443-87558 APSP German Practice Exam • InDS4 • Fonts: Minion Pro Serifa, Helvetica, Times, Universal • D1 2/23/11 RI64612 • D1a 2/28/11 RI64612 • D1b 3/1/11 RI64612 • D1c 3/11/11 RI64612 • D2 3/15/11 RI64612 • D2a 3/16/11 RI64612 • D3 3/17/11 RI64612

German Language and Culture112

Information for Interpersonal Speaking: Conversation

CourseTheme Personal and Public Identities / Persönliche und Öffentliche Identität

DescriptionofTask

The student participates in a conversation with a good friend named Martin. In the conversation, the student is asked to answer a question, comment on Martin’s situation, provide advice, give a suggestion for a birthday present, and make plans to talk again.

Timing

The student has 1 minute to read the preview and an outline of each turn in the conversation; then the conversation begins. The student has 20 seconds to give each one of the 5 responses in the conversation. The student has access to the outline during the entire conversation.

Source#1 This is an audio recording that contains a series of prompts with a male speaker’s voice.

TargetedLearningObjectives

• The student engages in the oral exchange of information, opinions, and ideas in a variety of time frames in informal situations.

• The student elicits information and clarifies meaning by using a variety of strategies.

• The student states and supports opinions in oral interactions.

• The student initiates and sustains interaction through the use of various verbal and nonverbal strategies.

• The student understands a variety of vocabulary, including idiomatic and culturally appropriate expressions.

• The student uses a variety of vocabulary, including idiomatic and culturally appropriate expressions on a variety of topics.

01443-87558 APSP German Practice Exam • InDS4 • Fonts: Minion Pro Serifa, Helvetica, Times, Universal • D1 2/23/11 RI64612 • D1a 2/28/11 RI64612 • D1b 3/1/11 RI64612 • D1c 3/11/11 RI64612 • D2 3/15/11 RI64612 • D2a 3/16/11 RI64612 • D3 3/17/11 RI64612

German Language and Culture 113

CharacteristicsofaSTRONGResponse(5)

In a strong response the student maintains the conversation during each turn and consistently responds in a culturally appropriate manner. For instance, when asked to comment on Martin’s statement Leider habe ich schlechte Nachrichten. Ich kann am Samstagabend doch nicht zu deiner Party kommen, the student might respond with an appropriate phrase of disappointment like Das ist aber schade before asking a question that would elicit more detail from Martin. The student’s response typically includes some complex sentences with connectors like dass, wenn, weil or um...zu, and different time frames with a few errors that don’t impede comprehensibility. For instance, when asked Was machst du denn die ganze Zeit? the student might respond with a description of an event in the recent past. A strong response might also involve successful self-correction. The student’s choice of register indicates that he or she knows to use more informal, colloquial speech when talking to a good friend. For instance, the conversation might close with Mach’s gut!The student’s pronunciation, intonation, and pacing might contain some errors, but these errors do not impede comprehensibility for someone unaccustomed to interacting with language learners.

01443-87558 APSP German Practice Exam • InDS4 • Fonts: Minion Pro Serifa, Helvetica, Times, Universal • D1 2/23/11 RI64612 • D1a 2/28/11 RI64612 • D1b 3/1/11 RI64612 • D1c 3/11/11 RI64612 • D2 3/15/11 RI64612 • D2a 3/16/11 RI64612 • D3 3/17/11 RI64612

German Language and Culture114

CharacteristicsofaGOODResponse(4)

In a good response the student maintains the conversation consistently after most of the recorded prompts, usually answering in an appropriate manner. Most responses are simple with some accurate compound sentences, and attempts at more complex structures with some errors. For instance, in response to the question Wie soll ich das nur aushalten? the student might attempt an answer that is hypothetical and uses the subjunctive, although this part of their response might contain some errors in structure. The student at this level might self-correct successfully from time to time. A good response also demonstrates that the student can narrate and describe in all time frames, although he or she uses the present tense most accurately. The student will mostly use familiar vocabulary with some idiomatic phrases that hold the conversation together. For instance, the student might use the phrase An deiner Stelle in response to the Martin’s question about the present. A good response will also demonstrate that the student is aware of the appropriate register for an informal conversation with a good friend, although the student might shift occasionally to a more formal response. Errors in pronunciation and intonation at this level do not impede comprehensibility.

CharacteristicsofaFAIRResponse(3)

In a fair response the student maintains the conversation, although he or she interacts in a culturally appropriate manner only some of the time. The student might try to self-correct, although these corrections might not always be successful. A fair response is usually characterized by strings of simple sentences with only a few compound sentences. For instance, the student might say Blumen sind sehr schön. Omas lieben Blumen. Sie sind gut für den Geburtstag. The student typically makes errors when using the past or future tenses, but uses the present tense accurately. Vocabulary in a fair response usually consists of familiar words in thematic groups with occasional idiomatic expressions. The student’s choice of register might also be inappropriate for a conversation with a friend. For instance, the student might address Martin with Sie or use a formal greeting. Although the pronunciation, intonation, and pacing of the student’s speech is comprehensible to someone accustomed to interacting with a language learner, the listener might occasionally have difficulties understanding the conversation.

01443-87558 APSP German Practice Exam • InDS4 • Fonts: Minion Pro Serifa, Helvetica, Times, Universal • D1 2/23/11 RI64612 • D1a 2/28/11 RI64612 • D1b 3/1/11 RI64612 • D1c 3/11/11 RI64612 • D2 3/15/11 RI64612 • D2a 3/16/11 RI64612 • D3 3/17/11 RI64612

German Language and Culture 115

Scoring Guidelines for Interpersonal Speaking: Conversation

5Strong

• Maintains the exchange with appropriate responses and elaboration

• Fully understandable, with ease and clarity of expression; occasional errors do not impede comprehensibility

• Varied and appropriate vocabulary and idiomatic expressions

• Control of time frames; accuracy and variety in grammar, syntax, and usage, with few errors

• Consistent use of register appropriate for the conversation

• Pronunciation, intonation, and pacing make the response comprehensible; errors do not impede comprehensibility

• Clarification or self-correction (if present) improves comprehensibility

4Good

• Maintains the exchange with appropriate responses and some elaboration

• Fully understandable, with some errors, which do not impede comprehensibility

• Generally appropriate vocabulary, including some idiomatic expressions

• Accurate use of present time and mostly accurate use of other time frames; general control of grammar, syntax, and usage

• Consistent use of register appropriate for the conversation except for occasional shifts

• Pronunciation, intonation and pacing make the response mostly comprehensible; errors do not impede comprehensibility

• Clarification or self-correction (if present) usually improves comprehensibility

3Fair

• Maintains the exchange with appropriate responses and limited elaboration

• Generally understandable, with errors that may impede comprehensibility

• Sufficient vocabulary, including a few idiomatic expressions

• Mostly accurate use of present time and some accuracy in other time frames; some control of grammar, syntax and usage

• Use of register may be inappropriate, several shifts occur• Pronunciation, intonation, and pacing make the

response generally comprehensible; errors occasionally impede comprehensibility

• Clarification or self-correction (if present) sometimes improves comprehensibility

01443-87558 APSP German Practice Exam • InDS4 • Fonts: Minion Pro Serifa, Helvetica, Times, Universal • D1 2/23/11 RI64612 • D1a 2/28/11 RI64612 • D1b 3/1/11 RI64612 • D1c 3/11/11 RI64612 • D2 3/15/11 RI64612 • D2a 3/16/11 RI64612 • D3 3/17/11 RI64612

German Language and Culture116

2Weak

• Maintains the exchange with minimal responses • Partially understandable, with errors that force

interpretation and cause confusion for the listener • Limited vocabulary and idiomatic expressions • Some accuracy in present time and little or no accuracy

in other time frames; limited control of grammar, syntax, and usage

• Use of register is generally inappropriate for the conversation

• Pronunciation, intonation, and pacing make the response difficult to comprehend at times; errors impede comprehensibility

• Clarification or self-correction (if present) usually does not improve comprehensibility

1Poor

• Unsuccessfully attempts to maintain the exchange by providing inappropriate responses

• Barely understandable, with frequent or significant errors that impede comprehensibility

• Very few vocabulary resources • Little or no control of grammar, syntax, usage, and time

frames • Little or no control of register • Pronunciation, intonation, and pacing make the

response difficult to comprehend; errors impede comprehensibility

• Clarification or self-correction (if present) does not improve comprehensibility; does not recognize errors

0Unacceptable

• Mere restatement of language from the prompts• Clearly does not respond to the prompts• “Ich weiß nicht.” “Ich verstehe nicht.” or equivalent • Not in the language of the exam • Blank (although recording equipment is functioning)

01443-87558 APSP German Practice Exam • InDS4 • Fonts: Minion Pro Serifa, Helvetica, Times, Universal • D1 2/23/11 RI64612 • D1a 2/28/11 RI64612 • D1b 3/1/11 RI64612 • D1c 3/11/11 RI64612 • D2 3/15/11 RI64612 • D2a 3/16/11 RI64612 • D3 3/17/11 RI64612

German Language and Culture 117

Information for Presentational Speaking: Cultural Comparison

CourseTheme Science and Technology / Naturwissenschaft und Technologie

DescriptionofTask

The student records an oral presentation on how social networks have influenced the lives of people in the community. The student is asked to compare his or her own community to an area of the German-speaking world with which he or she is familiar. The student should demonstrate understanding of cultural features of the German-speaking world. The student should also organize the presentation clearly.

TimingThe student has 4 minutes to read the presentation topic and prepare the presentation. Then, the student has 2 minutes to record the presentation.

TargetedLearningObjectives

• The student plans and produces spoken presentational communications.

• The student expounds on familiar topics and those requiring research.

• The student demonstrates an understanding of features of target culture communities (e.g., geographic, historical, artistic, social, or political).

CharacteristicsofaSTRONGResponse(5)

The presentation is organized and effectively addresses the topic, including supporting details and relevant examples: it might compare the use of Facebook and MySpace in a school in the U.S. with a school in a German-speaking country.The presentation is fully understandable and the student demonstrates an ease of expression, using transitional elements, and idiomatic expressions: Ich weiß nicht genau, wie lange ich schon auf Facebook bin, aber es fühlt sich an wie seit Ewigkeiten. The student uses different time frames, often accurately.Pronunciation, intonation, and pacing make it easy to follow the ideas, and the occasional errors do not impede comprehensibility. The student uses the informal register when addressing the audience of his/her classmates: Ihr kennt ja sicher alle mindestens ein soziales Netzwerk. The student demonstrates control of time frames and a variety of vocabulary.

01443-87558 APSP German Practice Exam • InDS4 • Fonts: Minion Pro Serifa, Helvetica, Times, Universal • D1 2/23/11 RI64612 • D1a 2/28/11 RI64612 • D1b 3/1/11 RI64612 • D1c 3/11/11 RI64612 • D2 3/15/11 RI64612 • D2a 3/16/11 RI64612 • D3 3/17/11 RI64612

German Language and Culture118

CharacteristicsofaGOODResponse(4)

The presentation is organized, and addresses the topic effectively, including some supporting details and mostly relevant examples such as explaining the use of Facebook in Germany and the U.S.The presentation is fully understandable with some errors that do not impede comprehension. Pronunciation, intonation, and pacing make the presentation mostly comprehensible, some occasional errors do not impede comprehension.The student uses the informal register when addressing the audience of his or her classmates, except for an occasional shift, for example Hören Sie nicht auf deine Eltern!The student demonstrates accurate use of the present tense, and mostly accurate use of other time frames. The vocabulary is generally appropriate and might include some idiomatic expressions.

CharacteristicsofaFAIRResponse(3)

The presentation addresses the topic, including a few supporting details and examples. Pronunciation, intonation, and pacing make the presentation generally understandable, although there are errors that might impede comprehensibility.The student demonstrates some control of grammar and syntax with mostly accurate use of the present tense. The student might shift register several times. He or she might start off with the appropriate informal address, but then switch to the formal form. The student might also mix possessive pronouns.The student demonstrates mostly accurate use of the present tense, and some accuracy in other time frames. The student uses sufficient vocabulary, including a few idiomatic expressions.

01443-87558 APSP German Practice Exam • InDS4 • Fonts: Minion Pro Serifa, Helvetica, Times, Universal • D1 2/23/11 RI64612 • D1a 2/28/11 RI64612 • D1b 3/1/11 RI64612 • D1c 3/11/11 RI64612 • D2 3/15/11 RI64612 • D2a 3/16/11 RI64612 • D3 3/17/11 RI64612

German Language and Culture 119

Scoring Guidelines for Presentational Speaking: Cultural Comparison

5Strong

• Thorough and effective treatment of topic, including supporting details and relevant examples

• Fully understandable, with ease and clarity of expression; occasional errors do not impede comprehensibility

• Varied and appropriate vocabulary and idiomatic expressions

• Control of time frames; accuracy and variety in grammar, syntax, and usage, with few errors

• Consistent use of register appropriate for audience • Pronunciation, intonation, and pacing make the

response comprehensible; errors do not impede comprehensibility

• Organized presentation; effective use of transitional elements or cohesive devices

• Clarification or self-correction (if present) improves comprehensibility

4Good

• Effective treatment of topic, including some supporting details and mostly relevant examples

• Fully understandable, with some errors, which do not impede comprehensibility

• Generally appropriate vocabulary, including some idiomatic expressions

• Accurate use of present time and mostly accurate use of other time frames; general control of grammar, syntax, and usage

• Consistent use of register appropriate for audience except for occasional shifts

• Pronunciation, intonation, and pacing make the response mostly comprehensible; errors do not impede comprehensibility

• Organized presentation; some effective use of transitional elements or cohesive devices

• Clarification or self-correction (if present) usually improves comprehensibility

01443-87558 APSP German Practice Exam • InDS4 • Fonts: Minion Pro Serifa, Helvetica, Times, Universal • D1 2/23/11 RI64612 • D1a 2/28/11 RI64612 • D1b 3/1/11 RI64612 • D1c 3/11/11 RI64612 • D2 3/15/11 RI64612 • D2a 3/16/11 RI64612 • D3 3/17/11 RI64612

German Language and Culture120

3Fair

• Competent treatment of topic, including a few supporting details and examples

• Generally understandable, with errors that may impede comprehensibility

• Sufficient vocabulary, including a few idiomatic expressions

• Mostly accurate use of present time and some accuracy in other time frames; some control of grammar, syntax, and usage

• Use of register may be inappropriate, several shifts occur• Pronunciation, intonation, and pacing make the

response generally comprehensible; errors occasionally impede comprehensibility

• Some organization; limited use of transitional elements or cohesive devices

• Clarification or self-correction (if present) sometimes improves comprehensibility

2Weak

• Inadequate treatment of topic, consisting mostly of statements with no development; examples may be inaccurate

• Partially understandable, with errors that force interpretation and cause confusion for the listener

• Limited vocabulary and idiomatic expressions • Some accuracy in present time and little or no accuracy

in other time frames; limited control of grammar, syntax, and usage

• Use of register is generally inappropriate for audience • Pronunciation, intonation, and pacing make the

response difficult to comprehend at times; errors impede comprehensibility

• Inadequate organization; ineffective use of transitional elements or cohesive devices

• Clarification or self-correction (if present) usually does not improve comprehensibility

01443-87558 APSP German Practice Exam • InDS4 • Fonts: Minion Pro Serifa, Helvetica, Times, Universal • D1 2/23/11 RI64612 • D1a 2/28/11 RI64612 • D1b 3/1/11 RI64612 • D1c 3/11/11 RI64612 • D2 3/15/11 RI64612 • D2a 3/16/11 RI64612 • D3 3/17/11 RI64612

German Language and Culture 121

1Poor

• Little or no treatment of topic; may not include examples• Barely understandable, with frequent or significant

errors that impede comprehensibility • Very few vocabulary resources • Little or no control of grammar, syntax, usage, and time

frames • Little or no control of register • Pronunciation, intonation, and pacing make the

response difficult to comprehend; errors impede comprehensibility

• Little or no organization; absence of transitional elements and cohesive devices

• Clarification or self-correction (if present) does not improve comprehensibility; does not recognize errors

0Unacceptable

• Mere restatement of language from the prompts• Clearly does not respond to the prompts; completely

irrelevant to the topic• “Ich weiß nicht.” “Ich verstehe nicht.” or equivalent • Not in the language of the exam • Blank (although recording equipment is functioning)

01443-87558 APSP German Practice Exam • InDS4 • Fonts: Minion Pro Serifa, Helvetica, Times, Universal • D1 2/23/11 RI64612 • D1a 2/28/11 RI64612 • D1b 3/1/11 RI64612 • D1c 3/11/11 RI64612 • D2 3/15/11 RI64612 • D2a 3/16/11 RI64612 • D3 3/17/11 RI64612

German Language and Culture122

Contact Us AP Services P.O. Box 6671 Princeton, NJ 08541-6671 609-771-7300 888-225-5427 (toll free in the U.S. and Canada) 610-290-8979 (Fax) E-mail: [email protected]

National Office 45 Columbus Avenue New York, NY 10023-6992 212-713-8000

AP Canada Office 2950 Douglas Street, Suite 550 Victoria, BC, Canada V8T 4N4 250-472-8561 800-667-4548 (toll free in Canada only) E-mail: [email protected]

International ServicesServing all countries outside the U.S. and Canada45 Columbus AvenueNew York, NY 10023-6992212-373-8738E-mail: [email protected]

apcentral.collegeboard.com I.N. 110084381